Sunteți pe pagina 1din 94

Universidad La Salle.

Facultad Mexicana de Medicina.


Curso de Extensin Universitaria para la Preparacin del Examen Nacional para
Aspirantes a Residencias Mdicas.

Examen de Ginecologa y Obstetrcia.


1.- Se trata de paciente femenino de 29 aos de edad con tumor anexial de 6 cm lquido, dolor
abdominal, fiebre, leucorrea, con historia de cervicovaginitis de repeticin y dispareunia
crnica, ltima menstruacin hace una semana. El diagnstico ms probable de sta paciente
es:
a)
b)
c)
d)

Cistoadenoma
Enfermedad plvica inflamatoria
Embarazo ectpico
Quiste de ovario

La EIP puede cursar con los siguientes sntomas:

Dolor abdominal bajo (incluyendo dolor anexial, dispareunia). Es el sntoma ms


frecuente (95%)
Aumento del flujo vaginal, flujo de caractersticas anormales (74%)
Sangrado anormal (intermestrual, poscoital) (45%)
Sntomas urinarios (35%)
Vmitos (14%)
Es posible la ausencia de sntomas

Y en ella podemos encontrar estos signos:

Dolor a la movilizacin del cuello, dolor anexial en la exploracin vaginal bimanual


(99%)
En el examen con espculo observamos cervicitis y descarga endocervical purulenta
(74%)
Fiebre (> 38 C) (menos del 47%).
Masa plvica: sugiere abceso tuboovrico (ATO)
Peritonitis

CRITERIOS CLINICOS PARA EL DIAGNOSTICO DE SALPINGITIS


a. Dolor abdominal con o sin rebote.
b. Sensibilidad a la movilizacin del crvix.
c. Sensibilidad anexial.
Los tres criterios anteriores son necesarios para establecer el diagnstico, con uno o ms de
los siguientes:

a. Extendido de Gram de endocrvix positivo, para diplococos gram negativos


intracelulares
b. Temperatura mayor de 38C
c. Leucocitosis (mayor de 10.000 por c.c.)
d. Material purulento (positivo para leucocitos) en la cavidad peritoneal obtenido
por culdocentesis o laparoscopia.
Establecido el diagnstico clnico de EPI, se debe hacer la definicin del estado clnico y
anatmico de la patologa plvica:
a) No complicada (limitada a trompas u ovarios)
1) Sin peritonitis plvica
2) Con peritonitis plvica
b) Complicada (masa inflamatoria o absceso que compromete trompa (s) u ovario (s)
1) Sin peritonitis plvica
2) Con peritonitis plvica
o
o
o
o
o
o
o

Beigi RH, Wiesenfeld HC. Pelvic inflammatory disease: new diagnostic criteria and
treatment. Obstet Gynecol Clin Norh Am. 2003; 30 (4): 777 93
Center for Disease Control. Guidelines for treatment of sexually transmited diseases.
MMWR Recomm Rep. 2002 May 10;51(RR-6):1-78
Center for Disease Control. Guidelines for prevention and management (MMWR. 40: 1 25 1991) Pelvic inflammatory disease: guidelines for prevention and management.
MMWR Recomm Rep. 1991 Apr 26;40(RR-5):1-25.
Hager WD, Eschenbach DA, Spence MR, Sweet RL. Criteria for diagnosis and grading
of salpingitis. Obstet Gynecol. 1983 Jan;61(1):113-4.
Prodigy Guidance. Pelvic inflammatory disease. [Internet]. UK : NHS, Department of
Health; 2003. [Acceso 18 de Junio de 2005]. Disponible en:
Ross J. Pelvic inflammatory disease. Clin Evid. 2004 Dec;(12):2259-65.
Royal College of Obstetricians and Gynecologists. Pelvic Inflammatory Disease.
Guideline n 32. [Internet]. RCOG; Mayo 2003. [Acceso 18 de Junio de 2005].

2.-Femenino de 25 aos con antecedentes de G/2, P/1, C/1 acude al servicio de consulta
externa, refiere que presenta una secrecin transvaginal bastante lquida, de baja viscosidad,
maloliente de color amarillo y gris, espumoso.
El tratamiento de eleccin para esta entidad es:
a) Metronidazol 500 mg. VO c/12 por 7 a 10 das
b) Ampicilina 1g VO c/ 6 hrs.
c) Clotrimazol + Metronidazol 500 mg VO c/12 hrs. por 7 a das
d) Clindamicina 300 mg VO cada 12 hrs. x 5 das.

Referencias bibliogrficas:
1. Secretara de Salud. Norma Oficial Mexicana NOM -039-SSA2-2002, Para la prevencin y
control de las infecciones de transmisin sexual. D.O.F. 19 de Septiembre 2003.
2. Kettler H, White K, Hawkes S. Mapping the landscape for sexually transmitted infections:
key findings and recommendations. Geneva, TDR (TDR/STI/ IDE/04.1).
3. CDC. Trends in Reportable Sexually Transmitted Diseases in the United States. CDC,
National Report. 2004
4. Distribucin de los casos nuevos de enfermedades por mes Estados Unidos Mexicanos
2004. Sistema nico de Informacin para la Vigilancia Epidemiolgica/Direccin General
de Epidemiologa/SSA
5. Aral S O. Sexual risk behavior and infection: epidemiological considerations. Sex. Transm.
Inf. 2004;80:8-12

3.- Femenino de 25 aos, con embarazo de trmino, sin antecedentes de control prenatal. G 3.
C-1. Se ingresa al servicio de obstetricia por presentar actividad uterina regular y dolorosa. Ef.:
Deambulante, tranquila, adecuada coloracin de tegumentos, abdomen con fondo uterino a 32
cm. con producto nico vivo en situacin transversa dorso inferior FCF 144, al tacto vaginal
crvix dilatado a 3 cm. y membranas ntegras. Se realiza cesrea con retencin de placenta e
invasin a vejiga.
Estudio de gabinete de eleccin previo al evento obsttrico es:
a)
b)
c)
d)

prueba sin estrs


perfil biofsico
simple de abdomen
ultrasonido doppler

PLACENTA:
Se divide en:
Placenta acreta.
Placenta increta.
Placenta percreta.

ACRETA: Las vellosidades se adhieren al miometrio.

INCRETA: Penetran ms de la mitad del espesor del miometrio.

PERCRETA: Atraviesa todo el espesor del miometrio, llegando a la


serosa, incluso atravesndola y adhirindose a rganos vecinos.

Factores:
Endometrisis previa.
Tumores submucosos. (Miomas)
Cicatrz uterina previa. (Cesrea, miomectoma)
Implantacin baja. (Placenta previa)
Malformaciones placentarias. (Placenta extracorial)
Legrado enrgico previo.
Extraccin manual previa de una placenta.
Diagnstico Pre-parto

Ultrasonografa Dppler.

Resonancia Magntica.

Diagnstico transparto-:
Placenta retenida por ms de 20 minutos.
Imposibilidad para encontrar un plano de separacin placentaria cuando se intenta su
extraccin manual.
Hemorragia incontrolable despus de la pseudoextraccin.
El diagnstico histopatolgico corrobora el diagnstico clnico.
Escenario menos deseable.
Tratamiento:
Histerectoma Obsttrica.
Constituye una ciruga no planeada y secundaria al hallazgo del acretismo
placentario con sangrado incohercible.

Cesrea-Histerectoma. (Con diagnstico previo)


Ciruga planificada ante un correcto diagnstico prenatal.

Recomendacin ACOG:
Maduracin pulmonar intrauterina.
Inyectar al cordn umbilical 50 mg de metrotexate.
Ligar el cordn en el nacimiento placentario y dejar la placenta in-situ.
Embolizacin inmediata de arterias uterinas bilaterales, as como de ramas de
la divisin anterior de la arteria iliaca interna con alcohol polivinlico.
Continuar con 5 dosis I.M. de 50 mg de metrotexate y cuantificar niveles de
hCG.
Programar Histerectoma Total Radical Abdominal y/o Cistectoma parcial y/o
reseccin pared anterior recto.

Lee et al. Conservative Management of Placenta Percreta. Obstet Gynecol, 112(2):421-424

4.- Mujer que acude al servicio de ginecologa por referir ciclos opso-menorreicos desde el
inicio de su menarquia, ha incrementado 15 Kg. de lo que pesaba habitualmente, se aprecia
una gran cantidad de acn, pero adems refiere depilarse el rea del bigote cada semana, y
cree que esto le sensibiliza la piel para que aumente el acn.
El diagnstico ms probable en esta paciente es:
a)
b)
c)
d)

Sx. De Asherman
Sx. Stein Leventhall
Sx. Amenorrea Galactorrea
Sx. Karman

Sndrome de Ovario Poliqustico (SOP) es uno de los ms comunes trastornos endocrinos que
afectan a las mujeres alrededor del 5% al 10% de las mujeres en edad reproductiva (12-45
aos) y se piensa que es una de las principales causas de la infertilidad femenina. Las
caractersticas principales son la obesidad, anovulacin (dando lugar a la menstruacin
irregular) o amenorrea, acn, y las cantidades excesivas o los efectos de andrognicos
(masculinizantes) hormonas. Los sntomas y la severidad del sndrome varan mucho entre las
mujeres. Si bien las causas son desconocidas, resistencia a la insulina, la diabetes y la
obesidad estn fuertemente correlacionadas con el SOP.
Bulun SE, Adashi EY. The physiology and pathology of the female reporductive axis. In:
Kronenberg HM, Melmed S, Polonsky KS, Larsen PR, eds. Williams Textbook of
Endocrinology. 11th ed. Philadelphia, Pa: Saunders Elsevier; 2008:chap 16.

5.- Se trata de mujer de 43 aos, G- 4, P-3, A-1, con diagnstico de anemia ferropnica, de 9.5
g/dl, refiere ciclos menstruales de 31,32 x 8,9 das de duracin, acompaados de cogulos, los
cuales aparecieron despus del nacimiento de su segundo hijo hace 13 aos. E.F.: Buen
estado general, TA 130/80, genitales con evidencia de sangrado activo, al tacto vaginal se
detecta tero de consistencia firme voluminosa, irregular, aproximadamente de 12 cm. anexos
libres.
El diagnstico ms probable es:
a) Adenomiosis uterina.
b) Cncer cervicouterino.
c) Miomatosis uterina.
d) Hiperpalsia adenomatosa de endometrio.

MIOMATOSIS UTERINA
Definicin:
Tumor benigno que se origina en el miometrio, por lo que su componente histolgico
predominante es el tejido muscular y, en menor medida, el conectivo y fibroso. El nico
tratamiento efectivo es el quirrgico; sin embargo, slo requieren ser tratados aquellos que
producen sntomas.
Evaluacin y Diagnstico:
Historia:
1. El sntoma ms frecuente suele ser la hemorragia uterina.
2. Los sntomas principales estn relacionados con el crecimiento del tumor.
3. La paciente puede notar una masa en hipogastrio o abdomen inferior.

4. La masa se puede asociar a dolor plvico, o manifestaciones por compresin de


rganos o estructuras vecinas.
5. Puede haber alteracin de la fertilidad.
Examen Fsico:
1. Se debe realizar con la vejiga y el recto vacuo.
2. El hallazgo primordial es el aumento de volumen y consistencia del tero, el cual puede
ser simtrico (ndulos submucosos) o irregular (ndulos intramurales o subserosos).
Exmenes Auxiliares:
1. Papanicolau crvicovaginal: Indicado siempre; permite descartar neoplasia epitelial
cervical o cncer infiltrante de crvix.
2. Ultrasonido transabdominal y transvaginal: Indicado siempre; permite evaluar la
localizacin, tamao y nmero aproximado de miomas.
3. Hemoglobina, hematocrito: Indicado cuando hay historia de sangrado; orienta en la
severidad del sangrado y anemia.
4. Hemograma y VSG: Indicado cuando hay historia de fiebre; si es anormal sugiere
infeccin o necrosis del mioma (puede ser apropiado descartar infeccin de otro rgano
o sistema).
5. Grupo sanguneo y factor Rh: Si hay anemia severa o en el preoperatorio.
6. Perfil de coagulacin (tiempo de protrombina y de tromboplastina parcial, recuento de
plaquetas): Si hay historia de sangrado exagerado.
7. Gonadotrofina corinica (subunidad srica): Permite descartar posible embarazo en
casos de ciclos irregulares, retraso menstrual o tero de consistencia blanda.
Diagnstico Diferencial:
1.
2.
3.
4.
5.
6.

Embarazo.
Tumor de ovario.
Enfermedad inflamatoria plvica, complejo inflamatorio anexial plvico.
Endometriosis.
Adenomiosis.
Tumor extragenital: colon, retroperitoneo.

Referencias Bibliogrficas:
1. Hillard PA. Benign Diseases of the Female Reproductive Tract: Symptoms and Signs.
En: Berek JS, Adashi EY, Hillard PA, eds. Novak's Gynecology. Baltimore: Williams and
Wilkins, 1996:331-97.
1. Hutchins FL, Greenber MD. Miomas Uterinos: Diagnstico e Indicaciones de
Tratamiento. Clinicas de Ginecologa y Obstetricia. Temas Actuales. 1995;5:609-14.
2. Davis KM, Sclass WD. Tratamiento Mdico para Miomatosis Uterina. Clinicas de
Ginecologa y Obstetricia. Temas Actuales. 1995;5:671-81.
1. Selwyn P, Oskowitz MB. Leiomyomata Uteri. En: Friedman EA, ed. Gynecological
Decision Making. St. Louis: Mosby, 1983:148-9.
1. Diaz Huamn V. Tumores Benignos del Aparato Reproductor Femenino. En: Ludmir A,
Cervantes R, Castellano C, eds. Ginecologa y Obstetricia, Prevencin - Diagnstico Tratamiento. Lima: Concytec, 1996:907-25.

6.- Femenino de 34 aos que cursa con 38.5 semanas de gestacin que ingresa al servicio
con trabajo de parto. En el transcurso de trabajo de parto durante la dilatacin presenta dolor
intenso y brusco. A la exploracin usted observa metrorragia escasa y aumento del tono
uterino a la palpacin abdominal que resulta muy doloroso. El diagnstico ms probable es:
a) Placenta previa.
b) Rotura de vasos previos.
c) Crioamnionitis hemorrgica.
d) Desprendimiento de placenta.
Fisiopatologa de la hemorragia
La hemorragia es el signo fundamental que domina el cuadro clnico de la placenta previa. Para
explicar su mecanismo existen distintas teoras:
Mecanismo de Jacquemier: Se produce crecimiento armnico de la placenta y del tero hasta
la semana 26, 28. Despus el segmento inferior crece ms deprisa y favorece el
despegamiento lo que origina la hemorragia en el embarazo.
Mecanismo de Schroeder: Las contracciones uterinas en el parto traccionan del segmento
inferior hacia arriba y empujan al feto hacia abajo despegando la placenta.
Mecanismo de Pinard: Explica las hemorragias gestacionales y del parto. El estiramiento de las
membranas de la zona de menor radio (orificio interno cervical) como consecuencia de las
contracciones tira de la placenta y la desprenden.
Mecanismo de Bartholomew: Explica la hemorragia en los casos de placenta previa central. La
zona placentaria que reviste el orificio interno es un rea isqumica ya que no recibe vasos
deciduales. A este nivel disminuye la presin sangunea, por lo que la sangre tiende a dirigirse
hacia esta zona y escapa por la cara materna.
En el alumbramiento tambin puede haber una hemorragia importante producida por un doble
mecanismo:
-desprendimiento parcial antes de la expulsin en los casos de placenta oclusiva.
-atona uterina en la zona de insercin despus de expulsada la placenta y vascularizacin
anmala.
Manifestaciones clnicas de la placenta previa en el embarazo
- Sntomas: Principalmente la hemorragia. Toda hemorragia vaginal acontecida en el tercer
trimestre debe hacer pensar en una placenta previa. Las hemorragias suelen ser espontneas,
no acompaadas de dolor, de sangre roja y se presentan de forma intermitente, con intervalos
variables entre las mismas. Progresivamente se van haciendo ms frecuentes y ms graves. La
primera hemorragia suele aparecer en forma inesperada generalmente nocturna, cesando en
menos de media hora. Las hemorragias ulteriores son ms graves y ms precoces.
- Signos: La consecuencia fundamental es la anemia materna que depende de la cuanta de la
hemorragia (la sangre es de origen materna ya que procede de espacios intervellosos).
- Exploracin:
Exploracin general para valorar la existencia de signos de anemia.
Exploracin obsttrica: valorar el tamao del tero (adecuado para la edad gestacional), es
blando e indoloro. A menudo la esttica fetal est alterada (transverso, oblicuo, nalgas). No
debe efectuarse tacto vaginal cuando haya existido hemorragia en embarazo avanzado por el
riesgo de infeccin y de despegamiento y aumentar as la hemorragia.
Auscultacin fetal normal.
Exploracin ecogrfica: es una tcnica fundamental en el diagnstico de la placenta previa.
Permite determinar la localizacin placentaria y la variedad de la placenta previa. En general, la
placenta puede identificarse a partir de la 9 semana. No obstante, a lo largo de la gestacin

por crecimiento uterino se produce un cambio en sus relaciones con el tero ("emigracin
placentaria", imagen de desplazamiento). As el diagnstico de certeza de lmites placentarios
solo puede establecerse hacia la semana 34. Siempre ser necesario hacer una adecuada
identificacin del orificio cervical interno (ms fcil con sonda transvaginal). Aadiendo al
estudio Doppler color se observa la vascularizacin y las zonas que sangran.

7.- Se trata de femenino de 24 aos de edad gesta 1, tuvo un parto vaginal espontneo con un
producto con peso de 4,350 g. despus de 5 minutos de traccin suave del cordn umbilical se
expuls la placenta, que parece estar intacta. Se inici el masaje del fondo uterino y se pidi a
la enfermera que administrara 20 unidades de oxitocina en 100 ml de solucin Ringer lactato.
Despus de una inspeccin cuidadosa del canal del parto se observa una laceracin de
segundo grado y una laceracin de 2 cm en la pared vaginal izquierda que se intent reparar.
En la exploracin fsica se encuentra un fondo uterino blando y atnico. Los signos vitales son:
temperatura 37.1C, TA 164/92, FC 130x, FR 18 X. El tratamiento ms adecuado en ste
caso es:
(a) Oxitocina 10 unidades directas en goteo intravenoso
(b) Metilergonovina 0.2 mg IM
(c) Prostaglandina F 0.25 mg IM
(d) Legrado

Morgan M, Siddighi S. Ginecologa y obstetricia, National Medical Series. 5 edicin. Mc Graw


Hill. Pp. 28. La atona uterina es la causa ms comn de hemorragia puerperal. El masaje
energtico y la oxitocina diluida no han sido tiles para interrumpir la hemorragia y por tanto el
siguiente paso es agregar un frmaco uterotnico. La metilergonovina est contraindicada
porque la paciente se encuentra hipertensa a pesar de la hemorragia intensa, el siguiente
frmaco es la prostaglandina. La administracin de oxitocina no diluida, 10 UI por va IV podra
causar hipotensin grave. La exploracin manual podra ser apropiada si se sospecha
laceracin como causa de hemorragia. El legrado es apropiado para la hemorragia puerperal
tarda, cuando se sospecha retencin de los productos de la concepcin.
8.- Se trata de paciente femenino de 20 aos de edad, que acude a consulta refiriendo
presentar 6 meses sin regla. Se manifiesta preocupada por su sobrepeso, ha estado a dieta y
ha perdido 6 kg en 8 meses. Actualmente pesa 46 Kg. con talla de 1,65 mts. Signos vitales
dentro de sus parmetros normales. Se realiza prueba de embarazo con resultado negativa. La
causa ms probable de su amenorrea es:
a) Disgenesia gonadal.
b) Hipogonadismo hipogonadotropo.
c) Sndrome de ovario poliqustico.
d) Adenoma hipofisario.

Sigue siendo prctica y eficaz la diferenciacin de los hipogonadismos en hipergonadotrpicos


(con fallo de funcin en la gnada) e hipogonadotrpicos (con fallo en hipotalmo y/o hipfisis).
La deficiencia puede ser total o parcial y afectar a una o ambas funciones gonadales
(produccin de esteroides y germinal). Las causas son muy variadas. En la tabla 1 se han
recopilado las principales de origen hipogonadotrpico, aunque algunas de ellas raramente se
observan durante la infancia (p. ej., hiperprolactinemia) y en la tabla 2 las de origen
hipergonadotrpico. Dentro del apartado de idioptico poco a poco se van dilucidando las

causas y, as, con la ayuda de la biologa molecular se encuentran mutaciones del gen del
receptor de GnRH, de LH o alteraciones del gen DAX 1. Por otra parte debe tenerse en cuenta
que en diferentes sndromes puede aparecer un hipogonadismo hipogonadotrpico (tabla 3).
MANIFESTACIONES CLNICAS Y ANALTICAS La ausencia de datos clnicos de
hipogonadismo durante la infancia es la regla. Tan slo la presencia de micropene o
criptorquidia en algn varn puede hacer sospechar de la existencia de un hipogonadismo,
pero ni es patognomnica (se da tambin en el dficit de GH y en otras situaciones no
hormonalmente deficitarias), ni su ausencia excluye la deficiencia gonadal. Por lo tanto, y si se
descarta la sospecha del hipogonadismo por otras caractersticas asociadas (p. ej., diversos
sndromes, alteraciones evidentes: tumoraciones o radioterapia hipotlamo-hipofisaria), slo el
retraso de aparicin de la pubertad ser la primera indicacin de una deficiencia de estas
caractersticas. Sin embargo, aqu se produce el principal problema para alcanzar un
diagnstico temprano al resultar muy difcil distinguirlo de una variante normal del desarrollo: el
retraso constitucional del crecimiento y adolescencia. Respecto al hipogonadismo
hipogonadotrpico los estudios hormonales son escasamente discriminatorios en la poca
previa a la pubertad por lo que el momento en que se realiza el diagnstico, y por lo tanto el
inicio del tratammiento, suele ser tardo. La ausencia de caracteres sexuales secundarios, junto
con un tamao infantil de las gnadas (por palpacin testicular o ecografa ovrica), es
suficiente para valorar una situacin prepuberal. En todo caso, la confirmacin, segn el sexo,
de bajos niveles de testosterona circulante, o de estrgenos (directamente o por citologa
vaginal/vesical), permiten una sencilla, fcil y rpida comprobacin. No debe olvidarse que en
relacin a la causa del hipogonadismo (p. ej., una tumoracin hipofisacualquier edad, es
objetivar una anosmia (sndrome de Kallmann), ms an si es parcial. En los casos de
hipogonadismo hipergonadotrpico puede encontrarse en ciertos periodos a lo largo de la
infancia niveles elevados de gonadotrofinas basales y que tras estmulo con GnRH muestran
una respuesta ms elevada de lo normal. Sin embargo, lo ms habitual es encontrarse con
niveles propios de la infancia al realizar las determinaciones basales. Recientemente se ha
demostrado por medio de una tcnica muy sensible que no era cierta la pretendida ausencia de
estradiol en las nias, por lo que si se consigue generalizar su empleo ser un arma valiosa de
diagnstico durante la infancia.
TABLA 1. Causas de hipogonadismo hipogonadotrpico
Panhipopituitarismo
Idioptico
Tumoracin hipofisaria o hipotalmica
Miscelneo
Enfermedad granulomatosa
Traumatismo
Vasculitis
Infarto
Hemocromatosis
Deficiencia aislada de gonadotrofinas
Sndrome de Kallman y variantes
Hipogonadismo hipotalmico idioptico
Deficiencia aislada de LH
Deficiencia aislada de FSH
Hipoplasia adrenal congnita ligada al cromosoma X
Defectos de desarrollo de la lnea media
Sndromes con diversas malformaciones (tabla 2)
Prader-Willi
Laurence-Moon-Bardet-Biedl
Alteraciones sistmicas
Deficiencias nutritivas
Enfermedades crnicas
Ponderales: desnutricin severa y obesidad mrbida
Yatrogenia
Farmacolgica
Radioterpica
Quirrgica
Hiperprolactinemia

Ejercicio fsico de alta competicin


Retraso constitucional de la pubertad (?)

BIBLIOGRAFA SELECCIONADA:
Henkin RI, Bartter FC Studies on olfactory thresholds in normal man and in patients
with adrenal cortical insufficiency: The role of adrenal cortical steroids and of serum
sodium concentration. J Clin Invest 1966; 45: 1631-1639. 2. Schwanzel-Fukuda M,
Abraham S, Crossin KL, Edelman GM, Pfaff DW. Immunocytochemical demonstration
of neural cell adhesion molecule (NCAM) along the migration route of luteinizing
hormone-releasing hormone (LHRH) neurons in mice J Comp Neurol 1992; 321: 1-18.
3. Schwanzel-Fukuda M, Bick D, Pfaff DW: Luteinizing hormone-releasing hormone
(LHRH)-expressing cells do not migrate normally in an inherited hypogonadal
(Kallmann) syndrome. Mol Brain Res 1989; 6: 311-326.

9.- Se trata de paciente femenino de 18 aos de edad la cual acude a consulta por presentar
amenorrea acompaada de profundas alteraciones del olfato. De los siguientes el diagnstico
ms probable es:
a) Sndrome de amenorrea-galactorrea.
b) Amenorrea de causa uterina.
c) Sndrome de ovario poliqustico.
d) Amenorrea por alteracin hipotalmica.
AMENORREA HIPOTALAMICA. (Hipogonadotrpica)
Psicgena. (Stress emocional)
Anorexia nerviosa. (Deficiencia nutricional)
Ejercicio excesivo. (Carrera de fondo, natacin, gimnasia, ballet)
Frmacos. (Fenotiazina, reserpina, bloqueadores ganglionares,
anticonceptivos)
Pseudocisis.
Sndrome de Kallman (Deficiencia selectiva de gonadotropinas y anosmia).
El ejemplo clsico de la alteracin hipotalmica que lleva a desrdenes del ciclo menstrual es
el Sndrome de Kallman.

Lectura Recomendada:
Etiopatogenia de la amenorrea hipotalmica funcional Interaccin de las respuestas
hormonales del Sistema Nervioso Central y Neuropptidos Perifricos
Revista Argentina de Endocrinologa y Metabolismo
Copyright 2008 por la Sociedad Argentina de Endocrinologa y Metabolismo
Vol 45 No. 2

10.- Una mujer de 27 aos G/4, P/0 con 6 SDG acude a su primera visita prenatal. Su historia
obsttrica pasada es importante porque tiene tres prdidas de producto en el segundo
trimestre. Refiere que en las tres ocasiones al presentarse al hospital presentaba dilacin
cervical completa. No recuerda haber tenido contracciones dolorosas. Niega antecedentes
mdicos y quirrgicos. El examen fsico es normal incluyendo un examen plvico que muestra
un cervix largo y cerrado. Despus de una larga discusin con la paciente ella pide que se le
practique un cerclaje durante este embarazo. El momento ms apropiado para realizarlo es:
a)
b)
c)
d)

Inmediatamente
13 a 17 semanas
24 a 28 semanas
32 a 36 semanas

El cerclaje cervical tiene sus indicaciones en la profilaxis y tratamiento de la incompetencia


cervical.
La incompetencia o insuficiencia cervical representa un 10% de las causas de parto pretrmino
y est asociada a una importante morbimortalidad neonatal.
Las modificaciones cervicales en el segundo trimestre de gestacin son causa de parto
prematuro y pueden deberse a:
1) Incompetencia cervical.
2) Prdida de tejido conectivo tras una ciruga cervical (conizacin).
3) Defectos congnitos como la hipoplasia cervical tras exposicin a dietilestilbestrol.
4) Infeccin intrauterina. Hasta un 51.5% de las pacientes con clnica compatible con
incompetencia cervical enmascaran un cuadro de infeccin intraamnitica subclnica.
Diferenciamos tres tipos de cerclaje:
1. El cerclaje se considera profilctico o electivo (o primario) cuando se realiza de forma
electiva por historia previa de incompetencia cervical antes de evidenciar cambios en el cerviz
y generalmente suele realizarse entre las 13 y 17 semanas de gestacin.
2. El cerclaje teraputico secundario que se realiza tras la deteccin, en el seguimiento
obsttrico, de modificaciones en el crvix antes de las 26 semanas de gestacin. Se realiza en
pacientes con un riesgo potencial de parto pretrmino.
3. El cerclaje teraputico terciario, en caliente, de rescate o emergent cerclage que se
realiza en pacientes que presentan la membrana amnitica visible a travs del orificio cervical
externo o en vagina.
GUIA CLNICA:
INDICACIONES DEL CERCLAJE
Unitat de Prematuritat. Servei de Medicina Maternofetal.
Institut Clnic de Ginecologia, Obstetrcia i Neonatologia, Hospital Clnic de Barcelona
Responsables del protocolo: T.Cobo, M. Lpez, M. Palacio
Creacin: 24/01/07
Modificaciones: 05/09/07
ltima actualizacin: 17/01/10

11.- Se trata de femenino de 67 aos refiere intenso prurito vulvar y sensacin quemante, al
examen el introito vaginal se encuentra estentico. De los siguientes es el tratamiento ms
apropiado es:
a)
b)
c)
d)

5-fluoracilo
Testosterona tpica
Corticoesteroides fluorados
Estrgeno tpico

Vulvovaginitis atrfica
El hipoestrogenismo conduce a atrofia de la vagina y el vestbulo vulvar, que los hace
fcilmente irritables y susceptibles a infecciones secundarias. Las pacientes refieren sensacin
de quemadura, prurito, disuria, hipersensibilidad y dispareunia. Puede encontrarse al examen
fsico atrofia, fisuras superficiales, y un flujo vaginal acuoso1. Hay disminucin del tamao del
introito2, prdida de la rugosidad y la vagina toma una apariencia lisa y brillante.
Los hallazgos histolgicos revelan un epitelio vaginal delgado, disminucin de los lechos
capilares, y la citologa muestra, a medida que la atrofia progresa, aumento de las clulas
basales y disminucin o ausencia de las clulas superficiales2.
Se aconseja evitar el uso de jabones y dems irritantes de la piel. Se pueden utilizar lubricantes
simultneamente con los estrgenos o como terapia nica, si hay alguna contraindicacin a las
hormonas.
El tratamiento con estrgenos por va sistmica o transvaginal mejora y restaura los signos y
sntomas, y una a dos semanas despus de iniciar el tratamiento los cambios de atrofia
empiezan a mejorar rpidamente, se reduce el pH y se induce maduracin vaginal y de la
mucosa uretral, reduciendo la frecuencia de las infecciones urinarias3. La dosis y va de
administracin debe ser debidamente individualizada4. Contraindicaciones al tratamiento con
estrgenos, incluyen: la presencia de tumores estrgenosensibles, falla heptica terminal y
antecedentes de tromboembolizacin relacionada con ellos.
Menopausia y Piel. Parte II: Manifestaciones clnicas
dermatolgicas durante la menopausia
MARA ISABEL BARONA C. Docente adjunto. Dermatloga Universidad del Valle-Cali.

12.- Mujer de 26 aos, es atendida en consulta en la clnica de displasias por papanicolau con
lesin NIC I. Antecedentes: menarca 14 aos, ritmo 30x5 eumenorreica, inicio de vida sexual a
los 15 aos, 2 parejas sexuales, mtodo de planificacin familiar oclusin tubaria bilateral,
gestas 3 partos 3, crvix con lesin acetoblanca con extensin lineal de 2 cm. Para confirmar
el diagnstico se debe realizar:

a)
b)
c)
d)

Crioterapia de lesin.
Captura de hbridos.
Repetir colposcopa
Biopsia de la lesin.

9.5.2 Las pacientes a quienes se les realiz citologa cervical, cuyo resultado es LEIBG
(infeccin por VPH, displasia leve o NIC 1); LEIAG (displasia moderada y grave o NIC 2 y 3) o
cncer deben enviarse a una clnica de colposcopa, para realizar estudio colposcpico.
9.5.3 Si el resultado de la citologa es LEIBG, la colposcopa es satisfactoria y sin evidencia de
LEIBG, se realizar control citolgico en un ao (Apndice Normativo A)
9.5.4 Si la citologa es de LEIBG, la colposcopa es satisfactoria y existe evidencia de
lesin, se debe tomar una biopsia dirigida.

9.5.4.1 Si la biopsia dirigida es negativa, se realizar nueva colposcopa para verificar el


diagnstico y en caso necesario, tomar nueva biopsia dirigida y revalorar.
9.5.4.2 Si la biopsia dirigida es reportada como LEIBG se podr dar tratamiento conservador:
criociruga, electrociruga o laserterapia (slo si cumple con las condiciones referidas en el
Apndice 1) o se podr mantener a la paciente en vigilancia en la clnica de colposcopa, con
colposcopa y estudio citolgico cada seis meses, durante 24 meses.
Jueves 31 de mayo de 2007 DIARIO OFICIAL (Primera Seccin)
9.5.4.3 Si la biopsia dirigida es reportada como LEIAG (Lesin Intraepitelial Escamosa de Alto
Grado) se realizar tratamiento conservador (electrociruga o laserterapia). En las mujeres
posmenopusicas, dependiendo de las condiciones anatmicas del crvix, se realizar
tratamiento conservador en la clnica de colposcopa o tratamiento quirrgico (histerectoma
extrafascial) en el servicio que corresponda.
9.5.4.4 Si la biopsia dirigida reporta cncer microinvasor o invasor, la paciente se transferir a
un Servicio o Centro Oncolgico para su tratamiento correspondiente.
9.5.4.5 Si la citologa reporta LEIBG y la colposcopa es no satisfactoria, se tomar cepillado
endocervical (Apndice Normativo A)
9.6 En caso de colposcopa no satisfactoria, negativa a LEIBG y con cepillado endocervical
negativo, se continuar su control en la clnica de colposcopa en seis meses, con colposcopa
y citologa.
9.6.1.1 Si el cepillado endocervical reporta LEIBG se tratar a la paciente como LEIAG, con
mtodos conservadores escisionales.

Jueves 31 de mayo de 2007 DIARIO OFICIAL (Primera Seccin)


Modificacin a la Norma Oficial Mexicana NOM-014-SSA2-1994, Para la prevencin,
deteccin, diagnstico, tratamiento, control y vigilancia epidemiolgica del cncer
crvico uterino.
Al margen un sello con el Escudo Nacional, que dice: Estados Unidos Mexicanos.- Secretara
de Salud.
MODIFICACION A LA NORMA OFICIAL MEXICANA NOM-014-SSA2-1994, PARA LA
PREVENCION,
DETECCION, DIAGNOSTICO, TRATAMIENTO, CONTROL Y VIGILANCIA EPIDEMIOLOGICA
DEL CANCER CERVICO UTERINO.

7.3 El resultado del estudio citolgico es descriptivo y debe ser informado de la siguiente
manera:
a.- Negativo a cncer.
b.- Negativo con proceso inflamatorio.
c.- Displasia leve (NIC 1).
d.- Displasia moderada (NIC 2).
e.- Displasia grave (NIC 3).
f.- Cncer del cuello del tero in situ (NIC 3).
g.- Cncer microinvasor e invasor.
h.- Adenocarcinoma.
i.- Maligno no especificado

13.- Se trata de paciente de 33 aos que cursa con 39 SDG; a la exploracin fsica reflejos
patelares hiperactivos, inquieta, se reportan cifras de TA 145/95, se realiza laboratorio que
reporta proteinuria 2+,. El diagnstico ms probable es:
a) Glomerulonefritis aguda
b) Hipertensin esencial
c) feocromocitoma
d) Preeclampsia
La hipertensin es la complicacin mdica ms comn del embarazo , aunque para algunos
autores es la segunda complicacin mdica del embarazo slo despus de la anemia; es ms
frecuente en jvenes durante el primer embarazo y en nulparas de mayor edad, hipertensas
previas y diabticas.
En Mxico, tambin es la complicacin ms frecuente del embarazo, la incidencia es de 47.3
por cada 1 000 nacimientos y es adems, la primera causa de ingreso de pacientes
embarazadas a las unidades de terapia intensiva (debido a hemorragia masiva, para recibir
soporte hemodinmico), segn la secretara de salud (2001) la mortalidad por complicaciones
del embarazo ocupa el 15 lugar en la mortalidad hospitalaria en general. Adems, la tasa de
preeclampsia se ha incrementado 40% en el periodo entre 1990 y 1999 y constituye hasta 40%
de los partos prematuros iatrognicos.
.

Preeclampsia
La preeclampsia es un sndrome clnico caracterizado por hipertensin con disfuncin orgnica
mltiple, proteinuria, edemas.
Es definida como un incremento de al menos 140/90 mmHg despus de la semana 20 de
gestacin, un incremento en la presin sangunea diastlica de al menos 15 mmHg respecto a
un nivel previo a la semana 20 combinado con proteinuria (> 300 mg en 24 horas). Las
mediciones de la presin arterial citadas deben ser medidas al menos 2 ocasiones con por lo
menos 6 horas de separacin. La proteinuria puede ser una toma simple de orina al azar que
indique al menos 30 mg/dL 3 ++ en dos muestras de orina1 segn el tipo de prueba. El criterio
del incremento de 30 mmHg en la presin sistlica y/o 15 mmHg en la presin diastlica
respecto a valores previos a la semana 20 de gestacin ha sido eliminado por ser poco
especfico15

Myers JE, Baker PN. Hupertensive diseases and eclampsia. Curr Opin Obstet Gynecol 2002;
14: 119-125
2. Tierney, McPhee, Papadakis. Diagnstico clnico y tratamiento 2003. 38 ed, Mxico, Manual
Moderno, 2003: 770-773
3. Wilson MI, Goodwin TM, Pan VI, Ingles SA. Molecular epidemiology of preeclampsia. Obstet
and Gynecol Survey 2003; 58(1):39-66
4. Burrow GM. Complicaciones mdicas durante el embarazo. 4 ed, Mxico, McGraw-Hill
panamericana: 1996: 1-25
5. Guyton AC, Hall JE. Embarazo y lactancia en: Tratado de fisiologa mdica, 10 ed, Mxico,
McGraw-Hill Interamericana 2001: 1135-45
6. Vaticon D. Fisiologa de la fecundacin, embarazo, parto y lactancia, en: Tresguerres JAF.
Fisiologa Humana. Mxico, Interamericana McGraw-Hill, 1992: 1086-1109

7. Pridjian G, Puschett JB. Preeclampisa. Part 1: Clinical and Pathophysiologic Considerations.


Obstet and Gynecol Survey 2002; 57 (9): 598-618
8. Pridjian G, Puschett JB. Preeclampisa. Part I1: Experimental and Genetic Considerations.
Obstet and Gynecol Survey 2002; 57 (9): 619-40
9. IMSS. Embarazo de alto riesgo. Gua diagnstica teraputica. Rev Med IMSS 1998;
36(1):45-60
14.- Femenino de 36 aos, con presencia de hiper y polimenorreas o prdidas de sangre
contnuas sin conservacin del ciclo, se sospecha de miomatosis uterina, ste tipo alteracin
se presenta con mayor frecuencia en los miomas de localizacin:
a) Submucoso
b) Intramural
d) Intraligamentaria
e) cervical

Descripcin
Los miomas son tumores monoclonales benignos de las clulas del msculo liso del miometrio.
Estn compuestos por grandes cantidades de matriz extracelular que contiene colgeno,
fibronectina y proteoglicanos. El colgeno tipo I y tipo II es abundante, pero las fibrillas de
colgeno son anormales y estn desorganizadas, de modo similar a lo que se observa en la
formacin de queloides.
Los miomas submucosos son los menos frecuentes, constituyendo nicamente el 5% de la
totalidad de los miomas, a menudo producen un aumento del sangrado menstrual en forma de
hiper y polimenorreas, e incluso hemorragias importantes que exigen tratamiento de urgencia.
Las metrorragias tambin son habituales en este tipo de miomas. Segn Novak, el peligro de
degeneracin sarcomatosa es mucho mayor en los miomas submucosos, e igualmente es
causa de dismenorreas ms intensas y frecuentes.

BIBLIOGRAFA
1. De la Fuente U. Tratado de Obstetricia y Ginecologa. Mc Graw-Hill. (Madrid). 1998.
Volumen II.
2. DI SAIA S. Tratado de Obstetricia y Ginecologa de Danforth. Sexta. Mc Graw-Hill. (Nueva
York). 1990. Sexta Edicin.
3. Gonzlez-Merlo J. Tratado de Obstetricia y Ginecologa. Salvat Editores S.A. (Barcelona).
1990. Quinta Edicin.
4. Aller J., Pages G. Obstetricia Moderna. Tercera Edicin. Mc Graw-Hill. (Caracas). 1999.
5. Formacin Mdica Continuada en Atencin Primaria. Marzo 1995. Vo. 5, N (3).
15.- Se trata de mujer de 26 aos, es atendida en consulta por secrecin lctea bilateral hace 4
meses y menstruaciones cada 21 das, en escasa cantidad desde hace 6 meses.
Antecedentes: hace 6 meses padece gastritis tratada con cimetidina durante 2 meses y
posteriormente ha tomado el medicamento de forma irregular. a.g.o.: menarca 12 aos, ritmo
30/7, nbil. e.f.: talla 1.63, peso 54 kg. Al efectuar compresin en glndulas mamarias se
produce salida de secrecin lctea bilateral, resto normal.
El estudio de eleccin en esta paciente para confirmar el diagnstico es:
a)
b)
c)
d)

perfil hormonal ginecolgico.


prolactina srica.
radiografa de silla turca.
ultrasonido mamario.

La hiperprolactinemia (hiperprl) es un trastorno frecuente, ms en la mujer que en el hombre, y


puede ser la manifestacin de un adenoma hipofisario (prolactinoma). La medicin de
prolactina (prl) en la sangre es ahora un procedimiento de rutina en la deteccin de trastornos
de la menstruacin y galactorrea que se resuelven efectivamente con el uso de frmacos con
actividad dopaminrgica (bromocriptina y cabergolina). La causa ms frecuente de hiperprl es
por el uso de frmacos que inhiben la accin de dopamina, pero se debe descartar inicialmente
hipotiroidismo y un prolactinoma mediante resonancia magntica.
Los prolactinomas son fcilmente controlados con dopaminrgicos y excepcionalmente se
requiere ciruga. En caso de infertilidad por exceso de prl existe una respuesta favorable con
bromocriptina o cabergolina, aunque en la actualidad se prefiere la ltima por su efectividad y
menos efectos indeseables.

Hiperprolactinemia. Gua teraputica y diagnstica Arturo Zrate*


* Unidad de investigacin de endocrinologa, diabetes y metabolismo,
Centro mdico nacional IMSS y hospital ngeles Mxico.

16.- Se trata de femenino de 31 aos nuligesta pero con actividad sexual regular, sin mtodo
de planificacin familiar, con ciclos regulares, sin leucorrea, refiere dispareunia profunda,
sangrado intermestrual y dismenorrea secundaria ocacionalmente presenta urgencia urinaria,
el mtodo de eleccin para confirmar el diagnstico clnico de sta paciente es:
a)
b)
c)
d)

Cultivo de exudado vaginal


Ultrasonido
Biopsia de endometrio
Laparoscopia con biopsias dirigidas

Endometriosis
Examinacin bimanual
Dolorenlosligamentos uterosacros
Nodularidad enelfondo desaco
Induracin delsepto rectovaginal
tero enretroversin fijo
Masa anexial

Endometriosis
Laparoscopia
visualizacin directa
estadificacin
toma demuestras

Estadio I Minimo
Estadio II Leve
Estadio III Moderado
Estadio IV Severo

Referencias bibliogrficas

1. Ruiz V. Endometriosis y fertilidad. Ed. Acosta y Warman, pp. 99


2. Lpez, VH. Palomo E. Incidencias de endometriosis en una poblacin infrtil. XXI Congreso
nacional de Ginecologa y Obstetricia. Guatemala, 1993.
3. El-Eoley, et al. Danazol but not ginadotropin releasing hormone agonists suppresses
autoantibodies in endomeriosis. Fertil Steril 1990; 54:725
4. Acosta AA. Buttram VC Jr. Besch PK, Malinak LR, Van Der Heyden J. A.proposed
classfication of pelvic endometriosis. Obstet Gynecol 1973;42:19.
5. Buttran VC Jr. Evolution of the revised American Fertility classification of endometriosis. Fert.
Steril 1985; 43: 347
6. Lpez VH. Tratamiento mdico-quirrgico de la endometriosis. Simposio El rostro cambiante
de la endometriosis panam 3. 12. 1993.

7. Steinleitner A. Heterolous transplation of activated murine peritonel macrophages inhibitis


gamete interaction in vivo; A paradigm fo endometriosis associted subfertility. Fertil Steril 1990;
54:725.
8. Damewood M. Effect of serum from patients with minimal to mild endometriosis on mouse
embryo growth. Fertil Steril 1990; 54: 917
9. Proug S. Peritoneal fluid fracctions from patients with endometriosis do not promote two-cell
mouse embryo growth. Fertil Steril 1990; 54: 927.

17.- Femenino de 22 aos, que presenta una tumoracin de 2 cm de dimetro en el cuadrante


nfero-externo de la mama izquierda, indolora, de consistencia firme, superficie lisa, forma
ovoidea, mvil y bien delimitada del parnquima vecino, sin antecedentes de derrame por el
pezn, sin piel de naranja ni retraccin del pezn. El diagnstico ms probable es:
a)
b)
c)
d)

Fibroadenoma.
Carcinoma.
Ectasia de los conductos mamarios.
Quiste solitario.

FIBROADENOMA MAMARIO
Tumor benigno ms frecuente en las mujeres entre los 20 y 35 aos.
ETIOLOGIA
Existen mltiples teoras siendo la ms aceptada la hormonal, generalmente son nicos, solo el
20% son mltiples o bilaterales. De tamao variable hasta de 10 cm. Ocupa el 13.6% de la
patologa mamaria benigna.
CUADRO CLNICO
Lesin nodular de consistencia dura, de larga evolucin y no dolorosa. Normalmente llegan a
los 3 cm. De dimetro. Durante la fase tarda del ciclo menstrual el tumor suele presentar un
leve aumento de tamao. Durante la menopausia presentan regresin hasta la calcificacin
(signo de palomitas de maz).
DIAGNOSTICO
Es clnico, se presenta como un tumor bien delimitado, desplazable, no adherido a piel ni a
planos profundos, liso o multilobulado en ocasiones. Se localiza frecuentemente en cuadrantes
externos.
EXAMENES DIAGNOSTICOS
ULTRASONIDO MAMARIO .- Identifica un ndulo slido, bien delimitado de bordes regulares .
TRATAMIENTO.Conservador con vigilancia estrecha dependiendo del tamao y en caso de ser necesario
exresis del ndulo para estudio histopatolgico

hospitalgeneral.salud.gob.mx/
BIBLIOGRAFIA

1. Snchez BC. Tratado de Enfermedades de la glndula mamaria. Ed. Manual Moderno. Cap.
13- 15.
2.- De Vita V. Cancer of the Breast. In Cancer: Principles and Practice of Oncology: Fifth Ed.
Philadelphia: Lippincott-Raven, Chapter 36; pp: 1521-1616.
3.-Consenso Nacional Acerca del Tratamiento de Cncer de Mama. En Tumores de mama:
Diagnstico y Tratamiento. 2 Ed. McGraw-Hill Interamericana; pp: 119-126.
4.-Eberlein T. Current management of carcinoma of the breast. Ann Surgery 1994; 220: 121136.
18.- A 26-year-old woman presents with malodorus gray-wellow discharge. You take a wet
mount preparation and observe Clue cells. The agent of this infection and its treatment is:
a)
b)
c)
d)

Gardnerella vaginalis / clindamycin


Gardnerella vaginalis / Ketoconazole
Trichomona vaginalis / metronidazole / treat the partner
Candida albicans / nistatin

Gardnerella vaginalis fue clasificada como una sola especie y fue establecida como agente
causal de la vaginosis (antes conocida como vaginitis inespecfica). El cuadro clnico que
presenta es caracterizado por una secrecin blanca o blanco-griscea que se percibe
generalmente despus de la relacin sexual con olor ftido aminado (pescado). El diagnstico
certero es la base para evitar posibles complicaciones como la enfermedad inflamatoria
pelviana y las complicaciones del embarazo. El tratamiento se basa principalmente en los
frmacos como: metronidazol y clindamicina, debido a su efectividad y espectro, pero como
todos se deben emplear con adecuada prudencia debido a su toxicidad. Adems de que se
deben corregir o modificar los factores predisponentes, ya que esta patologa va en aumento
convirtindose por su frecuencia en un problema de salud pblica.
Bibliografa:
1. Hernndez F. Gardnerella vaginalis mobiluncus en la etiologa de la vaginosis bacteriana.
Rev Costarricense Ciencias Mdicas 1998; 19: 57-61.
2. Hansen EA. Gardnerella. Rev Ginecol 2005; 25: 99.
3. Espinosa I, Lorenzo M, Bentancourt A, Rivern Y, Romero M. Caracterizacin bioqumica y
antignica de diferentes aislamientos de Gardnerella vaginalis.
Rev Cubana Invest Biomed 2005; 24: 22-7.
4. Taylor F. Vaginal flora morphotypic profiles and assessment of bacterial vaginosis in women
at risk for HIV infection.Infect Dis Obstet Gynecol
2004; 12: 121-6.

19.- Una mujer embarazada, puede afectar al feto y hacerlo contraer lesiones importantes
durante el embarazo o al salir al exterior (atravesando el canal de parto), s la gestante se
encuentra afectada de la siguiente patologa:
a) Tricomonas
b) Gardenerella
c) Herpes genital
d) Gonococos

Herpes genital
La prevalencia de herpes simplex genital o tipo 2 (VHS-2) en mujeres embarazadas vara entre
7 y 33% en distintas series. La prevalencia ha experimentado un sostenido aumento durante
los ltimos aos. Se estima que aproximadamente 1 a 3% de las mujeres adquiere cada ao la
infeccin. En el caso de parejas discordantes, la tasa de adquisicin aumenta hasta 10 a 30%
anual. La adquisicin durante el embarazo es ~2%.
La transmisin al hijo ocurre principalmente cuando la mujer embarazada adquiere una
infeccin primaria. La transmisin es de 30 a 50% cuando la infeccin primaria ocurre cerca del
momento del parto. La mayor transmisin (85%) ocurre durante el parto. Sin embargo, tambin
puede ocurrir transmisin intrauterina (5-8%) y post-natal (8-10%). Los factores que inciden en
la transmisin son: infeccin primaria mucho mayor eficiencia que infeccin recurrente, parejas
discordantes, ttulos de anticuerpos maternos y procedimientos obsttricos invasores, (los que
estn absolutamente contraindicados).
Las manifestaciones en la mujer embarazada son principalmente bajo la forma de herpes
genital localizado, muy raramente ocurre diseminacin cutnea y visceral, situacin de elevada
mortalidad (50%). La infeccin en el nio, si ocurre en las primeras 20 semanas del embarazo,
puede provocar aborto en 25%, malformaciones cerebrales, cicatrices, corioretinitis, RCIU. Si
ocurre despus de las 20 semanas, puede causar parto prematuro, RCIU, o herpes neonatal.
Esta condicin clnica tiene tres formas de presentacin, las dos primeras de elevada
mortalidad y secuelas: herpes diseminado y encefalitis herptica o infeccin localizada en piel,
ojo y boca.
Pass R, Weber T, Whitley RJ. Herpesvirus infections in pregnancy.
Recommendations from the International Herpes Management Forum. Management
Strategies Workshop and 7th Annual Meeting.
Whitley R J. Varicella - Zoster virus. Mandell, Douglas and Bennett's Principles and
Practice of Infectious Diseases. Mandell G, Bennett J, Dolin R, editors. Fifth edition,
2000 Churchill Livingstone, Philadelphia, pp: 1586-98.
Abarca K, Cohen B J, Vial P. Seroprevalence of parvovirus B19 in urban Chilean children and
young adults, 1990 and 1996. Epidemiol Infect 2002; 128: 59-62.

20.- Femenino de 20 aos, atendida en sala de urgencias ginecoobsttricas, Antecedente:


cursa embarazo de 38 SDG. Exploracin Fsica: en trabajo de parto. Repentinamente presenta
sangrado profuso transvaginal y dolor abdominal. A la paciente se le coloca un monitor fetal
externo. El tono uterino est incrementado y hay desaceleraciones variables y ocasionales de
la FCF que llegan a 90.
El manejo apropiado en esta paciente es:
a)
b)
c)
d)

Tocolisis con un agonista beta adrenrgico


Continuar el monitoreo de la madre y el producto
Amniotoma
Cesrea

El manejo de stas pacientes es estricto y urgente:


1- Ingreso inmediato de la paciente en vigilancia a sala de partos.
2- Venopuntura. Si existen signos de shock, instaurar preferentemente dos vas, una central y
otra perifrica.
3- Solucin de Ringer.
4- Monitoro de signos vitales cada 15.
5- Sonda vesical y monitoro de diuresis. Diuresis de 50 ml/h asegura perfusin perifrica
correcta.

Evitar diuresis inferiores a 30 ml/h.


6- Monitoro contnuo de LCF.
7- Laboratorio de urgencia: hemograma, ionograma, urea y creatinina, coagulograma.
8- Sangre como mnimo 4U para reserva (concentrado de glbulos rojos o sangre entera).
9- Ecografa obsttrica para localizar y medir el hematoma y descartar otras causas de
sangrado.
Dependiendo del grado de desprendimiento realizar:
1- Desprendimiento leve: sin compromiso materno ni fetal y el embarazo es de pretrmino,
conducta expectante. Uteroinhibicin y maduracin pulmonar fetal.
2- Desprendimientos moderados a severos: si hay compromiso materno o fetal u bito fetal,
terminar inmediatamente la gestacin.
3- Independientemente de la va del parto, es preceptiva la prctica de la amniotoma, para
reducir la presin intraamnitica y disminuir el paso de tromboplastina hstica a la circulacin y
la extravasacin de sangre al miometrio.
Las principales complicaciones son:
1- Hipotensin, shock hipovolmico.
2- Coagulacin intravascular diseminada.
3- Necrosis cortical y necrosis tubular aguda renal.
Obstetricia. Scwarcz, Sala, Duverges. 7 edic. Edit. El Ateneo. (Biblioteca Fac. Med.
UNNE).

21.- Mujer de 27 aos a quien despus de estudios de laboratorio y exploracin fsica se le


diagnostica hirsutismo asociado a ovario poliqustico, el siguiente frmaco que usted elige para
su tratamiento por ser el ms adecuado es:
a) Clomifeno
b) Estrgenos
c) Corticoide
d) Acetato de ciproterona

El acetato de ciproterona parece ser ms efectivo que otros frmacos para el hirsutismo en
mujeres causado por la produccin ovrica excesiva de andrgenos
Una de las causas de hirsutismo (crecimiento piloso excesivo) en mujeres es la
hiperproduccin de andrgenos a partir del ovario. Varios frmacos pueden utilizarse para
contrarrestar los efectos del andrgeno. El acetato de ciproterona es un frmaco
antiandrognico. Los efectos adversos informados con su uso fueron aumento de peso,
depresin, fatiga, sntomas mamarios y disfuncin sexual. La revisin de los ensayos encontr
que el acetato de ciproterona parece ejercer un efecto en el hirsutismo similar a otros frmacos
utilizados para el tratamiento del hirsutismo por exceso de andrgenos. No existen pruebas
suficientes para comparar los efectos adversos de las opciones de tratamiento.

Van der Spuy ZM, le Roux PA. Acetato de ciproterona para el hirsutismo (Revisin Cochrane
traducida). En: La Biblioteca Cochrane Plus, nmero 4, 2007. Oxford, Update Software Ltd.
Disponible en: http://www.update-software.com. (Traducida de The Cochrane Library, 2007
Issue 4. Chichester, UK: John Wiley & Sons, Ltd.).

22.- Mujer de 36 aos nulpara tras 3 aos de relaciones sexuales sin contracepcin, desde
hace 1 ao inici presentando dismenorrea, dispareunia y sangrado vaginal intermenstrual. El
diagnstico ms probable de sta paciente es:
a) Insuficiencia lutenica.
b) Enfermedad inflamatoria plvica.
c) Dismenorrea funcional.
d) Endometriosis.

La endometriosis consiste en la aparicin y crecimiento de tejido endometrial fuera del tero,


sobre todo en la cavidad plvica como en los ovarios, detrs del tero, en los ligamentos
uterinos, en la vejiga urinaria o en el intestino. Es menos frecuente que la endometriosis
aparezca fuera del abdomen como en los pulmones o en otras partes del cuerpo.
La endometriosis es una enfermedad relativamente frecuente, que puede afectar a cualquier
mujer en edad frtil, desde la menarquia hasta la menopausia, aunque algunas veces, la
endometriosis puede durar hasta despus de la menopausia. La endometriosis altera la calidad
de vida de las mujeres que la padecen, afectando a sus relaciones de pareja, familiares,
laborales y de reproduccin.

Sntomas
Los sntomas clsicos son la dismenorrea, dolor plvico, dispareunia, sangrados
intermestruales y en muchos casos, esterilidad.
El dolor no tiene que ver con el tamao y la severidad de la lesin; generalmente cuanto
menor es la lesin mayor dolor produce. El dolor se agrava con las menstruaciones y en los
casos en que la lesin ocupa el fondo de saco de Douglas, puede dar dispareunia. Existe un
aumento de la PGF2 alfa y PGE2 y un aumento de las contracciones uterinas que podra
deberse a un depsito de endometrio en la cavidad peritoneal.
La esterilidad debido a la endometriosis podra deberse a distintas causas de acuerdo a la
severidad de la patologa. En los casos de endometriosis severa puede haber un factor
tuboperitoneal con adherencias y alteracin en la anatoma de la pelvis que interfiera con el
transporte del esperma y el vulo. En los casos de endometriosis leve hay varios mecanismos
propuestos que justifican su relacin con la infertilidad: foliculognesis alterada, fase ltea
inadecuada, fagocitosis espermtica, mala calidad ovocitaria, embriotoxicidad y alteracin a
nivel de la implantacin.. La produccin de prostaglandinas por el endometrio ectpico puede
afectar la motilidad tubaria, la foliculognesis y la funcin del cuerpo lteo. Puede haber un
aumento de la activacin de los macrfagos peritoneales en la endometriosis que cause la
fagocitosis de los espermas o la secrecin de citoquinas que pueden ser txicas para el
embrin. Segn algunos investigadores habra un 60% de las mujeres con endometriosis que

presentan un sndrome de Folculo Luteinizado no roto (LUF) en el cual el folculo no se rompe


en la ovulacin y el vulo queda atrapado.
Referencias bibliogrficas

1. Ruiz V. Endometriosis y fertilidad. Ed. Acosta y Warman, pp. 99


2. Lpes,VH. Palomo E. Incidencias de endometriosis en una poblacin infrtil. XXI Congreso
nacional de Ginecologa y Obtetricia. Guatemala, 1993.
3. El-Eoley, et al. Danazol but not ginadotropin releasing hormone agonists suppresses
autoantibodies in endomeriosis. Fertil Steril 1990; 54:725
4. Acosta AA. Buttram VC Jr. Besch PK, Malinak LR, Van Der Heyden J. A.proposed
classfication of pelvic endometriosis. Obstet Gynecol 1973;42:19.
5. Buttran VC Jr. Evolution of the revised American Fertility classification of endometriosis. Fert.
Steril 1985; 43: 347
6. Lpez VH. Tratamiento mdico-quirrgico de la endometriosis. Simposio El rostro cambiante
de la endometriosis panam 3. 12. 1993.
7. Steinleitner A. Heterolous transplation of activated murine peritonel macrophages inhibitis
gamete interaction in vivo; A paradigm fo endometriosis associted subfertility. Fertil Steril 1990;
54:725.
8. Damewood M. Effect of serum from patients with minimal to mild endometriosis on mouse
embryo growth. Fertil Steril 1990; 54: 917
9. Proug S. Peritoneal fluid fracctions from patients with endometriosis do not promote two-cell
mouse embryo growth. Fertil Steril 1990; 54: 927.

23.-Se trata de femenino de 37 aos de edad, a quien se realiza diagnstico de placenta


percreta, el tratamiento e eleccin en esta patologa es:

a)
b)
c)
d)

Histerectoma.
Ergonovina a dosis altas.
Hemostasia con puntos transfictivos.
Taponamiento uterino.

La placenta anormalmente adherida es poco comn y tiene importancia clnica por su


morbimortalidad, a consecuencia de hemorragia, perforacin, invasin y lesin de las vas
urinarias. Esta adherencia anormal est asociada con la implantacin placentaria sobre
cicatrices de cesrea previa, incisiones uterinas o legrados. La placenta percreta consiste en la
penetracin del tejido placetario a travs de toda la pared uterina, traspasando la serosa de la
misma. La identificacin de esta anormalidad antes del parto es posible mediante mtodos de
imagen (escala de grises por ultrasonido, ecografa Doppler color pulsado o resonancia
magntica nuclear). El tratamiento conservador se acompaa de elevada morbilidad en
muchos casos, por lo que el tratamiento quirrgico se convierte en el definitivo. La

literatura sugiere un aumento previsto en la incidencia de esta condicin con base en el


incremento del nmero de cesreas, por lo que la histerectoma postcesrea ser una decisin
que enfrentarn los especialistas con mayor frecuencia.
Perucca E, Domnguez C, Yahng Ch, Garca R. Placenta previa percreta con invasin vesical.
Rev Chil Obstet Ginecol 1997; 62(3): 206-10.

2. Abbas F, Talati J, Wasti S et al. Placenta percreta with bladder invasion as a cause of life
threatening hemorrhage. J Urol 2000; 164: 1270-4.

3. Perucca E, Cazenave H, Barra A, Ochoa N, Villagrn G, Espinoza R, Estay R, Bustamante


R, Siebert A. Placenta previa percreta con invasin vesical. Rev Chil Obstet Ginecol 2002;
67(5): 364-7.
4. Price F, Resnik E, Heller K, Christopherson W. Placenta previa percreta involving de urinary
bladder. A report of two cases and review of the literature. Obstet Gynecol 1991; 78(3): 508-11.

24.- Se trata de paciente femenino de 37 aos de edad, G-3, C-2. Es ingresada a hospital
presentando cefalea, acfenos, fosfenos y epigastralgia en barra con embarazo de 34
semanas. E.F T/A 160/110, FC 84 x, FR 18 x, no presenta fiebre, somnolienta, sin agregados
cardioventilatorios, hepatalgia. F.U. de 25 cm. Producto nico vivo. FCF 110 lpm, genitales sin
prdidas ni modificaciones cervicales. Laboratorio: hb 9.8 g/dl, plaquetas de 54 mil, TP 11 seg
TPT 27, TGO 160 ng/dl TGP 160 ng/dl, hiperbilirrubinemia indirecta, albuminuria 300 mg/dl,
cido rico de 8.1 mg/dl, creatinina de 1.5 mg/dl.
El diagnstico ms probable es:
a)
b)
c)
d)

Sindrome de hellp.
Sx anticuerpos antifisfolpidos
Prpura trombocitopnica trombtica
Hgado graso

DEFINICIN:
Es una complicacin de la preeclampsia en la cual adems de la Hipertensin Arterial y
proteinuria hay presencia de anemia hemoltica, enzimas hepticas elevadas y recuento bajo
de plaquetas
MANIFESTACIONES CLINICAS:
Malestar general, fatiga y molestias inespecficas 90%
Cefalea 70%
Epigastralgia 64%
Vmito 22%
Fosfenos 15%
Visin Borrosa 11%
Acfenos 3%
Ictericia
Anemia no explicada
Oliguria

BIBLIOGRAFIA:
Sibai baha, El sndrome HELLP. Universidad de Valencia , revista quincenal de Obstetricia
clnica y ginecologa, Octubre 2003.
V. Cararach, Sndrome de HELLP y Repercusiones maternas. X curso intensivo de formacin
continuada materno fetal. Enero de 2003.
Toirac, Abelardo. Sndrome de Weistein HELLP Hospital Ginecoobstetrico Tamara Bunke.
Junio 2002
De la Fuente, David. Sndrome HELLP. Medicina Universitria 2003; 5 (19): 101 -9
Andrea G. Witlin, DO, Baha M. Sibai, MD. Diagnosis and Management of women with
Hemolysis Elevate Liver Enzymes, and Pletelet Count (HELLP) syndrome. Hospital Physician.
Febrero 1999.
CIFUENTES B, Rodrigo. Ginecologa y obstetricia.

25.- Se trata de femenino de 24 aos, G-1, que cursa con embarazo de 37 semanas de
gestacin, presenta prdida del estado de alerta posterior a crisis convulsivas tnico-clnicas,
signos vitales con T-A 170.120mmhg Fc 95x, reflejos osteotendinosos aumentados, se aprecia
una Fc fetal de 132x y edema importante de miembros inferiores, no se aprecian datos de
trabajo de parto ni modificaciones cervicales, El diagnstico ms probable es:

a)
b)
c)
d)

Pre eclampsia severa


Crisis epileptica de gran mal
Hipertensin inducida por el embarazo
Eclampsia

CUADRO 1. DIAGNSTICO*
Preeclampsia Leve: Se presenta despus de la semana 20 de gestacin, durante el parto, o
en las
primeras 6 semanas despus de ste
Presin sistlica a 140 mm Hg o presin diastlica 90 mm Hg
Proteinuria a 300 mg / orina de 24 hrs o su equivalente en tira reactiva
Preeclampsia Severa: Se presenta despus de la semana 20 de gestacin, durante el parto, o
en las primeras 6 semanas despus de ste
Presin sistlica a 160 mm Hg o presin diastlica 110 mm Hg
Proteinuria a 2 gr en orina de 24 horas o su equivalente en tira reactiva
Creatinina srica > a 1.2 mg/dl
Trombocitopenia 150 000 cel/mm3
Incremento de la deshidrogenasa lctica a 600 UI
Elevacin al doble de los valores de TGO/AST o TGP/ALT
Cefalea, alteraciones visuales o auditivas
Epigastralgia
Oliguria a 500 ml en 24 horas 7
Edema agudo de pulmn
Dolor en hipocondrio derecho
Restriccin en el crecimiento intrauterino
Oligohidramnios
Eclampsia Preeclampsia mas convulsiones sin otra causa. Se presenta despus de la semana
20 de gestacin, durante el parto, o en las primeras 6 semanas despus de ste.
Sndrome de HELLP Criterios para establecer el diagnstico del sndrome de HELLP:
Plaquetas < 100 000/mm3 TGO/AST 70U/L DHL 600U/LBilirrubina total > 1.2 mg/dl
Se presenta despus de la semana 20 de gestacin, durante el parto, o en las primeras 6
semanas despus de ste.
Hipertensin Crnica: Se diagnostica cuando existe hipertensin arterial a 140/90 mm Hg
antes de la semana 20 de gestacin o si persiste despus de doce semanas posteriores al
parto.
Las pacientes con hipertensin crnica deben ser evaluadas antes del embarazo para
determinar la severidad de la hipertensin y facilitar la planeacin de un embarazo mediante el
cambio de medicamentos y de hbitos higinicos y dietticos para evitar complicaciones.
Hipertensin Gestacional: Presencia de hipertensin arterial a 140/90 mm Hg despus de
la semana 20 de gestacin y se mantiene hasta las doce semanas despus del parto
Ausencia de proteinuria
Presencia o no de cefalea, acfenos y fosfenos
Despus de 12 semanas de la interrupcin del embarazo se revalorar la presencia de
hipertensin, si contina, se reclasifica como hipertensin crnica: es un diagnstico
retrospectivo. 8 Si no hay, se clasifica como hipertensin transitoria.
1. Aagaard-Tillery KM, Belfort MA. Eclampsia: morbidity, mortality, and management. Clin
Obstet Gynecol 48:12-23, 2005.

2. Atallah AN, Hofmeyr GJ, Duley L. Calcium supplementation during pregnancy for preventing
hypertensive disorders and related problems. Cochrane Database Syst Rev 1:CD001059, 2001.
3. Barton JR, Sibai BM. Diagnosis and management of hemolysis, elevated liver enzymes, and
low platelets syndrome. Clin Perinatol 31:807-33, 2004.
4. Baxter JK, Weinstein L. HELLP syndrome: the state of the art. Obstet Gynecol Surv 59:83845, 2004.
5. Cetin A. Eclampsia. In Mohler III ER, Townsend RR. Advanced therapy in hypertension and
vascular disease. Ontario: B.C. Decker Inc. pp. 407-15, 2006.
6. Cetin A. Hemolysis, elevated liver enzymes, and low platelets (HELLP). In Mohler III ER,
Townsend RR. Advanced therapy in hypertension and vascular disease. Ontario: B.C. Decker
Inc. pp. 416-20, 2006.
7. Chappell LC, Seed PT, Briley AL, Kelly FJ, Lee R, Hunt BJ, Parmar K, Bewley SJ, Shennan
AH, Steer PJ, Poston L. Effect of antioxidants on the occurrence of pre-eclampsia in women at
increased risk: a randomised trial. Lancet 354:810-16, 1999.

26.- Femenino que cursa con 12 semanas de gestacin, refiere tres das con sangrando por
genitales, con nuseas continuas y constantes, tero mayor aumentado de tamao y valores
de beta HCG muy elevados, debemos sospechar de:
a) Amenaza de aborto.
b) Aborto diferido.
c) Mola hidatdica.
d) Amenaza de aborto en un tero con miomas.

La enfermedad trofoblstica gestacional agrupa a diferentes entidades interrelacionadas: mola


completa, generalmente diploide con origen cromosmico paterno, mola parcial generalmente
triploide, tumor trofoblstico del lecho placentario y coriocarcinoma, con tendencias variables a
la invasin local y a las metstasis, cuyo denominador comn es la hipersecrecin de hCG. El
coriocarcinoma es diploide y proviene de ambos progenitores, excluyendo probablemente su
origen directo en la mola completa. El tumor trofoblstico del lecho placentario est constituido
por trofoblasto mononuclear intermedio no conteniendo vellosidades corinicas e
inmunohistoquimicamente caracterizado por expresar muchas de sus clulas hPL y unas pocas
hCG.

Cuadro clnico

Tras un periodo de amenorrea, y a partir del segundo mes, hay un aumento de los sntomas
subjetivos del embarazo, sobre todo nuseas y vmitos o hipermesis en un 30%. Hay
metrorragias irregulares en el 96% de los casos, en principio de escasa cantidad, pero que se
va incrementando. Dicha hemorragia no es continua sino que se repite cada dos o tres das, de
color roja o negruzca, que se produce por la ruptura de vasos maternos al separarse las
vesculas de la decidua.
Ocasionalmente, hay expulsin de restos molares, que lo refiere la paciente como expulsin de
vesculas en el 11 % de los casos y que es patognomnico pero aparece tardamente. Tambin
puede presentar mal estado general, dolor difuso en hipogastrio y anemia. Esta ltima en
relacin con las metrorragias.
Pueden haber signos y sntomas de hipertiroidismo, como taquicardia, sudoracin, y temblores,
en el 7%, y es debido a que la fraccin -hCG es similar a la hormona TSH. Existen signos de
preeclampsia o hipertensin gestacional del primer trimestre hasta en el 50% de los casos, y
signos de insuficiencia respiratoria aguda en el 2% por embolismo pulmonar de las clulas
trofoblsticas, o por la asociacin entre hipertiroidismo e hipertensin arterial. Como
complicaciones pueden aparecer coagulopatas y metstasis.
A la exploracin el tero esta aumentado de tamao, en el 50% de los casos, por encima de lo
que correspondera a la edad gestacional,3 de consistencia blanda, sin signos de actividad fetal
a partir de la semana 12 y siempre que se trate de una mola total. Tambin puede que la
paciente presente un tamao uterino menor al esperado para la edad gestacional.3 El cuello
est cerrado, con metrorragia en cantidad variable, y raramente se observa la expulsin de
vesculas. Pueden haber quistes teca-lutenicos bilaterales en ovario en cerca del 20% de los
casos,3 debido al estmulo de la -hCG.

La enfermedad trofoblstica maligna va a cursar con metrorragias por lo general intensas, hay
una elevacin de la -hCG y en la ecografa se objetiva la cavidad uterina con signos de
ocupacin atpica.

Diagnstico
Por la clnica, y pruebas complementarias como la determinacin de la -hCG y la ecografa.
La determinacin de la -hCG se basa en que el trofoblasto produce la hormona gonadotropina
corinica, presentando cifras elevadas, y su cuantificacin va a servir para diagnstico, valorar
el pronstico, y el seguimiento postratamiento. La ecografa revela un tero aumentado de
tamao que no corresponde con la amenorrea, con ecos en su interior, puntiformes que
corresponderan a las vesculas y que asemejan copos de nieve o panal de abeja. No se
aprecia saco gestacional ni estructuras fetales y, en ambos ovarios se aprecian quistes tecalutenicos como formaciones ovricas redondas, econegativas, con mltiples tabiques en su
interior

BIBLIOGRAFA
1. Mazur MT, Kurman RJ. Gestational trophoblastic disease and related lesions. En:
Kurman RJ editor. Blaunsteins pathology of the female genital tract. 4th ed. New York:
Springer-Verlag. 1994, p. 1049-93.
2. Kurman RJ, Young RH, Norris HJ, Main CS, Lawrence WD, Scully RE.
Immunocytochemical localization of placental lactogen and chorionic gonadotrophin in
the normal placenta and trophoblastic tumors, with emphasis on intermediate
trophoblast and the placental site trophoblastic tumor. Int J Gynecol Pathol 1984; 3:
101-21.
3. Berkowitz RS, Golstein DP. The management of molar pregnancy and gestational
trophoblastic tumours. En Knapp RC, Berkowitz RS, editores. Gynecologic Oncology,
2nd ed. New York: Mc Graw-Hill 1992, p. 328-38.
4. De Agustn P, Ruiz A, Lpez F, Contreras F. Patologa de la enfermedad trofoblstica.
Simposio Enfermedad Trofoblstica 1972; 79-98.
5. Salem S. Ultrasound diagnosis of trophoblastic disease. En: Sanders RC, James AE(Jr)
editores. Ultrasonography in Obstetrics and Gynaecology. New York: Appleton-Century
Crofts: 1977; p. 255-66.
6. Silverberg SG, Kurman RJ. Tumors of the uterine corpus and gestational trophoblastic
disease. En: Rosai J, Sobin LJ, editores. Atlas of tumor pathology: tumors of the uterine
corpus and gestational trophoblastic disease, fasc. 3, ser. 3. Washington DC: Armed
Forces Institute of Pathology; 1992, p. 219-85.

Szulman AE, Surti U The syndromes of hydatiform mole II. Morphologic evolution of the
complete and partial mole. Am J Obstet Gynecol. 1978; 132: 20-7.

27.- Mujer que acude al servicio de ginecologa por referir ciclos opso-menorreicos desde el
inicio de su menarquia, ha incrementado 15 Kg. de lo que pesaba habitualmente, se aprecia
una gran cantidad de acn, pero adems refiere depilarse el rea del bigote cada semana, y
cree que esto le sensibiliza la piel para que aumente el acn.
En el caso de ovario poliqustico el dato clnico que con ms frecuencia les acompaa es:
a)
b)
c)
d)

anovulacin y esterilidad
Hirsutismo
Amenorrea
Obesidad

El sndrome de ovarios poliqusticos (SOPQ) afecta aproximadamente a un 4% de mujeres en


edad reproductiva y se caracteriza por anovulacin crnica e hiperandrogenismo. Es la causa
ms comn de infertilidad en mujeres.
Se caracteriza clnicamente por acn, alopecia, hirsutismo, irregularidades menstruales e
infertilidad.
Los hallazgos de laboratorio ms frecuentes son: aumento de la hormona luteinizante (LH),
aumento de la relacin LH/FSH (hormona folculoestimulante), aumento de andrgenos (tanto
ovricos como adrenales) y de estrgenos circulantes. Otros hallazgos de laboratorio
habituales son una prueba tolerancia oral a la glucosa anormal y alteraciones en el perfil
lipdico.
Todo esto junto con las imgenes ecocardiogrficas caractersticas define al sndrome.
La teraputica permite dos grandes enfoques que pueden superponerse: la correccin de las
manifestaciones de hiperandrogenismo y el tratamiento de las alteraciones del eje reproductivo
(anovulacin, esterilidad). Los antiandrgenos estn fundamentalmente indicados para tratar
los sntomas virilizantes.
Las alternativas para inducir la ovulacin son numerosas: al citrato de clomifeno y a la antigua
reseccin en cua se agregan las gonadotrofinas humanas, pulsos de GnRH (hormona
liberadora de gonadotrofinas), medidas o frmacos para modificar los niveles de insulina, y
finalmente tcnicas quirrgicas endoscpicas para reducir la masa ovrica.
BIBLIOGRAFIA
1. Guzick D.Polycystic ovary syndrome: Symptomatology, pathophysiology, and
epidemiology. Am J Ostetric Gynecol 1998; 179 (6): 89-93.
2. Stephen Franks. Polycystic ovary syndrome. N Engl J Med 1995; 333(13): 853-861.
3. Gori J.R., Larusso A. Ginecologa de Gori. 2 Edicin. Buenos Aires, Argentina.
Editorial El Ateneo. 2001.
4. Adams J., Polson D. W., Franks S. Prevalence of polycystic ovaries in women with
anovulation and idiopathic hirsutism. Br Med J 1986; 293: 355-9.
5. Copeland L. J . Ginecologa. Buenos Aires, Argentina. Editorial Panamericana. 1
Edicin. 1994.
6. Ehrmann D.A., Rosenfield R.L., Barnes R.B., Brigell D.F., Sheikh Z. Detection of
functional ovarian hyperandrogenism in women with androgen excess. N Engl J Med
1992; 327:157-162.
7. Kahasar-Miller M., Conway Myers B., Boots L., Azziz R. Steroidogenic acute regulatory
protein (StAR) in the ovaries of healthy women and those with polycystic ovary
syndrome. Am J Obstet Gynecol 2001; 185(6): 1381-7.
8. Prez Snchez A. Ginecologa. Santiago de Chile. Publicaciones Tcnicas
Mediterrneo. 3 Edicin. 1995.
9. Velzquez E., Mendoza S., Hamer T., Sosa F., Glucck C. Metformin therapy in women
with polycistic ovary syndrome reduces hiperinsulinemia, insulin resistence,

hyperandrogenemia, and systolic blood pressure, while facilitating menstrual regularity


and pregnancy. Metabolism 1994 ; 43: 647-655.
28.- Femenino de 29 aos. Acude al servicio de urgencias por presentar salida de lquido
vaginal. Antecedentes: G2, P1, cursa embarazo de 36 semanas de gestacin exploracin
fsica: cervix con 10% de borramiento, 1 cm de dilatacin y Tarnier positivo.
La complicacin ms frecuente en esta paciente es:

a)
b)
c)
d)

Corioamnioitis.
Parto pre trmino.
Sepsis neonatal.
Endometritis.

Corioamnioitis: El diagnstico de la infeccin intraamnitica (IIA) es bsicamente clnico. La


corioamnionitis se debe descartar en toda gestante que presente fiebre sin foco aparente,
sobre todo si se sospecha o se ha confirmado una rotura de membranas. Los criterios ms
empleados para el diagnstico son: fiebre materna y, al menos, 2 de los siguientes signos:
taquicardia materna, taquicardia fetal, irritabilidad uterina, leucocitosis materna o lquido
amnitico purulento o maloliente.
Progresos de obstetricia y ginecologa: revista oficial de la Sociedad espaola de ginecologa y
obstetricia, ISSN 0304-5013, Vol. 48, N. 6, 2005 , pags. 316-317

29.- Mujer de 23 aos diagnosticada de E. Ectpico a nivel ampular, con saco gestacional
menor de 3 cm., sin actividad cardaca embrionaria, asintomtica y con niveles de B-HCG
menores para su edad gestacional. El tratamiento ms indicado es:
a) Salpingocentesis
b) Reseccin segmentaria
c) Tratamiento mdico con Metrotexate.
d) Histerectoma total con doble anexectoma.

TRATAMIENTO

METOTREXATE
cido 4 amino 10 metil folnico, antagonista del cido flico
Glucosa hiperosmolar
Prostaglandina F2a
Actinomicina D
Mifespristona

TRATAMIENTO CON METOTREXATO


Mayor xito:

Embarazo menor a 6 SDG


Masa tubaria menor de 3.5cm
Feto sin latido cardaco

PROTOCOLO CON DOSIS NICA DE METOTREXATE


Da 0
Da 1
Da 4
Da 7

hGC, Biometra hemtica, transaminasas, creatinina, grupo


hGC
hGC
hGC

sanguneo

Disminucin de hCG <15%. Segunda dosis


Si la hCG declina seguirla semanalmente
Si la hCG est en meseta o en ascenso, segunda dosis
El raspado endometrial se realiza slo en pacientes con hCG <2000 mIU/mL al
momento de comenzar el tratamiento

Graczykowski JW, Mishell DR. Methotrexate prophylaxis for persistent ectopic pregnancy after
conservative treatment by salpingostomy. Obstet Gynecol.

30.- Femenino de 19 aos acude a consulta por padecer un cuadro de dolor abdominal de
inicio sbito, refiere alteraciones en su ciclo menstrual. La exploracin fsica revela una
tumoracin dolorosa en el anexo izquierdo. La prueba de embarazo es negativa. La radiografa
muestra una masa opaca en la fosa ilaca izquierda con reas de calcificacin. El diagnstico
clnico ms probable es:

a) Cistadenoma mucinoso
b) Teratoma qustico
c) Quiste folicular
d) Cistadenoma seroso
Los tumores de clulas germinales constituyen casi el 20% de los tumores de ovario y de ellos
un 95% son benignos, siendo el tipo ms frecuente el teratoma maduro benigno o tambin
denominado quiste dermoide (1).
Aproximadamente el 80% se presentan en mujeres en edad frtil. Se originan a partir de
clulas embrionarias pluripotenciales presentes habitualmente en ovario, testculo, mediastino,
retroperitoneo y regin sacrocoxgea, esto explica que la coloracin de las faneras del quiste
coincida con el fenotipo del paciente (2).
El teratoma qustico benigno con relativa frecuencia es un tumor bilateral (del 7 al 25% segn
los autores) (1) y se caracteriza por una cpsula gruesa, bien formada, revestida por epitelio
plano estratificado. Bajo este se pueden encontrar una variedad de apndices cutneos que
incluyen glndulas sudorparas, apocrinas y sebceas. La cavidad se llena de los detritus de
ste y sus anexos, que es de color amarillo plido, grasoso, espeso y suele contener pelo.
Otros tejidos que se pueden encontrar son dientes, cartlago, plexos coroideos, falanges, tejido
nervioso y en ocasiones tejido tiroideo (struma ovarii) con potencial tirotxico o de
degeneracin maligna tiroidea.
La mayor parte de los quistes dermoides son asintomticos y la forma ms frecuente de
presentacin son el dolor abdominal (48%) y hemorragia uterina anormal o concomitante (15%)
o aumento del volumen abdominal (15%). La rotura de un quiste dermoide es rara, entre el 11,2% y constituye una urgencia quirrgica (3).

El tratamiento es quirrgico, siendo posible la mayor parte de las veces una reseccin del
mismo, respetando el resto del ovario.
La ecografa constituye el modelo diagnstico de eleccin y la combinacin de ecografa con
radiografa simple de abdomen proporciona un diagnstico ms exacto en la mayora de los
casos, siendo la resonancia magntica o la TAC el que aporte el diagnstico diferencial.
Las Rx de quiste dermoide se caracteriza por una cpsula bastante radioopaca y el lquido
oleoso que contiene es radiolcido, esta conjugacin presenta muchas veces
una
caracterstica de aspcto radiolgico de calcificacin en la pared del mismo.
Clico nefrtico, teratoma ovrico y radiografa simple de aparato urinario Romero Prez P,
Martnez Hernndez MC.
Servicio de Urologa. Policlnico San Carlos Denia (Alicante).Actas Urol Esp. 2007:31(8):936937ACTAS UROLGICAS ESPAOLAS SEPTIEMBRE 2007

31.- Se trata de femenino de 33 aos que acude al servicio cursa en ste momento con
diagnstico de preclampsia , el frmaco de eleccin que se administra en esta patologa es:

a)
b)
c)
d)

Nifedipina.
Inhibidores de la enzima convertidora de angiotensina.
Clonidinas.
Alfametildopa.

Prevenir complicaciones a corto plazo de las mujeres con PA elevada que comprometa
el bienestar fetal
Cuando la PAS es mayor o igual a 150 mmHg y la PAD mayor o igual a 100 mmHg.
El propsito es alcanzar cifras de TA alrededor de 140/90.
La medicacin antihipertensiva se reserva para los casos en que la PAD 100 mmHg.
Se recomienda continuar el tratamiento antihipertensivo previo al embarazo,
exceptuando el uso de IECA.
La alfametildopa y la hidralazina va oral son los frmacos de eleccin dado su uso
extensivo con seguridad y eficacia y sin efectos colaterales para el feto (excepto
hidralazina en lupus).

ALFA METILDOPA
500-2000 MG/DA

HIDRALAZINA
50-200 MG/DA
LABETALOL
100-400 MG/DIA
ATENOLOL
50-200 MG/DA
NIFEDIPINA
10-30 MG/DA

1. Aagard K, Belfort M. Eclampsia: Morbility, mortality, and management. Clin


Obstet Gynecolol. 2005; 48: 12-23.
2. Oyarzn E. Sndrome hipertensivo del embarazo en Oyarzn E. Ed. Embarazo de alto
riesgo. Ediciones Universidad Catlica de Chile. Santiago. 1997: 157175.
3. Roberts J, Redman C. Pre-eclamsia: More than pregnancy induced hypertens

32.- Se trata de femenino de 56 aos de edad refiere aumento de volumen a nivel abdominal,
con predominio de hemiabdomen inferior con la siguiente sintomatologa: plenitud,
estreimiento, se acompaa de USG plvico que demuestra imagen qustica en ovario
derecho de 15 por 15 cms. El diagnstico ms probable es:

a)
b)
c)
d)

Teratoma qustico.
Disgerminoma.
Endometrioma.
Cistadenoma seroso.

Los Tumores de Ovario son una patologa frecuente dentro del contexto de la patologa
femenina. Por esta causa consultan un grupo elevado de mujeres, tanto las consultas de
ginecologa como las de Ciruga propiamente dicha. Las edades oscilan desde las tempranas
hasta las ya avanzadas, siendo el riesgo de degeneracin maligna muy variable y relacionado
con le edad. La experiencia de la clnica revela la alta incidencia de tumores de ovario en la
etapa del climaterio, comprendida entre los 35 y 65 aos de edad 1.
El cistoadenoma seroso de ovario (CSO) es un tipo de tumor derivado del epitelio superficial
(celmico), formado por reas qusticas. El cistoadenoma seroso de ovario es el tumor ms
frecuente de aquellos que provienen del epitelio celmico superficial. Hay tumores pequeos
macroscpicamente y tumores masivos que ocupan toda la pelvis e incluso la cavidad
abdominal. Estas frecuentes neoplasias qusticas uniloculares estn tapizadas por clulas
epiteliales altas, cilndricas y ciliadas, llenas de un lquido seroso claro y de superficie lisa con
abundantes vasos. Las variedades benigna, limtrofe y maligna representan, en conjunto, 30%
aproximadamente de todos los tumores del ovario. El riesgo de presentar tumores epiteliales se
incrementa con el paso de la edad, ya que pese a que la declinacin de la funcin ovrica
marca el envejecimiento gonadal progresivo, el ovario humano nunca pierde su capacidad para
generar tumores. Por lo general, cuando es detectado, su tamao es grande, en donde la
imagenologa puede ayudarnos a considerar su diagnstico

1.
Captulo 22 Tumores Benignos de Ovario. En: Novak ER, Jones G., Jokes HW. Tratado
de Ginecologa. 9 ed. Ciudad de la Habana. Editorial Cientfico Tcnica; 1977.p.432 66.
2.
MedlinePlus Enciclopedia Mdica en Espaol: Quistes Ovricos. Disponible en:
http://vsearch.nlm.nih.gov/vivisimo/cgibin/querymeta?v%3Aproject=medlineplusspanish&spell=s
pell&query=Quistes+Ov%C3%A1ricos Acceso: Actualizado 20/6/06.
Captulo XL Tumores Ovricos En: Llusi Botella J, Nez Clavero JA. Tratado de Ginecologa.
Ciudad de la Habana. Editorial Cientfico Tcnica. 1983; T 3.1; p. 751 803.

33.- Paciente femenino de 27 aos de edad con deseo de un embarazo, antecedentes de


G3 A2 - P1 se le realiza una histerosalpingografa, se constata que existe un sndrome de
Asherman. Ello significa que se trata de:

a)
b)
c)
d)

tero bicorne
Endometriosis en la trompa
Sinequias uterinas
Insuficiencia istmico cervical

El sndrome de Asherman es una enfermedad ginecolgica rara que se caracteriza por la


presencia de sinequias (adherencias) intrauterinas que pueden ocasionar amenorrea (ausencia
de perodos menstruales regulares) e infertilidad.

En 1894 Heinrich Fritsch describe por primera vez la presencia de sinequias intrauterinas de
tipo postraumtico, en una paciente que desarroll una amenorrea secundaria a un curetaje.
Posteriormente en 1927 Bass inform de veinte casos de atresia (oclusin de una abertura
natural) cervical tras abortos inducidos, pero no fue hasta 1948, cuando Joseph G. Asherman
recopil la informacin hasta entonces existente y acu el nombre con el que se conoce
actualmente a la enfermedad.

Asherman describi originalmente dos tipos diferentes de amenorrea secundaria, en funcin de


su etiologa (estudio de las causas de las enfermedades): la amenorrea traumtica atrtica,
debida a estenosis del orificio cervical interno y la amenorrea debida a adherencias
intrauterinas. Posteriormente ambas entidades se agruparon en una nica entidad bajo el
nombre de sndrome de Asherman.

Suele presentarse en mayor proporcin tras dilataciones y curetajes uterinos de repeticin y


sobre todo si se realizan durante el embarazo o si existe infeccin uterina en el momento en el
que se realizan estas intervenciones.

Las adherencias intrauterinas pueden producirse debido a cualquier factor que lleve a una
destruccin de las paredes del miometrio (capa muscular de la pared del tero). Sin embargo,
hay que distinguir entre factores predisponentes, siendo el principal de ellos el embarazo y
factores causales, entre los que se encuentran: traumatismos uterinos, intervenciones
quirrgicas que afecten al tero, agentes fsicos o qumicos e infecciones uterinas por
tuberculosis o esquistosomiasis. En cualquier caso, el factor ms importante es el trauma
uterino en el momento del parto o el puerperio.

El cuadro clnico es muy variable y las manifestaciones clnicas varan con el grado de oclusin
de la cavidad uterina y la severidad de las adherencias, pudiendo presentarse: esterilidad
cuando la oclusin de la cavidad uterina incluye porciones proximales (ms cerca de un centro,
tronco o lnea media) de las trompas de Falopio o cuando las adherencias impiden la nidacin
del huevo; las pacientes presentan con frecuencia amenorrea, oligomenorrea (disminucin de
la frecuencia de las menstruaciones), dismenorrea (menstruacin dolorosa) y abortos
repetidos.
1. Hysteroscopic treatment of severe Asherman's syndrome and subsequent fertility. CapellaAllouc S; Hum Reprod, 1999 May.

34.- Se trata de femenino de 31 aos de edad, la cual inicia con hiperemesis gravdica de
difcil control, as como sangrado trasvaginal. Se realiza el diagnstico de mola hidatiforme. La
primera opcin de tratamiento indicado en esta patologa es:

a)
b)
c)
d)

Histerectoma total abdominal.


Legrado por aspiracin.
Metotrexate y seguimientos radiogrficos.
Legrado uterino instrumental

La enfermedad trofoblstica gestacional agrupa a diferentes entidades interrelacionadas: mola


completa, generalmente diploide con origen cromosmico paterno, mola parcial generalmente
triploide, tumor trofoblstico del lecho placentario y coriocarcinoma, con tendencias variables a
la invasin local y a las metstasis, cuyo denominador comn es la hipersecrecin de hCG. El
coriocarcinoma es diploide y proviene de ambos progenitores, excluyendo probablemente su
origen directo en la mola completa. El tumor trofoblstico del lecho placentario est constituido
por trofoblasto mononuclear intermedio no conteniendo vellosidades corinicas e
inmunohistoquimicamente caracterizado por expresar muchas de sus clulas hPL y unas pocas
hCG

Tratamiento
Hay que tratar las complicaciones como la hipermesis, anemia, hipertensin, y alteraciones
electrolticas, coagulopatas, alteraciones cardio-respiratorias y preeclampsia, procediendo a
evacuar la mola lo antes posible, con lo que se producirn menos malignizaciones.
La evacuacin del contenido uterino se realiza mediante dilatacin, y legrado por aspiracin.
Adems se pauta profilaxis antibitica y oxitcicos.3

La histerectoma, con la mola en su interior, est indicada en pacientes de edad superior a 40


aos o en mujeres con ms de tres hijos, ya que en ambos grupos se ha demostrado una
mayor incidencia de malignizacin.
. Tras la ciruga, se mide la concentracin de gonadotropina corinica humana para determinar
si la extirpacin ha sido completa. Si es as, el valor de esta hormona vuelve a la normalidad,
en unas 8 semanas, y se mantiene en esos valores. Si una mujer a la que se le ha extirpado
una mola queda embarazada, es difcil interpretar un valor alto de gonadotropina corinica
humana, porque podra estar causado tanto por el embarazo como por una parte de la mola
que no se ha extirpado. En consecuencia, a las mujeres a las que se les ha extirpado una mola
se les recomienda no quedar embarazadas durante un ao. Las molas hidatiformes benignas
no necesitan quimioterapia, pero las malignas s. Los frmacos que se usan para este
tratamiento son el metotrexato, la dactinomicina o una combinacin de ambos.
Silverman L,Romero Zambrano F, Saldao S. Enfermedad molar. Diagnstico, tratamiento y
seguimiento, 1987.
4- Puertas A, Lpez Fernandez J et al. Enfermedad trofoblstica. Casustica del Hospital Virgen
de las Nieves de
Granada. Cln Invest Gin Obs 1993; 20: 98-103.
5- Enfermedad trofoblstica gestacional. Propuesta Normativa Perinatolgica y Ginecolgica de
Alto Riesgo.
Ministerio de Asuntos Sociales. Tucumn. 1996-1997; 19:171-183.
6- Jones. Enfermedad Trofoblstica Gestacional: qu hemos aprendido en la ltima dcada.
Am J Gynecol Obstet
1990;162: 1286-1292.

7- Resmen del simposio Enfermedad Troblstica Gestacional del Segundo Congreso Nacional
de AGORA, 1990.
8- Gonzalez Merlo et al. Protocolos de diagnstico y tratamiento en Obstetricia y Ginecologa,
Barcelona: editorial
Salvat, 7:35-45.

35.- Se trata de femenino de 33 aos que ingresa a la sala de urgencias, inicia con
convulsiones por presentar preclampsia, decide iniciar tratamiento, el frmaco eleccin en
sta patologa es:
a)
b)
c)
d)

Diacepam.
Fenitona.
Donadores de xido ntrico.
Sulfato de magnesio.

Manejo de la Preeclampsia
1. Manejo ambulatorio: HTA sin proteinuria significativa, se recomienda el reposo en cama.
Monitoreo de TA, peso, presencia de protenas en orina. Ecografas peridicas para ver el feto
y evaluar posibles retardo de crecimiento.
2. Manejo hospitalario: para mujeres con HTA inducida por el embarazo y 2+ o ms o
proteinuria significativa y en quienes fall el manejo ambulatorio.
3. Laboratorio y evaluacin del peso: debe realizarse diariamente. Evaluacin de la dinmica
fetal. Monitoreo de sntomas como cefalea, alteraciones visuales y dolor epigstrico.
4. El parto es el tratamiento de eleccin: el cual debe realizarse cuando el feto est maduro
pero puede realizarse en forma temprana si la salud de la madre est en peligro o si hay
evidencia de distress fetal. El parto est indicado cuando la paciente cumple con los criterios de
preeclampsia severa. Betametasona 12.5 mg IM dos veces por da puede estimular la
maduracin de los pulmones fetales.
5. Terapia antihipertensiva: est indicada slo si la TA es persistentemente > 160/110 , es
importante disminuir la TA hasta una diastlica de 90 a 100 mmHg porque la presin normal
podra resultar en hipoperfusin de la placenta. Los diurticos nunca estn indicados, estas
pacientes ya son hipovolmicas. Los IECA no deben ser usados durante el embarazo. Las
medicaciones de largo plazo, incluyen alfa metildopa, atenolol y labetalol.
6. Terapia anticonvulsivante:
A- Profilaxis de las convulsiones: est indicada en todas las pacientes pre-eclmpticas
durante el trabajo de parto y el parto y por un mnimo de 24 hs luego del mismo. Algunos
mantienen la terapia con magnesio hasta que comienza la diuresis. El Sulfato de Magnesio es
la droga de eleccin. La dosis profilctica es de 4 a 6 g de sulfato de magnesio IV y contina
con 2 g c/ hora.
B- Tratamiento de las convulsiones: Sulfato de Magnesio 1 g/min IV hasta controlar las
convulsiones hasta un mximo de 4 a 6 g. El nivel teraputico es de 4 meq/l. Toxicidad del
magnesio: ausencia de reflejo patelar, debilidad muscular, parlisis respiratoria y depresin
cardaca, 10 ml al 10 % de gluconato de calcio puede ser administrada IV. La terapia con
sulfato de magnesio contina por lo menos 24 horas en el post parto, la terapia puede
detenerse si la excrecin urinaria es > 200 ml/h por cuatro horas consecutivas.
C- Prevencin: 81 mg de aspirina diarios pueden ser administrados luego del primer
trimestre en mujeres con hipertensin crnica o historia previa de preeclampsia, sin embargo la
eficacia de esta indicacin ha sido cuestionada.
Myers JE, Baker PN. Hupertensive diseases and eclampsia. Curr Opin Obstet Gynecol 2002;
14: 119-125
2. Tierney, McPhee, Papadakis. Diagnstico clnico y tratamiento 2003. 38 ed, Mxico, Manual
Moderno, 2003: 770-773
3. Wilson MI, Goodwin TM, Pan VI, Ingles SA. Molecular epidemiology of preeclampsia. Obstet
and Gynecol Survey 2003; 58(1):39-66

4. Burrow GM. Complicaciones mdicas durante el embarazo. 4 ed, Mxico, McGraw-Hill


panamericana: 1996: 1-25
5. Guyton AC, Hall JE. Embarazo y lactancia en: Tratado de fisiologa mdica, 10 ed, Mxico,
McGraw-Hill Interamericana 2001: 1135-45
6. Vaticon D. Fisiologa de la fecundacin, embarazo, parto y lactancia, en: Tresguerres JAF.
Fisiologa Humana. Mxico, Interamericana McGraw-Hill, 1992: 1086-1109
7. Pridjian G, Puschett JB. Preeclampisa. Part 1: Clinical and Pathophysiologic Considerations.
Obstet and Gynecol Survey 2002; 57 (9): 598-618
8. Pridjian G, Puschett JB. Preeclampisa. Part I1: Experimental and Genetic Considerations.
Obstet and Gynecol Survey 2002; 57 (9): 619-40
9. IMSS. Embarazo de alto riesgo. Gua diagnstica teraputica. Rev Med IMSS 1998;
36(1):45-60
36.- Se trata de femenino de 47 aos que presenta sinusorrragia de 3 meses, sin alteraciones
menstruales previas, por lo dems asintomtica, el diagnstico ms probable a la
especuloscopa es:
a) Adenomiosis
b) Malformaciones Mllerianas
c) Plipo endometrial
d) Plipo endocervical
Plipo es toda formacin ssil o pediculada que se fija a la cavidad uterina. Las
formaciones polipoideas dentro del aparato genital femenino se dividen en endometriales y
cervicales. Dentro de los plipos cevicales se diferencian los ectocervicales y los
endocervicales; estos ltimos son considerados como los verdaderos por estar constituidos
por epitelio cilndricco. No se conocen con certeza los factores asociados a su histognesis
(teora ms admitida: hiperplasia focal de la mucosa endocervical por estmulo estrognico)
(1). Los plipos cervicales se consideran la forma ms frecuente de tumor benigno de
crvix.
Constituyen de un 3 a un 10% de las consultas ginecolgicas; presentan mxima incidencia
entre la 4 y 5 dcada de la vida. Algunos estudios demuestran que hasta en el 56% de
casos de mujeres postmenopusicas se encuentran asociados plipos cervicales y
endometriales; es mucho menor esta frecuencia antes de la menopausia (2). La clnica de
presentacin ms comn es la hemorragia intermenstrual, espontnea o durante el coito (3).
Tambin pueden ser asintomticos (hallazgo casual en estudio ecogrfico) o, en ocasiones,
alcanzar grandes tamaos: llegan a aflorar por el orificio cervical externo; se denominan
"plipos gigantes" cuando superan 2 cm (4). Para el diagnstico habitualmente se utiliza la
ecografa. Los casos sintomticos se consideran de indicacin quirrgica por su riesgo de
degeneracin a adenocarcinoma cervical.
Los plipos cervicales constituyen una patologa frecuente en la consulta ginecolgica, sobre
todo en mujeres mayores de 20 aos que han tenido hijos y son poco comunes antes de la
menarquia. En la mayora de los casos, solamente se presenta un plipo, aunque en ocasiones
se pueden encuentrar 2 3. La poca ms frecuente es al final de su vida reproductiva y
despus de la menopausia. En las primeras, las lesiones suelen ser ms grandes, difciles de
tratar y con frecuencia recidivan.
La causa de los plipos cervicales an no se ha comprendido completamente, pero con
frecuencia son el resultado de una infeccin crnica, una respuesta local anormal a los niveles
de estrgeno o a una congestin local de los vasos sanguneoscervicales. Clnicamente se
manifiesta con sangrado genital fuera de la menstruacin, sobre todo durante la relacin
sexual. Son benignos, no se malignizan pero deben ser extirpados para controlar la
sinusorragia.

37.- A 23-year-old nulligravid female has not menstruated in the past 4 months. Previously, her
menstrual cycles were regular. She is otherwise well and denies recent onset of stress, change
in exercise routine, headaches, visual field alterations, or galactorrhea.. She has a body mass
index of 24, blood pressure of 120/78 mm Hg, and does not appear hirsute. No adnexal masses
can be palpated. Laboratory investigations reveal a negative beta human chorionic
gonadotropin (-hCG), normal thyroid-stimulating hormone (TSH), and prolactin levels.
What is the next best step in the management of this patient?
a)
Measurement of luteinizing hormone.
b)
CT scan of the sella turca.
c)
Prescribe oral estrogen for 21 days followed by 7 days of medroxyprgesterona and
reevaluate.
d)
Preescribe 7 days of medroxyprgesterona and reevaluate.

La Prueba de respuesta a progestgenos se basa en la observacin de que el tratamiento con


progestgeno (acetato de medroxiprogesterona 10 mg por 5 a 6 das) solo induce la
menstruacin en las mujeres con concentraciones normales de estrgenos circulantes. Una
prueba positiva (hemorragia despus de concluir el tratamiento con progestgenos) seala
cifras normales de produccin de estrgenos y una prueba negativa (sin hemorragia por
privacin), hipogonadismo franco.
Danforth, Tratado de Obstetricia y Ginecologia, 9 Edicin, Ed. Mc Graw Hill Interamericana,
Pg 668

38.- Se trata de femenino de 27 aos, Gesta 1, Para 1. Con dos citologas lesin de alto
grado, prueba de Schiller positiva y biopsia de crvix que demuestra carcinoma In Situ. La
conducta es:
a)
b)
c)
d)

Conizacin.
Histerectoma total abdominal.
Histerectoma y salpingooforectoma bilateral.
Electrocauterizacin del crvix.

La conizacin cervical es el tratamiento de eleccin en pacientes con cncer cervicouterino


microinvasor y ms si existedeseo de fertilidad. Asimismo, la histerectoma extrafasciales un
mtodo adecuado en lesiones de 0.5 a 3 mm de invasin.Adems se propone que, para
pacientes con lesiones de 3.1 a 5 mm de invasin, a partir de la membrana inicial sinfactores
de mal pronstico como invasin vascular y linftica, sean tratadas con histerectoma
extrafascial, ya que en aquellas a las que se realiz linfadenectoma plvica, con este tipo de
lesin, no se encontr metstasis a ganglios linfticos.
Resultados del tratamiento en cncer cervicouterino microinvasor en el Instituto Nacional de
Cancerologa de Mxico (1980-1999)

1.- Mestwerdt G. Fruhdiagnose des Kollumkarzinoms. Zentralb Gynaekol, 1947 ;69 :326.
2. - Morrow CP, Curtin JP. Surgery for cervical neoplasia. In Gynecologic Cancer Surgery. New
York, Churchill Livingstone, 1996, p 472.3.
3. - Burghardt E, Holzer E. Diagnosis and treatment of microinvasive carcinoma of the cervix
uteri. J Obstet and Gynecol 1977; 49:641-653.

39.- Se trata de femenino de 35 aos la cual presenta un ndulo mamario palpable de


aparicin brusca. La ecografa revela un ndulo anecognico, de limites muy precisos,
morfologa regular y refuerzo posterior, nico de 3.5 cms. de dimetro. El diagnstico ms
probable es:
a) Cncer.
b) Displasia fibrosa.
c) Fibroadenoma.
d) Quiste

Quistes. Los quistes mamarios son fciles de detectar con la ecosonografa. Pueden ser
lesiones nicas o mltiples que se observan como imgenes redondeadas, anecognicas, de
paredes delgadas, contornos bien definidos, con importante reforzamiento acstico posterior y
sombras laterales delgadas. Pueden presentar septos intraqusticos y, en ocasiones, se
pueden observar ecos internos que sugieren detritus celulares o proceso inflamatorio. Se debe
descartar la presencia de lesiones intraqusticas o la coexistencia de otras alteraciones
benignas o malignas. En caso de ser sintomticos, el tratamiento adecuado es la puncin y
aspiracin de la lesin con aguja guiada por palpacin o ecosonografa de acuerdo con ell
tamao, profundidad y caractersticas del contenido. El uso del ultrasonido garantiza el
vaciamiento completo.

REFERENCIAS:
Barth V, Prechtel K. Mama normal. En: Barth V, Prechtel K, editores. Atlas de patologa de la
glndula mamaria.
2da ed. Madrid: Editorial Mdica Panamericana, 1991.
Bush H, McCredie A. Carcinoma of the breast during pregnancy and lactation. In: Allen HH,
Nisker JA. Cancer in pregnancy. New York: Futura Publishing Co. Inc., 1986.
Byrd BF, Bayer DS, Robertson JC, Stephenson JE Jr.
Treatment of breast tumor associated with pregnancy and lactation. Ann Surg. 1962; 155:940-7.

40.- Femenino de 36 aos, es atendida en consulta externa con reporte de papanicolaou que
reporta un NIC I, la especuloscopa se observa crvix con ectropin periorificiario.
El agente etiolgico ms probable causante de esta infeccin es:
a)
b)
c)
d)

neisseria gonorreae.
clamidya trachomatis.
virus del papiloma humano.
treponema pallidum.

9.5.2 Las pacientes a quienes se les realiz citologa cervical, cuyo resultado es LEIBG
(infeccin por VPH, displasia leve o NIC 1); LEIAG (displasia moderada y grave o NIC 2 y 3) o
cncer deben enviarse a una clnica de colposcopa, para realizar estudio colposcpico.
9.5.3 Si el resultado de la citologa es LEIBG, la colposcopa es satisfactoria y sin evidencia de
LEIBG, se realizar control citolgico en un ao (Apndice Normativo A)
9.5.4 Si la citologa es de LEIBG, la colposcopa es satisfactoria y existe evidencia de lesin, se
debe tomar una biopsia dirigida.
9.5.4.1 Si la biopsia dirigida es negativa, se realizar nueva colposcopa para verificar el
diagnstico y en caso necesario, tomar nueva biopsia dirigida y revalorar.
9.5.4.2 Si la biopsia dirigida es reportada como LEIBG se podr dar tratamiento conservador:
criociruga, electrociruga o laserterapia (slo si cumple con las condiciones referidas en el
Apndice 1) o se podr mantener a la paciente en vigilancia en la clnica de colposcopa, con
colposcopa y estudio citolgico cada seis meses, durante 24 meses.
Jueves 31 de mayo de 2007 DIARIO OFICIAL (Primera Seccin)
9.5.4.3 Si la biopsia dirigida es reportada como LEIAG (Lesin Intraepitelial Escamosa de Alto
Grado) se realizar tratamiento conservador (electrociruga o laserterapia). En las mujeres
posmenopusicas, dependiendo de las condiciones anatmicas del crvix, se realizar
tratamiento conservador en la clnica de colposcopa o tratamiento quirrgico (histerectoma
extrafascial) en el servicio que corresponda.
9.5.4.4 Si la biopsia dirigida reporta cncer microinvasor o invasor, la paciente se transferir a
un Servicio o Centro Oncolgico para su tratamiento correspondiente.
9.5.4.5 Si la citologa reporta LEIBG y la colposcopa es no satisfactoria, se tomar cepillado
endocervical (Apndice Normativo A)
9.6 En caso de colposcopa no satisfactoria, negativa a LEIBG y con cepillado endocervical
negativo, se continuar su control en la clnica de colposcopa en seis meses, con colposcopa
y citologa.
9.6.1.1 Si el cepillado endocervical reporta LEIBG se tratar a la paciente como LEIAG, con
mtodos conservadores escisionales.

Jueves 31 de mayo de 2007 DIARIO OFICIAL (Primera Seccin)


Modificacin a la Norma Oficial Mexicana NOM-014-SSA2-1994, Para la prevencin,
deteccin, diagnstico, tratamiento, control y vigilancia epidemiolgica del cncer
crvico uterino.
Al margen un sello con el Escudo Nacional, que dice: Estados Unidos Mexicanos.- Secretara
de Salud.
MODIFICACION A LA NORMA OFICIAL MEXICANA NOM-014-SSA2-1994, PARA LA
PREVENCION,
DETECCION, DIAGNOSTICO, TRATAMIENTO, CONTROL Y VIGILANCIA EPIDEMIOLOGICA
DEL CANCER CERVICO UTERINO.
El agente etiolgico del cncer de cuello uterino es el papiloma virus humano (hpv).
Existen lesiones precursoras del cncer cervical, son las llamadas lesiones intraepiteliales de
cuello uterino (tambin conocidas como sil: squamous intraepithelial lesion) son lesiones que
no atravesaron la membrana basal del epitelio y que por lo tanto no pueden invadir ni
diseminarse por el resto del cuerpo, como s lo hace un cncer invasor. anteriormente a las
lesiones intraepiteliales se las llamaba neoplasia intraepitelial cervical (tambin conocidas
como cin: cervical intraepithelial neoplasia), y anteriormente se las llamaba displasias de
cuello uterino.

41.- Paciente de 50 aos, con mioma uterino de tamao equivalente a una gestacin de 12
semanas, que presenta hipermenorreas y hemoglobinemia de 9 gr%. No se demuestra
patologa asociada. Se encuentra en espera para la prctica de una histerectoma programada
a realizar en 4 meses. En esta paciente est indicado el tratamiento preoperatorio con:
a) Estrgenos.
b) Inhibidores de la fibrinlisis.
c) Anlogos de la GnRH.
d) Derivados del cornezuelo del centeno.
Anlogos de la GnRH: Son derivados de la hormona GnRH en donde se ha realizado una
sustitucin peptdica en posicin 6 y en algunos casos en la 10, obteniendo compuestos hasta
unas 200 veces ms potentes debido a mayor afinidad por los receptores y a su resistencia a la
degradacin por peptidasas. Aunque su accin inicial produce un incremento en la produccin
de FSH-LH (efecto flure-up o llamarada) tras 56 das de exposicin contnua, los receptores son internalizados producindose un estado de
hipogonadismo hipogonadotropo y niveles de estradiol similares a los de la postmenopausia.
Los anlogos de GnRH estn disponibles en distintas frmulas: administracin nasal (varias
aplicaciones al da), subcutnea (aplicacin diaria) o intramuscular (preparados depot
mensuales o trimestrales) (Shaw RW 1999).
Marco Filicori y sus colaboradores de la Universidad de Bolonia fueron los primeros en utilizar
en 1983 los aGnRH en un estudio que confirm su eficacia para reducir el tamao de los
miomas uterinos y secundariamente sntomas como alteraciones menstruales, dolor plvico y
sntomas de presin local. Otros autores como Minaguchi H y colaboradores continan
comprobando la efectividad de los anlogos de la GnRH en el tratamiento del mioma uterino
tras evaluar en el ao 2000 seis estudios con un total de 602 pacientes tratadas con nafarelina.
La disminucin del tamao se calcula entre un 30-70%, y se ha observado como el mayor
porcentaje de reduccin ocurre tras el primer mes de tratamiento, no existiendo reducciones o
siendo stas mnimas despus del tercer mes
(Healy et al 1986; Friedman et al 1989; Matta et al 1989; Williams y Shaw 1990). En miomas
pediculados o con gran proporcin de calcio o colgeno (hialinizacin) la repuesta es tambin

menor. Debe tenerse en cuenta que si despus de dos meses de tratamiento no se ha


producido un significativo descenso del tamao del mioma, ste ya no debe ser esperado y
debe pensarse en la posibilidad de la existencia de un tumor muscular maligno no
diagnosticado (Messia AF et al 1998). Tras finalizar el tratamiento y recuperarse el estado de
hipogonadismo, el mioma retorna rpidamente a su tamao inicial (Friedman AJ et al 1987;
Matta WH et al 1989).
En casos prximos a la menopausia, la reduccin del tamao del mioma y su sintomatologa,
permitira hablar de una solucin mdica del problema, pero en todo caso la utilizacin de
aGnRH facilitara la intervencin quirrgica al acortar el tiempo de intervencin, la hemorragia y
el acceso a localizaciones complicadas como el caso de miomas interligamentarios o situados
en istmo o crvix. En el caso de la ciruga histeroscpica la reduccin del tiempo de ciruga
permitira reducir el volumen de fluidos aportado a cavidad uterina y los riegos de absorcin e
hiponatremia.
Los mecanismos de accin por los que los aGnRH actan son: o Hipoestrogenemia: es
necesario mantener la hipoestrogenemia, pues la elevacin de sus niveles lleva a un rpido
incremento del tamao del mioma. El crecimiento del mioma es dependiente de los niveles de
estrgenos (aumentan de tamao con el embarazo y se reducen durante la menopausia o el
tratamiento con aGnRH, pudiendo volver a crecer durante la THS), pero aunque los estrgenos
parecen ser importantes en el crecimiento del mioma, su relacin debe ser algo ms compleja
pues no se han descrito incrementos significativos del tamao de miomas durante el
tratamiento con gonadotrofinas en RA (situaciones con elevados niveles de E2), algunos de
ellos no se modifican durante el embarazo o incluso decrecen y se han encontrado
crecimientos despus del tratamiento con citrato de clomifeno (antiestrgeno). En relacin con
la hipoestrogenemia podran estar los cambios inducidos en el flujo vascular uterino
(incrementos en el ndice de resistencia de las arterias uterinas) que suponen una reduccin de
la vascularizacin o las modificaciones de distintos factores de crecimiento. o Cambios
histolgicos: el tratamiento con aGnRH puede producir degeneracin roja, infiltracin
linfocitaria, y necrosis, as como reduccin de la proliferacin celular e incremento de la
apoptosis. Pero en otras circunstancias no es posible encontrar 7 diferencias. No se ha
encontrado una relacin entre los cambios histolgicos y el porcentaje de reduccin del tamao
del tero, y existe una gran variabilidad entre distintas pacientes o entre distintos miomas de
una misma paciente, no existiendo pues un patrn histolgico caracterstico de respuesta ante
el tratamiento con aGnRH.
BIBLIOGRAFA:
1. Abad L, Abad de Velasco L, Parilla JJ. Etiopatogenia. Papel de las hormonas
esteroideas, factores de crecimiento y otras sustancias. Cuad Med Reprod
1999;5(1):15-29.
2. Albano C, Platteau P, Devroey P. Gonadotropin-releasing hormone antagonist: how good is
the new hope? Curr Opin Obstet Gynecol
2001;13(3):257-62.
3. Coutinho EM.Treatment of large fibroids with high doses of gestrinone.
Gynecol Obstet Invest 1990;30(1):44-47.
4. Chavez NF, Stewart EA. Medical treatment of uterine fibroids. Clin Obstet
Gynecol 2001;44(2):327-84.
5. De Leo V, la Marca A, Morgante G. Shortterm treatment of uterine
fibromyomas with danazol. Gynecol Obstet Invest 1999;47(4):258-262.
6. Eldar-Geva T, Healy DL. Other medical management of uterine fibroids.
Baillieres Clin Obstet Gynaecol 1998;12(2):269-88.
7. Felberbaum RE, Germer U, Ludwig M, Riethmuller-Winzen H, Heise S,
Buttge I, Bauer O, Reissmann T, Engel J, Diedrich K. Treatment of uterine fibroids with a slowrelease formulation of the gonadotrophin releasing hormone
antagonist Cetrorelix.HumReprod 1998;13(6):1660-8

42.- Se presenta paciente de 40 aos de edad a su consultorio refiriendo mastalgia que es


ms severa antes de la menstruacin. A la palpacin hay nodularidad excesiva, hiperestesia y
reas qusticas que la paciente refiere disminuyen en tamao despus de la menstruacin. El
diagnstico ms probable es:

a)
b)
c)
d)

Fibroadenomas
Papiloma intraductal
Mastopata fibroqustica
Cncer de mama

Es raro encontrar una mujer mayor de 35 aos a quien no le hayan dicho, en un examen fsico
mamario, ecogrfico o mamogrfico, que tiene quistes en la mama o que su mama es
mastoptica.
Es el trastorno benigno de la mama ms frecuente y consiste en un aumento del tejido
mamario, especialmente en las zonas superiores y externas de las mamas, hacia las axilas,
que las hace ms densas.
La mastopata fibroqustica suele presentarse en ambas mamas, aunque puede ser de
diferente intensidad en una que en otra.
Puede presentarse a cualquier edad despus del inicio de la menstruacin, pero es ms
probable que aparezca entre los 30 aos y la menopausia. Raramente se presenta ms tarde
de esa edad.
El origen de este trastorno es funcional y responde a desequilibrios de las hormonas sexuales
femeninas y puede condicionar la aparicin de quistes mamarios.

Los sntomas pueden fluctuar de leves a severos en una mastopata fibroquistica mamaria, se
acentan tpicamente antes de cada perodo menstrual y desaparecen inmediatamente
despus.
Los sntomas abarcan:

Consistencia de protuberancias (como de "guijarros"), irregular y densa del tejido


mamario
o generalmente ms notoria en la parte superior externa de la mama
Molestia en las mamas
o generalmente en ambas mamas
o puede ser persistente o puede aparecer y desaparecer
Sensacin de llenura en las mamas
Sensibilidad y dolor sordo e intenso
Sensibilidad y edema premenstrual
Secrecin ocasional del pezn

Bibliografa:

Jones III HW, Wentz AC. Tratado de Ginecologa de Novak. Editorial InteramericanaMcGraw Hill. Undcima Edicin 1994.
DiSaia-Creasman. Oncologa Ginecolgica Clnica. Editorial Mosby. Cuarta Edicin
1994.
Van Dinh T. Sumario de Patologa Ginecolgica. Editorial La Prensa Mdica Mexicana.
1992.
Pernoll ML. Diagnstico y Tratamiento Ginecoobsttricos. El Manual Moderno. Mxico.
Sexta Edicin 1991.
Alvarez-Bravo A. Diagnstico de los trastornos menstruales y hemorrgicos. En:
Alfonso Alvarez Bravo y su obra. Editorial Marketing y Publicidad SA. Tomo I. 1993.
Vzquez E. Aspectos histoqumicos del endometrio humano despus del tratamiento
con progestgenos sintticos. Gac Md Mx 1966; 96: 1277-93.
Huerta MR, Malacara JM, Rivera-Cisneros A, Daz Cisneros FJ. Sntomas en
adolescentes de dos ciudades de Mxico y su asociacin con el ciclo menstual. Ginec
Obstet Mx 1994; 62: 146-50.

43.- En una consulta prenatal de rutina, una mujer de 28 aos de edad, G5 P4, con 28 SDG,
refiere que no ha sentido movimiento fetal durante los ltimos 2 das. Su embarazo ha sido
complicado debido a que padece hipertensin crnica, para lo cual se le recetaron tabletas de
alfa-metildopa 2 veces al da. Al examen, su FU es de 30cm, y las maniobras de Leopold
demuestran que el feto se encuentra en situacin transversa. Su TA es 145/85mmHg. No se
encuentra latido cardiaco con el Doppler. El paso ms apropiado a seguir en el manejo de la
paciente es:
a)
b)
c)
d)

Realizar un test sin estrs


Amniocentesis
USG
Beta-HCG (cuantitativa)

Probable bito:
Sintomatologa y diagnstico Signos funcionales: No se perciben movimientos fetales por 1224 horas. Disminucin o ausencia de sntomas y/o signos como nauseas vmito, hipertensin,
albuminuria) Paraclnicos: *ecografa: diagnstico precoz y exacto: Doppler. *Radiologa: hay 3
signos: +deformacin del crneo +curvatura y torsin de la columna +presencia de gas en el
feto *lquido amnitico: puede estar meconiado, o sanguinolento Signos locales: en los senos
hay secrecin calostral, sangrado leve y oscuro por vagina, el feto se vuelve blando a la
palpacin, fetocardia (-), puede haber detencin y/o disminucin de la altura uterina, bajo peso
corporal, entre otros
Bibliografa: 1. OBSTETRICIA, Schwarcz R, editorial El ateneo, 2003. 2. Sociedad espaola de
ginecologa y obstetricia, junio 2002. 3. OBSTETRICIA CLNICA, Llaca V, edicin 2000,
captulo 24; Pg, 315-316.

44.- Femenino de 34 aos, es atendida en sala de partos secundario a eutocia, durante la


reparacin de la episiotoma media hay un marcado incremento en el sangrado transvaginal. El
manejo inmediato a realizar en ste caso es:
a) 20 unidades IV de oxcitocina
b) 0.2mg Im de metilergonovina
c) Masaje y compresin del fondo uterino

d) Empaquetar con gasas

Atona Uterina
a) Compresin bimanual externa.
b) Masaje uterino combinado (endouterino y abdominal).
c) Medicamentos: uso de oxitcicos.
d) Evacuacin uterina de cogulos.
e) Transfusin de sangre o hemoderivados.
f) Evaluar factores de coagulacin.
g) De persistir el cuadro clnico, proceder a:
1. Ligadura de arteria uterovrica.
2. Sutura de las paredes uterinas (puntos de colchonero).
3. Sutura de Lynch.
4. Ligadura bilateral de arterias hipogstricas.
5. Histerectoma abdominal subtotal.
6. Histerectoma abdominal total.
h) Hemoglobina - hematocrito control seriado.
(1) Gua Clnica Basada en las Evidencias: Manejo de la Hemorrasia Postparto. Centro
Latinoamericano de
Perinatologia y Desarrollo Humano (CLAP). Novedades del CLAP no 16, abril 2002.
(2) OMS/FNUAP/UNICEF/BM. Integrated Management Of Pregnancy And Chidbirth - IMPAC,
2000.
(Traduccin al Espaol: Manejo de las complicaciones del embarazo y el parto: Gua para
obstetrices y mdicos 2002
(3) OutLook. Prevencin de la Hemorragia Postparto: Manejo del Tercer Perodo del Parto.
Nmero

45.- Femenino de 25 aos, con embarazo de trmino, sin antecedentes de control prenatal. G
3. C-1. Se ingresa al servicio de obstetricia por presentar actividad uterina regular y dolorosa.
Ef.: Deambulante, tranquila, adecuada coloracin de tegumentos, abdomen con fondo uterino a
32 cm. con producto nico vivo en situacin transversa dorso inferior FCF 144, al tacto vaginal
crvix dilatado a 3 cm. y membranas ntegras. Se realiza cesrea con retencin de placenta e
invasin a vejiga.
La alteracin placentaria que presenta esta paciente es:
a)
b)
c)
d)

placenta increta
placenta acreta
placenta percreta
placenta marginal

Es la penetracin y adherencia anormal de la placenta en la pared uterina.

Se divide en:
Placenta acreta.
Placenta increta.
Placenta percreta.

ACRETA: Las vellosidades se adhieren al miometrio.

INCRETA: Penetran ms de la mitad del espesor del miometrio.

PERCRETA: Atraviesa todo el espesor del miometrio, llegando a la


serosa, incluso atravesndola y adhirindose a rganos vecinos.

Factores:
Endometriosis previa.
Tumores submucosos. (Miomas)
Cicatrz uterina previa. (Cesrea, miomectoma)
Implantacin baja. (Placenta previa)
Malformaciones placentarias. (Placenta extracorial)
Legrado enrgico previo.
Extraccin manual previa de una placenta.
Diagnstico transparto-:
Placenta retenida por ms de 20 minutos.
Imposibilidad para encontrar un plano de separacin placentaria cuando se intenta su
extraccin manual.
Hemorragia incontrolable despus de la pseudoextraccin.
El diagnstico histopatolgico corrobora el diagnstico clnico.
Escenario menos deseable.
Tratamiento:
Histerectoma Obsttrica.
Constituye una ciruga no planeada y secundaria al hallazgo del acretismo
placentario con sangrado incohercible.

Cesrea-Histerectoma. (Con diagnstico previo)


Ciruga planificada ante un correcto diagnstico prenatal.

Recomendacin ACOG:
Maduracin pulmonar intrauterina.
Inyectar al cordn umbilical 50 mg de metrotexate.
Ligar el cordn en el nacimiento placentario y dejar la placenta in-situ.
Embolizacin inmediata de arterias uterinas bilaterales, as como de ramas de
la divisin anterior de la arteria iliaca interna con alcohol polivinlico.
Continuar con 5 dosis I.M. de 50 mg de metrotexate y cuantificar niveles de
hCG.
Programar Histerectoma Total Radical Abdominal y/o Cistectoma parcial y/o
reseccin pared anterior recto.

Lee et al. Conservative Management of Placenta Percreta. Obstet Gynecol, 112(2):421-424

46.- Femenino 40 aos, G-3 P-2 A-1, se detecta de anemia ferropnica, de 9.5 g/dl, refiere
ciclos menstruales de 31,32 x 8,9 das de duracin, acompaados de cogulos, los cuales
aparecieron despus del nacimiento de su segundo hijo hace 12 aos. e.f.: con ligera palides
de tegumentos, S/V dentro de los parmetros normales, , genitales con evidencia de sangrado
activo, al tacto vaginal bimanual se detecta tero de consistencia firme voluminoso, irregular,
aproximadamente de 12 cm. anexos libres. El tratamiento para esta paciente es:

a)
b)
c)
d)

histerectoma total sin conservar anexos.


histerectoma total conservando anexos.
histerectoma vaginal.
histerectoma radical.

Tratamiento: La ciruga es el tratamiento ms comn del mioma uterino:


Miomectomia: Extirpar slo el mioma, los tumores nicos y accesibles, la reseccin
histeroscpica de miomas submucosos tambin es posible realizarlo con electrocoagulacin en
pacientes con hemorragias (>90%).
Histerectoma:
La ciruga puede ser abdominal o laparoscpica, la decisin final de la tcnica depender de la
eleccin del cirujano, en base al caso individual y a su experiencia, el procedimiento puede ser:
-histerectoma subtotal.
-histerectoma total (remocin cervical). Tcnicamente ms sencillo en su realizacin. La
incidencia de carcinoma con origen en el mun cervical es menor al 1% en nuestros das.

Bibliografa:
1. hanafi m. predictors of leiomyoma recurrence after myomectomy. am coll obstet ginecol
2005;
105: 877-880.
2. inclan j, mojarra j. miomectoma histeroscpica. abordaje actual para el manejo de los
miomas
submucosos. reporte de un caso y revisin de la literatura. bol clin hosp. infant edo son
2001;18: 29-34.
3. goldrath mh, husain m. the hysteroscopic management of endometrial leiomyomatosis. j am
assoc gynecol laparosc 1997; 4: 263-267.
4. clement pb, scully re. mullerin adenofibroma of the uterus with invasion of myometrium and
pelvic veins. int j gynecol pathol 1990; 9: 363-371.
5. ravina jh, herbreteau d, ciraru-vigneron n, bouret jm, houdart e, aymard a, merland jj. arterial
embolization to treat uterine myomata. lancet 1995; 346: 671-672.

6. kunhardt-urquiza e, cruz si, fernndez-martnez rl, hernndez-ziga ve: miomatosis de


localizacin poco frecuente. ginecol obstet mex 1997; 65: 541-544.

47.- Paciente femenino de 18 aos diagnosticada de embrazo ectpico mediante ecografa que
tambin muestra gran cantidad de lquido libre en Douglas, con mal estado general. No tiene
hijos y conserva la trompa contralateral en perfecto estado. El tratamiento de eleccin es:
a) Expectante y determinacin del B-HCG.
b) Salpingectoma.
c) Legrado de cavidad uterina.
d) Alta y revisin en una semana

SANGRADOS 1era MITAD EMBARAZO


EMBARAZO ECTOPICO Tratamiento
Quirrgico.

Salpingostoma lineal.
Salpingectoma total.
Ordeamiento fimbrial.
Legrado uterino instrumental post-laparoscpico.
(Reaccin deciduoide)

Mdico.
Metrotexate y Mifepristona (RU 486)

48.- Se trata de femenino de 31 aos nuligesta pero con actividad sexual regular, sin mtodo
de planificacin familiar, con ciclos regulares, sin leucorrea, refiere dispareunia profunda,
sangrado intermestrual y dismenorrea secundaria ocacionalmente presenta urgencia urinaria.
El diagnstico clnico ms probable es:

a)
b)
c)
d)

Enfermedad plvica inflamatoria


Absceso tubo ovrico
Endometritis
Endometriosis

La endometriosis consiste en la aparicin y crecimiento de tejido endometrial fuera del tero,


sobre todo en la cavidad plvica como en los ovarios, detrs del tero, en los ligamentos
uterinos, en la vejiga urinaria o en el intestino. Es menos frecuente que la endometriosis
aparezca fuera del abdomen como en los pulmones o en otras partes del cuerpo.
La endometriosis es una enfermedad relativamente frecuente, que puede afectar a cualquier
mujer en edad frtil, desde la menarquia hasta la menopausia, aunque algunas veces, la
endometriosis puede durar hasta despus de la menopausia. La endometriosis altera la calidad
de vida de las mujeres que la padecen, afectando a sus relaciones de pareja, familiares,
laborales y de reproduccin.

Sntomas
Los sntomas clsicos son la dismenorrea, dolor plvico, dispareunia, sangrados
intermestruales y en muchos casos, esterilidad.
El dolor no tiene que ver con el tamao y la severidad de la lesin; generalmente cuanto
menor es la lesin mayor dolor produce. El dolor se agrava con las menstruaciones y en los
casos en que la lesin ocupa el fondo de saco de Douglas, puede dar dispareunia. Existe un
aumento de la PGF2 alfa y PGE2 y un aumento de las contracciones uterinas que podra
deberse a un depsito de endometrio en la cavidad peritoneal.
La esterilidad debido a la endometriosis podra deberse a distintas causas de acuerdo a la
severidad de la patologa. En los casos de endometriosis severa puede haber un factor
tuboperitoneal con adherencias y alteracin en la anatoma de la pelvis que interfiera con el
transporte del esperma y el vulo. En los casos de endometriosis leve hay varios mecanismos
propuestos que justifican su relacin con la infertilidad: foliculognesis alterada, fase ltea
inadecuada, fagocitosis espermtica, mala calidad ovocitaria, embriotoxicidad y alteracin a
nivel de la implantacin.. La produccin de prostaglandinas por el endometrio ectpico puede
afectar la motilidad tubaria, la foliculognesis y la funcin del cuerpo lteo. Puede haber un
aumento de la activacin de los macrfagos peritoneales en la endometriosis que cause la
fagocitosis de los espermas o la secrecin de citoquinas que pueden ser txicas para el
embrin. Segn algunos investigadores habra un 60% de las mujeres con endometriosis que
presentan un sndrome de Folculo Luteinizado no roto (LUF) en el cual el folculo no se rompe
en la ovulacin y el vulo queda atrapado.
Referencias bibliogrficas

1. Ruiz V. Endometriosis y fertilidad. Ed. Acosta y Warman, pp. 99


2. Lpez ,VH. Palomo E. Incidencias de endometriosis en una poblacin infrtil. XXI Congreso
nacional de Ginecologa y Obtetricia. Guatemala, 1993.
3. El-Eoley, et al. Danazol but not ginadotropin releasing hormone agonists suppresses
autoantibodies in endomeriosis. Fertil Steril 1990; 54:725

4. Acosta AA. Buttram VC Jr. Besch PK, Malinak LR, Van Der Heyden J. A.proposed
classfication of pelvic endometriosis. Obstet Gynecol 1973;42:19.
5. Buttran VC Jr. Evolution of the revised American Fertility classification of endometriosis. Fert.
Steril 1985; 43: 347
6. Lpez VH. Tratamiento mdico-quirrgico de la endometriosis. Simposio El rostro cambiante
de la endometriosis panam 3. 12. 1993.
7. Steinleitner A. Heterolous transplation of activated murine peritonel macrophages inhibitis
gamete interaction in vivo; A paradigm fo endometriosis associted subfertility. Fertil Steril 1990;
54:725.
8. Damewood M. Effect of serum from patients with minimal to mild endometriosis on mouse
embryo growth. Fertil Steril 1990; 54: 917
9. Proug S. Peritoneal fluid fracctions from patients with endometriosis do not promote two-cell
mouse embryo growth. Fertil Steril 1990; 54: 927.

49.- Femenino de 39 aos con embarazo de 32 semanas. Ingresa al servicio de urgencias por
presentar cuadro de 1 da de evolucin por sangrado transvaginal leve, rojo brillante, sin
contractilidad uterina. A.G.O.: G-3, P-0, C-2. E.F.: IMC: 30 kg/m2, F.U. de 28 cm, fcf presente,
especuloscopa crvix cerrado con huellas de sangrado. USG reporta producto nico vivo con
fetometra normal, lquido amnitico normal y placenta anterior que cubre parcialmente el
orificio cervical interno.
El siguiente paso en la atencin de esta paciente es:

a)
b)
c)
d)

manejo por consulta externa y vigilar sangrado


manejo por consulta externa con uteroinhibidores
ingreso a hospital e inductores de maduracin pulmonar
ingreso a hospital y cesrea.

50.- Femenino de 45 aos es atendida en consulta externa por presentar desde hace 6 meses
flujo transvaginal, mucosanguinolento y sinusorragia importante, refiere dispareunia de 3
meses de evolucin. AGO. G-5 P-5. Oclusin tubrica bilateral hace 6 aos. E.F.: TA 130/80
MMHG, FC 80 LPM, TEMP. 36.7 C. Laboratorio: HB 9.7 G/DL, HTO 37 %, Se reporta
Papanicolaou clase V.
El siguiente paso para confirmar el diagnstico es:
a)
b)
c)
d)

histeroscopa.
ecosonografa.
colposcopa.
biopsia dirigida.

El examen citolgico de papanicolau, realizado como examen de tamizaje en ginecologa,


permite establecer las caractersticas de las clulas que se descaman del cuello uterino. La
tcnica de papanicolau ha permitido disminuir la mortalidad e incidencia de cncer invasor de
cuello uterino. Se define examen de papanicolau anormal cuando se presentan alteraciones en
el ncleo, citoplasma y/o la relacin ncleo/citoplasma de las clulas examinadas. El

papanicolau anormal, de acuerdo a la clasificacin de bethesda abarca desde un diagnstico


citolgico de ascus (siglas en ingls de clulas atpicas de significado no determinado), lesin
escamosa intraepitelial (lei) de bajo grado (corresponde a displasia leve y cambios por
papiloma virus), lesin escamosa intraepitelial (lei) de alto grado (displasia moderada, displasia
severa o carcinoma in situ) y clulas de cncer invasor.
4.6. Colposcopa
Ideada por Hinselmann en Alemania en 1924, lleg a Amrica por el cono sur.
Consiste en la visualizacin y amplificacin del cuello uterino mediante un sistema binocular de
lentes, entre 25 y 40 aumentos, lo cual permite la observacin de las estructuras del cuello
uterino mediante la asociacin con imgenes preestablecidas. La colposcopa tiene una mayor
sensibilidad que la citologa, pero su menor especificidad. de conducir a procedimientos
diagnsticos invasivos (biopsias y conizaciones) innecesarios y su mayor costo, son sus
principales limitaciones. Combinadas la citologa y la colposcopia brindan una seguridad
diagnstica que excede EL 95%.
4.6.1. Indicaciones de la colposcopia
La colposcopa est indicada en las siguientes circunstancias (20), (21):
- pacientes con citologa cervical ( papanicolaou) clase III, IV o V, o sus equivalentes en los
otros sistemas de clasificacin.
- pacientes con citologa clase II con atipia inflamatoria, escamosa o endocervical, o cuando se
informe la presencia de coilocitos.
- pacientes con crvix macroscpicamente normal, pero quienes presentan
sinusorragia.
- pacientes con crvix macroscpicamente anormal, en ausencia de carcinoma evidente.
- pacientes con citologa clase II persistente, pese a tratamiento de posibles causas. (ejemplo:
trichomonas).
Tamizaje en cncer ginecolgico autores de la gua.
Dr. Miguel Bueno Montao
Profesor asociado departamento de ginecologa universidad libre gineclogo centro mdico
Imbanaco profesor titular de ginecologa y obstetricia universidad del valle Dr. Jaime Rubiano
universidad del valle Dra. Derry Trujillo
51.- Femenino de 20 aos de edad refiere irregularidades menstruales tipo hipo-opsooligomenorrea desde hace 3 aos. Niega tener vida sexual activa y no recuerda su fecha de
ltima menstruacin. No hay antecedente de galactorrea ni de uso de hormonales exgenos.
Mide 164cm y pesa 60kg. Sin datos de hirsutismo, las mamas, tero y anexos son normales.
El ultrasonido plvico es normal, as como el perfil hormonal. El diagnstico clnico ms
probable es:
a)
b)
c)
d)

Hiperplasia del endometrio


Sangrado uterino disfuncional
Alteracin menstrual fisiolgica
Endometriosis

En la prctica mdica se utiliza un grupo de trminos para hacer referencia a las diversas
alteraciones del ciclo menstrual, que requieren precisin por la frecuencia en que son
diagnosticados.
Segn Schiavon (2000), las alteraciones menstruales ms frecuentes son:
oligoamenorrea: episodios de sangrado infrecuentes, irregulares, con intervalo de ms de
cuarenta das;
polimenorrea: episodios frecuentes pero regulares de sangrado uterino, que ocurren a
intervalos menores de veintin das;
menorragia: sangrado excesivo, tanto en cantidad como en duracin, que ocurre con

regularidad y es sinnimo de hipermenorrea;


metrorragia: sangrado generalmente no excesivo, que ocurre a intervalos irregulares;
menometrorragia: sangramiento generalmente excesivo y prolongado, que ocurre a intervalos
frecuentes e irregulares;
hipomenorrea: sangrado uterino regular, pero disminuido en cantidad;
sangrado intermenstrual: sangrado uterino generalmente no excesivo, que ocurre entre
perodos menstruales regulares.
Las irregularidades menstruales son causa frecuente de consulta en las adolescentes, siendo
95 % de las veces de naturaleza disfuncional, por inmadurez del eje hipotlamohipofiso
ovrico (HHO).
Uno de los primeros problemas que hay que plantear ante estas irregularidades menstruales,
es la hemorragia uterina disfuncional (HUD). Su definicin guarda relacin con las
caractersticas en cantidad y frecuencia que difieren del sangrado menstrual normal.
Con mayor frecuencia se encuentra en forma de sangrados excesivos y prolongados,
asociados a ciclos anaovulatorios, en ausencia de una patologa o enfermedad existente,
aunque raramente la HUD puede presentarse con ciclos ovulatorios.
De forma prctica, consideramos una hemorragia uterina (HU) como anormal cuando el
sangrado es excesivo, con cualquier desvo o alteracin de su duracin, cantidad o intervalo.
El diagnstico de HUD supone una alteracin de origen endocrino (eje HHO); por lo tanto, su
diagnstico impone haber descartado cualquier patologa orgnica y sistmica que produzca
hemorragia genital. Es un diagnstico por exclusin.
ETIOPATOGENIA DE LA HUD EN LA ADOLESCENCIA
Los ciclos anovulatorios son ms frecuentes en las adolescentes por la inmadurez del eje HHO
en el primer ao tras la menarquia.
En esos casos de HUD, como ya se mencion, se producen ciclos anovulatorios que se
traducen en una proliferacin desorganizada del endometrio por falta de efecto progestagnico.
Una vez que el endometrio alcanza un grosor crtico, comienza a descamarse en forma
irregular, traducindose en un sangrado permanente de cuanta variable.
Las manifestaciones clnicas de la HUD son:
fases de amenorrea de dos a cuatro meses, seguidas de salida de sangre abundante durante
tres o cuatro semanas; en oportunidades existe irregularidad completa en el sangrado;
sangrado de ms de seis compresas (bien empapadas) al da;
presencia de cogulos;
suele ser indolora;
menstruaciones de ms de siete das de duracin;
ciclos de menos de veintin das.
La gravedad de esta hemorragia se clasifica, de acuerdo con el grado de anemia que
produzca, en metrorragia leve, moderada o grave:
Leve:
metrorragia leve y prolongada,
ciclo menstrual acortado,
hemoglobina y hematocrito normales.
Moderada:
metrorragia copiosa prolongada,
ciclo menstrual acortado,
anemia leve (cifras de hemoglobina inferior a diez gramos por litro).
Grave:
metrorragia copiosa prolongada,

ciclo acortado e irregular,


anemia grave (cifras de hemoglobina de ocho gramos por litro o menos).
Ante un sangramiento uterino en estas edades se debe realizar el diagnstico diferencial con:
a) gestacin y problemas relacionados con sta, tales como abortos y gravidez ectpica;
b) coagulopata: 20 % de las adolescentes con hemorragia uterina tienen un defecto de la
coagulacin. La manifestacin ms precoz de alteraciones de la coagulacin sangunea puede
ser evidenciada por un sangramiento genital anormal, lo cual puede estar relacionado con
deficiencias de plaquetas, leucemias, prpuras, enfermedad de Von Willebrand, deficiencia de
protrombina u otros factores de la coagulacin;
c) malformaciones del aparato genital, traumatismosgenitales, presencia de cuerpos extraos;
d) dispositivos intrauterinos;
e) tumores uterinos, sarcoma botroides o tumores anexiales;
f) hipo o hipertiroidismo;
g) insuficiencia renal o heptica.
Como el diagnstico de HUD es de exclusin, hay que hacer una historia clnica minuciosa,
exmenes complementarios y sin falta descartar las otras causas de sangramiento
transvaginal.
Se debe precisar con detalle el nivel de desarrollo puberal, la actividad sexual y la presencia de
situaciones concomitantes como: a) contacto sexual sin proteccin contraceptiva;
b) uso irregular de anticonceptivos orales o antecedentes de insercin de dispositivos
intrauterinos;
c) ejercicios fsicos extenuantes;
d) historia previa de sangrado excesivo, asociado a extracciones dentarias, pequeas heridas,
epistaxis y otras;
e) dolencias renales u hepticas preexistentes.
Ante cualquier demanda de atencin por adolescentes con sangramiento genital con las
caractersticas descritas, se requiere de un examen fsico general que incluya exploracin
general completa, toma de tensin arterial y pulso, bsqueda de exoftalmia, fascie Cushing,
visceromegalias o presencia de masas abdominales palpables, edemas parpebrales y de
miembros superiores, as como puntos hemorrgicos en epidermis y otros signos de
coagulopata.
El examen ginecolgico debe realizarse en todas las adolescentes, con excepcin de las que
no han tenido actividad sexual y presentan sangramiento leve. Adems del examen de sus
genitales, hay que efectuar una valoracin citolgica y microbiolgica en particular en quienes
presenten manifestaciones clnicas.
En la inspeccin de los genitales durante el examen de la paciente, es importante evaluar que
el sangramiento se origine en lesiones ubicadas en los genitales externos, uretra u
hemorroides, as como indagar acerca de la posibilidad de abuso sexual. Si despus del
tratamiento de la HUD leve contina el sangrado, se recomienda la realizacin de ecografa
abdominal para precisar el diagnstico.
En pacientes que ya han tenido relaciones sexuales se debe buscar si el tero tiene
caractersticas gravdicas, la posibilidad de un aborto en curso, as como la presencia de una
masa anexial que permita corroborar la existencia de embarazo o alguna neoplasia benigna o
maligna. El examen con espculo podr demostrar un cuello hipermico, sangrante o gravdico;
de encontrarse estos hallazgos, se descartara la etiologa disfuncional del sangrado.
Exmenes complementarios bsicos que no pueden faltar:
hemograma completo,
coagulograma completo,
orina,
ultrasonido ginecolgico abdominal, transvaginal o transrectal segn proceda,
ecografa abdominal. Si fuese necesario por los signos identificados en el examen de la

paciente, se deben realizar:


dosificacin de FSH, LH, T3, T4, TSH y prolactina si hay sospechas clnicas de otras
enfermedades endocrinas concomitantes,
laparoscopia en casos seleccionados por patologa de base,
otros, segn hallazgos de la historia clnica y la exploracin.

52.- Femenino de 55 aos, se queja de presin plvica y una masa en la entrada vaginal.
Antecedentes: G.3 P.3, el ltimo con peso al nacer de 4,500 grs. FUR hace tres aos. Sin
terapia de reemplazo. Historia de tabaquismo positivo a razn de 40 cajetillas ao. Actualmente
refiere dificultad para evacuar, tos crnica, presenta una orina de 60 cc. En la exploracin
plvica, el hallazgo ms probable es:
a)
b)
c)
d)

Rectocele
Cistocele
Enterocele
Uretrocele

El Rectocele es una hernia de la pared anterior del Recto hacia la porcin posterior de la
vagina. La incidencia real de esta deficiencia anatmica es desconocida y en muchas
ocasiones es un resultado del paso del tiempo. Es un hallazgo muy frecuente del examen
perineal, siendo en mltiples ocasiones asintomtico.
El rectocele puede ser un hallazgo importante del sndrome de Obstruccin Defecatoria
(SOD). No debe tomarse como una deficiencia anatmica nica, sino como parte importante de
un problema anatomo-fisiolgico complejo.
Un principio importante es la etiologa y anatomo-patologa del rectocele. Existen varias teoras
sin consenso principal. La existencia, deficiencia o alteraciones del septo rectovaginal son
controversias importantes. No existe una fascia visceral que separe el recto de la vagina o que
forme un septo especfico. Existe frontera entre donde termina la pared anterior del recto y
donde comienza la pared de la vagina, pero mltiples estudios no han encontrado un septo
rectovaginal especfico. El septo puede estar formado de una pelcula casi transparente hasta
una pared de consistencia fibromuscular fuerte.
El rectocele es un hallazgo comn. Se presenta en el 80% de las pacientes femeninas y
13% de los masculinos en una defeco grafa (> a 1cm.) Entre ms grande es el rectocele,
mayor son los sntomas asociados, Dificultad en la evacuacin, constipacin crnica, dolor
rectal y perineal, sensacin de masa y en ocasiones sangrado. La necesidad de presin
manual para ayudar a la evacuacin o para vaciar el rectocele es comn en ms del 50% de
los pacientes.
El diagnostico se hace con un simple tacto rectal e inspeccin vaginal, pero debe recordarse
que rara vez es el rectocele un hallazgo aislado. La presencia cistocele, peritoneocele,
enterocele u otros prolapsos perineales y problemas funcionales deben ser descartados antes
de proponer la reparacin quirrgica del rectocele. Defeco grafa,
Pruebas de funcin fisiolgica del piso plvico, pruebas de funcin urinaria, evaluacin del
esfnter anorectal por ultrasonido y hasta la resonancia magntica han sido propuestos antes
de la ciruga.

Bibliografa:
Rectocele: Pathogenesis and surgical managment. Zbar AP, Linemann A, Fritsch H,
Beer-Gabel M, Pescatori M. Int J Colorectal Dis. (2003) 18:369-384.
Evaluation and Treatment of Women with rectocele. Cundiff GW, Fenner D, Obstetrics
and Ginecology 104(6): 1403-1416
Stapled transanal rectal resection to treat obstructed defecation caused by rectal
intussusseption and rectocele. Renzi A, Izzo D, Di Sanrno (26) 21:661-667
Rectocele repair using biomaterial augmentation. Altman D, Melgren A, Zetterstrom J.

Obstet Gynecol (2005) 60(11)753-760.

53.- Femenino de 22 aos de edad, acude al servicio de urgencias por referir malestar general,
fosfenos, nusea y vmito. Antecedentes: cursa con embarazo de 36.5 SDG, G2, A1, C0.
Exploracin fsica: TA 185/110 mmHg, FC 120 lpm, FR 35 x, T37.9, alerta, inquieta, aprecia
ictericia, cardiopulmonar sin compromiso, abdomen globoso a expensas de tero grvido, se
detecta PUVI, longitudinal, ceflico, dorso a la derecha, FCF 142 x, al tacto genital crvix,
central, formado, cerrado, extraccin de guantes sin evidencia de prdidas genitales,
extremidades inferiores edema +++.
La medida teraputica inicial en esta paciente es:
a)
b)
c)
d)

Diurticos.
Sedantes.
Antihipertensivos.
Analgsicos.

Referencia:
La preeclampsia se define por aumento de la presin arterial y la presencia de proteinuria
durante el embarazo.
1) Leve: las pacientes suelen tener unas cuantas manifestaciones, y su presin arterial
diastlica es menor de 110 mmHg. En ocasiones hay edema. La cifra de plaquetas es
mayor de 100 000/l.
2) Grave: los sntomas son ms notorios y persistentes. La presin arterial casi siempre
es con niveles mayores de 160/110 mmHg. Puede haber trombocitopenia que avance
hasta coagulacin intravascular diseminada.
Se requiere hospitalizacin para las mujeres con preeclampsia; debe obtenerse una biometra
hemtica completa con cifra de plaquetas y determinacin de electrolitos, que adems incluya
enzimas hepticas. Se obtiene una muestra de orina de 24 horas para determinar la
depuracin de creatinina y protenas totales al ingreso hospitalario.
Se debe controlar la hipertensin arterial, para evitar sufrimiento fetal, as como empeoramiento
de la paciente.
Bibliografa:
1. McPhee S, Papadakis M, et. al. Diagnstico Clnico y Tratamiento 2010. Lange, McGraw
Hill,
49 edicin, Mxico, 2010.
2. Sibai BM, Diagnosis, prevention, and management of eclampsia. Obstet Gynecol. 2005.
Feb; 105;: 402 410.

54.- Femenino de 42 aos portadora de DM, acude a consulta quejndose de descarga


vaginal prurtica y blanquecina. De los siguientes exmenes de diagnsticos el ms til para
identificar el patgeno es:
a)
b)
c)
d)

Wet prep.
Tincion de Gram
PH
KOH

Prueba del KOH es un procedimiento en el cual hidrxido del potasio (KOH) se utiliza detectar
hongos disolviendo las clulas humanas en una cultura. La diferencia en la composicin de la
pared de clula de clulas humanas y de clulas fungicidas permite que este procedimiento
ayude a distinguir las dos clulas. El KOH desnaturaliza las protenas en la clula humana;
solamente sigue habiendo las clulas fungicidas ser considerado debajo del microscopio.
1. Bernal B. Fisiologa y ecologa de la vagina. Rev Chil Obstet Ginecol 1986; 51:56-60.
2. Ibrcena E. Vaginosis bacteriana; diagnstico y prevalencia. XII Congreso Peruano de
Obstetricia y Ginecologa 1996; 204-6.
3. Scapini JC, Guzmn CA. Deteccin de bacilos Gram negativos curvos anaerobios en
pacientes con vaginosis. Obstet Ginecol Latinoam 1986; 44: 320-5.
4. Soihet S. El flujo vaginal en la consulta ginecolgica. Ginecol Obstet
55.- Mujer de 26 aos, es atendida en consulta en la clnica de displasias por papanicolau con
lesin NIC I. Antecedentes: menarca 14 aos, ritmo 30x5 eumenorreica, inicio de vida sexual a
los 15 aos, 2 parejas sexuales, mtodo de planificacin familiar oclusin tubaria bilateral,
gestas 3 partos 3, crvix con lesin acetoblanca con extensin lineal de 2 cm. Se realiza
biopsia de la lesin, en caso de corroborarse el diagnstico, el siguiente paso en el manejo de
esta paciente es realizar:
a)
b)
c)
d)

biopsia.
electrociruga.
cepillado de canal.
ultrasonido endovaginal.

9.5.2 Las pacientes a quienes se les realiz citologa cervical, cuyo resultado es LEIBG
(infeccin por VPH, displasia leve o NIC 1); LEIAG (displasia moderada y grave o NIC 2 y 3) o
cncer deben enviarse a una clnica de colposcopa, para realizar estudio colposcpico.
9.5.3 Si el resultado de la citologa es LEIBG, la colposcopa es satisfactoria y sin evidencia de
LEIBG, se realizar control citolgico en un ao (Apndice Normativo A)
9.5.4 Si la citologa es de LEIBG, la colposcopa es satisfactoria y existe evidencia de lesin, se
debe tomar una biopsia dirigida.
9.5.4.1 Si la biopsia dirigida es negativa, se realizar nueva colposcopa para verificar el
diagnstico y en caso necesario, tomar nueva biopsia dirigida y revalorar.
9.5.4.2 Si la biopsia dirigida es reportada como LEIBG se podr dar tratamiento conservador:
criociruga, electrociruga o laserterapia (slo si cumple con las condiciones referidas en el
Apndice 1) o se podr mantener a la paciente en vigilancia en la clnica de colposcopa, con
colposcopa y estudio citolgico cada seis meses, durante 24 meses.
Jueves 31 de mayo de 2007 DIARIO OFICIAL (Primera Seccin)
9.5.4.3 Si la biopsia dirigida es reportada como LEIAG (Lesin Intraepitelial Escamosa de Alto
Grado) se realizar tratamiento conservador (electrociruga o laserterapia). En las mujeres
posmenopusicas, dependiendo de las condiciones anatmicas del crvix, se realizar
tratamiento conservador en la clnica de colposcopa o tratamiento quirrgico (histerectoma
extrafascial) en el servicio que corresponda.
9.5.4.4 Si la biopsia dirigida reporta cncer microinvasor o invasor, la paciente se transferir a
un Servicio o Centro Oncolgico para su tratamiento correspondiente.
9.5.4.5 Si la citologa reporta LEIBG y la colposcopa es no satisfactoria, se tomar cepillado
endocervical (Apndice Normativo A)
9.6 En caso de colposcopa no satisfactoria, negativa a LEIBG y con cepillado endocervical
negativo, se continuar su control en la clnica de colposcopa en seis meses, con colposcopa
y citologa.
9.6.1.1 Si el cepillado endocervical reporta LEIBG se tratar a la paciente como LEIAG, con
mtodos conservadores escisionales.

Jueves 31 de mayo de 2007 DIARIO OFICIAL (Primera Seccin)


Modificacin a la Norma Oficial Mexicana NOM-014-SSA2-1994, Para la prevencin,
deteccin, diagnstico, tratamiento, control y vigilancia epidemiolgica del cncer
crvico uterino.
Al margen un sello con el Escudo Nacional, que dice: Estados Unidos Mexicanos.- Secretara
de Salud.
MODIFICACION A LA NORMA OFICIAL MEXICANA NOM-014-SSA2-1994, PARA LA
PREVENCION,
DETECCION, DIAGNOSTICO, TRATAMIENTO, CONTROL Y VIGILANCIA EPIDEMIOLOGICA
DEL CANCER CERVICO UTERINO.

56.- A 38-year-old man comes to the physician because of slowly progressive visual problems
that make him bump into objects on both sides. He also reports that, while driving, he has
trouble switching lanes because he needs to turn his head all the way backward to look for
other cars. Ocular examination shows bitemporal field loss with preserved visual
acuity.Examination of the fundus is unremarkable. Which of the following is the most likely
diagnosis?

a)
b)
c)
d)

Pituitary adenoma
Occipital lobe meningioma
Optic neuritis
Retinal detachment

. PRESENTACIN CLNICA
Los tumores hipofisarios se reconocen clnicamente por uno o ms de tres patrones de
presentacin muy constantes:
- Sntomas de hipersecrecin hipofisiaria
- Sntomas de hiposecrecin hipofisaria
- Sntomas neurolgicos
El tercer patrn de presentacin es el dominado por los sntomas neurolgicos, aislados o
coexistentes con una o varias de las alteraciones endocrinolgicas antes descritas. Como se
ha dicho, una masa hipofisaria progresivamente creciente generar una constelacin de signos
y sntomas neurolgicos que dependern de la trayectoria del crecimiento y de las estructuras
nerviosas vecinas que resulten alteradas. Los sntomas y signos a encontrar son:
a. Cefaleas: Pueden ser un signo precoz y se atribuyen al estiramiento de la duramadre que
recubre la hipfisis o del diafragma de la silla turca. Est presente en el 75% de los casos y se
localiza ms frecuentemente en la regin frontal y orbital.
b. Prdida de visin: Debido a la compresin de los nervios pticos o del quiasma ptico. El
patrn clsico de prdida visual es una hemianopsia bitemporal a menudo asociada con
disminucin de la agudeza visual. Posteriormente puede haber una ceguera completa de uno o
ambos ojos.
c. Manifestaciones hipotalmicas: Debido a la compresin del hipotlamo por grandes
adenomas hipofisarios que provocan:

- Alteraciones del sueo


- Alteraciones de la atencin
- Alteraciones de la conducta
- Alteracin de la alimentacin
- Alteracin de las emociones
d. Hidrocefalia obstructiva: Debido a la infiltracin de las lminas terminales que hace que el
tumor penetre en la regin del III ventrculo, ocasionando obstruccin a la salida de LCR.
e. Compromiso de nervios craneales. Debido a la extensin lateral del tumor hacia la regin
del seno cavernoso, por donde transcurren los nervios craneanos. As las manifestaciones ms
caractersticas son:
- Ptosis: Por compromiso del III nervio craneal (motor ocular comn).
- Dolor o alteraciones sensitivas faciales. Por compromisos de las ramas oftlmica y maxilar
superior del Nervio Trigmino.
- Diplopia: Por compromiso del III, IV y VI nervios craneales.

Bibliografa
1. Kovacs K, Horvath E, Vidal S: Classification of pituitary adenomas. J Neurooncol 54 (2):
121-7, 2001.
2. Ironside JW: Best Practice No 172: pituitary gland pathology. J Clin Pathol 56 (8): 5618, 2003.
3. Scheithauer BW, Kovacs KT, Laws ER Jr, et al.: Pathology of invasive pituitary tumors
with special reference to functional classification. J Neurosurg 65 (6): 733-44, 1986.
4. Ezzat S, Asa SL, Couldwell WT, et al.: The prevalence of pituitary adenomas: a
systematic review. Cancer 101 (3): 613-9, 2004.

57.- An 18-year-old woman complains of myalgias, a sore throat, and painful mouth sores for 3
daysduration. Her temperature is 38.2 C (100.8 F), blood pressure is 110/80 mm Hg, pulse is
84/min, respirations are 15/min. Her gingival are edematous and erythematous, and there are
vesicles on her right upper and lower lips. Her pharynx is mildly erythematous but without
exudates, and there is tender mobile cervical lymphadenopathy. Her breath is not fetid, and the
dentition is normal. Which of the following is the most likely causal agent?
a) Actinomyces israelii
b) Herpes simplex virus 1
c) Nocardia asteroids
d) Streptococcus pyogenes

Las infecciones por herpes simplex son comunes en la prctica diaria, y con frecuencia el
paciente acude a los servicios de urgencia. Estas infecciones son ocasionalmente recurrentes,
generalmente dolorosas y asociadas con sntomas sistmicos, por lo cual el mdico de
urgencias debe estar familiarizado con el cuadro clnico y su manejo.
Existen dos variedades de virus del Herpes simplex (VHS) capaces de causar infeccin en el
hombre: el tipo 1 (VHS-1) y el tipo 2 (VHS-2) que se distinguen entre s por varias
caractersticas, incluyendo sus comportamientos clnico y epidemiolgico, antigenicidad,
composicin del ADN y la sensibilidad a diferentes agentes fsicos y qumicos (Cuadro No.1)
Cuadro No. 1
DIFERENCIAS ENTRE LOS VIRUS HERPES SIMPLEX TIPOS 1 Y 2
Caractersticas clnicas
Va de transmisin
Sndromes caractersticos
Reactividad a antgenos
monoclonales especficos

VHS-1
Oral
Oral-facial
Ocular
Encefalitis
Paroniquia
VHS-1 especfico

VHS-2
Genital
Genita
PerianaL
Neonata
Paroniquia
l VHS-2 especfico

El VHS-1 es de localizacin primordialmente extragenital, con predileccin por los tejidos de


origen ectodrmico, mientras que el VHS-2 corresponde al "Herpes progenitalis" descrito por
separado, dentro de las infecciones de transmisin sexual.
La regin oral es la localizacin habitual del Herpes simplex 1, el cual es causa frecuente de
lesiones orofaciales recurrentes y de otro tipo de enfermedades (encefalitis).
EPIDEMIOLOGIA
El ser humano es el nico reservorio natural conocido del virus herpes simplex, aunque
algunos animales de experimentacin pueden infectarse con facilidad.
La infeccin primaria del VHS-1 ocurre sobre todo durante la infancia, mientras que el tipo 2 se
presenta en la adolescencia y adultos jvenes activos sexualmente. Las tasas de infeccin son
inversamente proporcionales al estrato socioeconmico.
El principal mecanismo de transmisin es el contacto directo con las secreciones infectadas. El
VHS-1 se trasmite por saliva y el VHS-2 por va genital. Aunque los ttulos virales son ms altos
cuando existen lesiones activas, tambin es frecuente la liberacin viral en infectados
asintomticos. Por lo tanto, la transmisin viral puede efectuarse an en ausencia de lesiones
activas.
La persistencia de la infeccin y la recurrencia de las lesiones son un fenmeno frecuente tanto
para el VHS-1 como para el VHS-2 y por lo comn se producen por reactivacin endgena. Los
factores que la precipitan van desde la luz solar, el viento, traumatismos locales, fiebre,
menstruaciones y hasta estrs emocional.

DIAGNOSTICO

Cuadro Clnico. Los cuadros clnicos causados por este virus se suelen dividir en dos grupos:
el debido a la infeccin primaria y el correspondiente a la infeccin recurrente. En el primer
grupo se incluyen la gingivoestomatitis aguda, la vulvovaginitis aguda y la infeccin herptica
del ojo, que puede llegar a queratitis. Las recurrentes se circunscriben al "Herpes labialis",
queratitis, blefaritis y queratoconjuntivitis. Todos los cuadros son autolimitados, pero tanto las
formas primarias como las recurrentes, se pueden complicar. Una de estas complicaciones es
la Encefalitis herptica y el Eczema herpeticum.
Infeccin primaria. El primer contacto clnico de infeccin por virus del herpes simple suele ser
el ms grave. Los enfermos aquejan fiebre, malestar general, artralgias y por ltimo la
presencia de un grupo de vesculas sobre una base eritematosa, dolorosa, inflamada y
sensible. La gingivoestomatitis es la manifestacin ms comn, cuya gravedad vara desde la
erosin de pequeas reas a la ulceracin extensa de la boca, lengua y encas. La infeccin
puede ser bastante grave como para dificultar la ingesta de alimentos y lquidos (odinofagia).
La curacin tiene lugar en 7 a 14 das, a menos que las lesiones se sobreinfecten con
estafilocos o estreptococos.
Infeccin recurrente. Generalmente existe prurito, dolor o molestias focales que preceden la
aparicin de las vesculas. Las vesculas se rompen espontneamente despus de unos
cuantos das y sanan en una semana sin dejar secuelas.
LECTURAS RECOMENDADAS
1. Callen JP, Cooper Ma. Dermatologic emergences. Emerg Med. Clin North Am
3:641, 1985
2. Guzmn M. Herpes simple, varicela zoster. En: Medicina Interna. Segunda
Edicin. Editado por F Chalem, JE Escandn, J Campos, R Esguerra.
Fundacin Instituto de Reumatologa e Inmunologa. Editorial Presencia Ltda.
Santaf de Bogot, 1992
3. Guerra Flecha J, Lizarraga Bonelli S. Enfermedades de transmisin sexual:
herpes genital Trib Med 79:29, 1989
Jaramillo AC. Infecciones virales de la piel y sus anexos. En: Fundamentos de Medicina.
Enfermedades Infecciosas. Cuarta edicin. Corporacin para Investigaciones Biolgicas. CIB.
Medelln, 1989
58.- Femenino de 40 aos de edad G.3 P.1 C 2, la cual es diagnosticada por miomatosis
uterina de pequeos y medianos elementos sintomticos, sus antecedentes refieren cirugas
plvicas previas, el tratamiento de eleccin es:

a)
b)
c)
d)

Histerectoma total abdominal.


Progesterona.
Observacin
Anlogos de GnRH.

CUADRO CLINICO
La miomatosis uterina muestra manifestaciones clnicas en menos del 50%, de estas las ms
frecuentes son:
1. Hemorragia uterina anormal.
2. Dolor.

3.
4.
5.
6.

Distensin abdominal.
Compresin genitouterina.
Compresin gastrointestinal.
Compresin plvica.

10. DIAGNOSTICO
El diagnstico se realiza a travs de imagenologa:
1-Ecografa.
2-TAC
3-Rayos X
4-Histeroscopia.
11. TRATAMIENTO
La miomatosis uterina debe ser tratada cuando produzca cualquiera de las manifestaciones
clnicas anotadas, toda paciente que se programe para histerectoma debe tener previamente
legrado biopsia
Lo podemos dividir en Conservador o Radical.
1. CONSERVADOR:
Este tratamiento se puede instaurar en pacientes con deseo de preservar el tero.
Igualmente se puede subdividir:
-Expectante
-Quirrgico: miomectoma
-Medico
Tratamiento expectante: Esta indicado en pacientes, cuyos sntomas son leves y no deseen o
tengan alguna contraindicacin mdica para tratamiento quirrgico. En ellas se recomienda
controles clnicos y ecogrficos cada 6 meses a 1 ao.
Tratamiento mdico.
AINES
Anlogos GnRH:
Progestgenos:
Andrgenos.
Antiandrgenos
Tratamiento quirrgico:
MIOMECTOMIA:
1. Criterios del ACOG en pacientes infecundas.
Procedimientos:
Va endoscpica:
Laparoscopia: Miomas subserosos sesiles o pediculados < 5cm.
Histeroscopia: miomas submucosos

Va laparotoma

Aquellos miomas que se salgan de las caractersticas anteriores.


Indicaciones:

Hemorragia anormal.
Perdida reproductiva.
Infecundidad.
Dolor.

El tratamiento previo con anlogos est indicado cuando se desee disminuir el tamao del
mioma para prevenir sangrado quirrgico.
Contraindicaciones:

Embarazo.
Cncer endometrial.
Infecciones.
Dificultad tcnica.

2. Criterios del ACOG para Miomectoma en pacientes que desean conservar el tero.
Procedimiento:

Va endoscpica.
Va abdominal.
Va vaginal.

Indicacin:

a.

Presencia de uno o dos Leiomiomas asintomticos de tamao tal que se pueden palpar por
va abdominal y constituyen una preocupacin para la paciente.

b.

Pacientes ovulatorias con miomas como posible causa de hemorragia uterina excesiva,
demostrada por cualquiera de las siguientes circunstancias:
Hemorragia profusa: de duracin mayor de 8 das.
Anemia por prdida sangunea aguda o crnica.

2. RADICAL:
HISTERECTOMIA
Para pacientes post menopusicas, con paridad satisfecha o sin deseo de preservar el tero.
Criterios del ACOG para Histerectoma por miomas.
1. Presencia de 1, 2, o 3 Miomas asintomticos de tamao tal que son palpables por va
abdominal y preocupan a la paciente.
2. Hemorragia uterina excesiva.

Duracin mayor de 8 das.


Anemia por prdida sangunea aguda o crnica.

1. Molestias plvicas producidas por los miomas: signos compresivos.


Contraindicaciones:

1. Deseo de conservar la fecundidad.


2. Miomas asintomticos.
3. Contraindicacin mdica o dificultades tcnicas para la ciruga.
Guarnaccia M. and Rein M. Traditional Surgical Approaches to Uterine Fibroids
Abdominal. Myomectomy and Hysterectomy. Clinical Obstetrics and Gynecology
2001. 44.2. 385-400
- Milad. M and Sankpal R. Laparoscopic Approaches to Uterine Leiomyomas.
Clinical Obstetrics and Gynecology. 2001. 44-2. 401-411
- Carlson K. et al. Indications for Hysterectomy. N. Engl. J. Med. 1993. 328(12) 56-

59.- Se trata de femenino de 23 aos que cursa con amenorrea secundaria la cual presenta
menstruacin posterior a la administracin de progestgenos, lo ms probable en su perfil
hormonal es que presente:

a)
b)
c)
d)

Estrgenos bajos
Estrgenos normales
Progesterona elevada
Gonadotropinas altas

La presencia de estrgenos se puede establecer de dos maneras: con la prueba de desafo


con acetato de medroxiprogesterona (AMP) y el ndice de maduracin vaginal. Si hay
estrgenos, luego de la administracin de 10 mg de AMP diarios por 10 das debera
presentarse un sangrado por vagina. Y de esta forma corroborar los niveles de estrgenos
serian normales.
La ausencia de esta metrorragia sugiere una obstruccin o un
hipogonadismo. La otra alternativa es realizar el ndice de madurez vaginal que es un
procedimiento sencillo que puede obtenerse en el consultorio. Usando una esptula como las
de Papanicolaou el gineclogo extrae una muestra de la vagina y la extiende en un portaobjeto.
Este portaobjeto se procesa y se estudia en el microscopio. Un extendido maduro tendr
clulas epiteliales superficiales grandes y numerosas con un ncleo rodeado por una gran
cantidad de citoplasma. Por el contrario, una muestra no estrognica tendr un mayor nmero
de clulas parabasales y basales con ncleo grande rodeado por escaso citoplasma.

Speroff L, Glass RH, Kase NG, eds. Clinical Gynecologic Endocrinology and Infertility, 5th ed.
Baltimore: Williams & Wilkins; 1994:334-335.

60.- En el servicio de consulta externa recibe un frotis vaginal el cual reporta la presencia al
microscopio de clulas clave las cuales son propias de infeccin por:
a) Cndida albicans
b) Gardnerella
c) Tricomonas
d) Gonococos

La infeccin por gardnerella (bacteria) tiende a producir una secrecin blanca, gris o de color
amarillo turbio, con un olor ftido o a "pescado" que aumenta cuando la secrecin se vuelve

alcalina, como sucede despus del coito o de lavarse con jabn. Puede haber prurito o
irritacin vulvar, pero por lo general no son muy pronunciados. Al microscopio se aprecia las
famosas clulas clave. Realizando el diagnstico diferencial.

Clulas escamosas de capa intermedia alta, algunas de ellas con ncleos picnticos, y dos de
ellas con el citoplasma cubierto por formas cocceas que borran sus bordes, dndole el
aspecto de clula rebozada "clula clave". Gardnerella. .

Pernoll M. Enfermedades de trasmisin sexual. En: Manual de Obstetricia y Ginecologa.


Benson/Pernoll Editores.Editorial Interamericana S A. Mxico DF, 1994

61.- Se trata de paciente femenino de 29 aos de edad con tumor anexial de 6 cm lquido, dolor
abdominal, fiebre, leucorrea, con historia de cervicovaginitis de repeticin y dispareunia
crnica, ltima menstruacin hace una semana
El manejo ms adecuado es:
a)
b)
c)
d)

Histeroscopa
Colposcopa
Laparoscopa
Histerosalpingografa

CLASIFICACION LAPAROSCOPICA DE LA ENFERMEDAD PELVICA INFLAMATORIA


El stndar de oro para el diagnstico de EPI es la laparoscopia, ya que adems de visualizar
directamente los rganos plvicos, permite la toma de muestras para estudios bacteriolgicos.

Los criterios laparoscpicos para el diagnstico de EPI, se describen en la tabla No.3.


Tabla No. 3
CLASIFICACION LAPAROSCOPICA DE LA EPI
Leve

Eritema, edema, las trompas se mueven


libremente. No hay exudado purulento.

Moderada

Eritema, edema mas marcado, material


purulento evidente. No hay movimiento libre de
las trompas. La fimbria puede no ser evidente.

Severa

Presencia de pioslpinx y/o absceso

EPI
Diagnstico
Dolorabdominalbajo y/o doloralamovilizacin cervical
yanexial y:
Fiebre de38 Comayor
leucocitosis >10,500mm3,eritrosedimentacin yPCRelevada
Bacterias oleucocitos enliquido peritoneal
Masa anexial palpableopor USG
Laparoscopia
Sndrome deFitz HughCurtis

62.- Femenino de 34 aos que inicia tratamiento con sulfato de magnesio por presentar
eclampsia, se presentan signos de sobre dosificacin. El antdoto especfico que se debe de
utilizar es:
a) Gluconato clcico
b) Nitroprusiato.
c) Simpaticomimticos.
d) Carbonato sdico.

NIVEL DE PRIMER CONTACTO (ATENCION PRIMARIA)


Se debe instruir a todas las embarazadas que deben acudir inmediatamente a un centro de
salud en cualquiera de los siguientes casos:

edema que se desarrolla rpidamente (en pocos das)


cefalea severa y persistente
dolor en la regin abdominal superior
visin borrosa

Se debe realizar la medicin de la presin arterial y un anlisis de orina para la deteccin de


proteinuria a las mujeres que acudan a centros de salud presentando estos sntomas.
Convulsiones
Si se asiste a una mujer con eclampsia en un centro de atencin primaria,
1. deben mantenerse las vas respiratorias permeables;
2. se debe colocar a la mujer de costado (posicin decbito lateral izquierda) para evitar la
aspiracin del vmito u otras secreciones;
3. si es posible, se debe establecer una va intravenosa;
4. se debe administrar sulfato de magnesio.
Monitoreo de la administracin de sulfato de magnesio: Durante el tratamiento con sulfato de
magnesio, se recomienda realizar un control cada 4 horas, como mnimo, para detectar la
presencia de: Reflejo rotuliano, frecuencia respiratoria superior a 16 por minuto, volumen de
orina >100 ml en las 4 horas previas.
- Sobredosis de sulfato de magnesio: Todo centro de salud que utilice sulfato de magnesio
debe disponer de ampollas de gluconato de calcio (1 g) como antdoto para la sobredosis de
dicho frmaco.

Se sugiere medir la presin arterial y administrar antihipertensivos segn corresponda.


Convulsiones recurrentes: en caso de convulsiones recurrentes, se administran otros 2
a 4 g de sulfato de magnesio por va IV en el lapso de 5 minutos, tanto para el rgimen
IM como el IV; la dosis se determina en funcin del peso de la paciente.

El sulfato de magnesio es un frmaco usado en el control de las convulsiones eclmpticas,


para suprimir o controlar las contracciones uterinas sean estas espontneas o inducidas, y
como broncodilatador luego del uso de beta agonistas y agentes anticolinergicos. Tambin
tiene indicacin como terapia de reemplazo en la deficiencia de magnesio, como laxante para
reducir la absorcin de txicos del tracto gastrointestinal. El sulfato de magnesio est ganando
popularidad como tratamiento de inicio en el manejo de algunas arritmias, particularmente en
Torsades de Pointes, y en arritmias secundarias a sobredosis de antidepresivos tricclicos o
toxicidad digitlica. Esta tambin considerado clase Ila (probable beneficio) para la fibrilacin
ventricular refractaria y la taquicardia ventricular, luego de la administracin de dosis de
lidocaina y bretilio.

FARMACODINAMIA
El sulfato de magnesio tiene la capacidad de alterar la excitabilidad de la fibra miometrial,
afecta el acoplamiento excitacin contraccin y el proceso mismo de contraccin, inhibe la
entrada de calcio al sarcoplasma y reduce la frecuencia de los potenciales de accin. Inhibe
tambin la liberacin de acetilcolina. Por ser estas acciones comunes en las fibras musculares
se pueden ver afectadas tambin la musculatura voluntaria e incluso las fibras miocrdicas.(1)

Bibliografa:
Graves C. Frmacos que contraen o relajan el tero. En: Hardman J, Limbird L, Molinoff P,
Ruddon R, Goodman A, eds. Goodman & Gilman. Las Bases Farmacolgicas de la
Teraputica. 9 ed. Mxico DF: McGraw-Hill Interamericana; 1996. pp. 1012-3.

63.- Femenino que cursa con 36. 5 semanas de gestacin acude al servicio por referir malestar
general, fosfenos, nausea y vmito, aprecia moderada ictericia, usted sospecha de un
sndrome de HELLP. Las alteraciones de laboratorio que espera encontrar al confirmar el
diagnstico son:
a)
b)
c)
d)

Anemia Hemoltica, trombocitopenia, enzimas hepticas elevadas.


Trombocitosis, Enzimas heptica elevadas, Anemia hemoltica.
Anemia hemolitica, Trombocitosis, Fosfatasa Alcalina elevada.
Trombocitopenia, Leucopenia, Hipertensin Arterial.

DEFINICIN:
Es una complicacin de la preeclampsia en la cual adems de la Hipertensin Arterial y
proteinuria hay presencia de anemia hemoltica, enzimas hepticas elevadas y
recuento bajo de plaquetas

EPIDEMIOLOGIA:
Se presenta en un 4 a 10% de las preeclmpticas, diagnosticndose anteparto en un 70%
de los casos preferentemente antes de las 37 semanas, mientras que el 30% de los
casos restantes enferma en los primeros 7 das del puerperio, sobre todo en las 48 h
iniciales.
La proteinuria e hipertensin pueden estar ausentes en un 15 al 20% de los casos.
Incidencia mayor en multigestantes y en edades avanzadas.
Ocurre ms frecuentemente cuando se demora la salida del feto y cuando se presenta
desprendimiento de la placenta
Mortalidad materna del 24% y mortalidad perinatal del 30-40%.

CLASIFICACION:
Sndrome de HELLP. Clasificacin de Mississipi.
CLASE
Plaquetopenia
LDH
1
Severa <50000
>600 IU/L
2
Moderada
>600 IU/L
>50000

AST-ALT
>70 IU/L
>70 IU/L

3
PE severa
Eclampsia (sin
HELLP)

<100000
Ligera >100000
<150000
>150000

>600 IU/L
<400 IU/L

>40 IU/L
<70 IU/L
<40IU/L

MANIFESTACIONES CLINICAS:
Malestar general, fatiga y molestias inespecficas 90%
Cefalea 70%
Epigastralgia 64%
Vmito 22%
Fosfenos 15%
Visin Borrosa 11%
Acfenos 3%
Ictericia
Anemia no explicada
Oliguria

Si se aade una HEMORRAGIA HEPTICA, el paciente puede quejarse de dolor en el


HOMBRO DERECHO y EL CUELLO, adems de las molestias abdominales.
Equimosis en los sitios de punciones venosas, petequias en los sitios de presin del
brazo, pero pueden tener pruebas de Rumpel Leed negativas.
En casos severos se pude presentar ascitis como causa de hipertensin portal.
DIAGNSTICO:
El diagnstico clnico del sndrome de HELLP se plantea en gestantes o purperas con
preeclampsia severa-eclampsia, excepto en el 15-20%, en las cuales esta asociacin no puede
ser demostrada, en tanto se cumplan los criterios de Sibai:

MANIFESTACIONES CLNICAS:
Malestar general, fatiga y molestias inespecficas 90%
Cefalea 70%
Epigastralgia 64%
Vmito 22%
Fosfenos 15%

Visin Borrosa 11%


Acfenos 3%
Ictericia
Anemia no explicada
Oliguria

Si se aade una HEMORRAGIA HEPTICA, el paciente puede quejarse de dolor en el


HOMBRO DERECHO y EL CUELLO, adems de las molestias abdominales.
Equimosis en los sitios de punciones venosas, petequias en los sitios de presin del
brazo, pero pueden tener pruebas de Rumpel Leed negativas.
En casos severos se pude presentar ascitis como causa de hipertensin portal.
DIAGNSTICO:
El diagnstico clnico del sndrome de HELLP se plantea en gestantes o purperas con
preeclampsia severa-eclampsia, excepto en el 15-20%, en las cuales esta asociacin no puede
ser demostrada, en tanto se cumplan los criterios de Sibai:
HEMOLISIS
Frotis perifrico anormal (eritrocitos fragmentados)
Hematocrito (>24%)
Bilirrubina indirecta (>1.2mg/dL)
Deshidrogenasa lctica (>218 UI/L)
ENZIMAS HEPTICAS ELEVADAS
LDH >218UI/L
AST >30UI/L
ALT >37UI/L
PLAQUETAS BAJAS
<100.000/mm3

BIBLIOGRAFIA
Sibai baha, El sndrome HELLP. Universidad de Valencia , revista quincenal de Obstetricia
clnica y ginecologa, Octubre 2003.
V. Cararach, Sndrome de HELLP y Repercusiones maternas. X curso intensivo de formacin
continuada materno fetal. Enero de 2003.

Toirac, Abelardo. Sndrome de Weistein HELLP Hospital Ginecoobstetrico Tamara Bunke.


Junio 2002
De la Fuente, David. Sndrome HELLP. Medicina Universitria 2003; 5 (19): 101 -9
Andrea G. Witlin, DO, Baha M. Sibai, MD. Diagnosis and Management of women with
Hemolysis Elevate Liver Enzymes, and Pletelet Count (HELLP) syndrome. Hospital Physician.
Febrero 1999.
CIFUENTES B, Rodrigo. Ginecologa y obstetricia basadas en las evidencias. Bogot:
Distribuna, 2006. Sexta edicin. 447 - 283 p.

64.- Se trata de femenino de 28 aos de edad, acude por presentar retraso menstrual de 7
semanas, asintomtica, con antecedentes de enfermedad plvica inflamatoria tratada
anteriormente, sin evidencia mediante ecografa de tero ocupado. Al determinar la B-HCG, se
obtiene cifra de 2,500UI/l. Ante estos datos, en primer lugar habr que pensar en:
a)
b)
c)
d)

Gestacin de evolucin normal correspondiente a amenorrea.


Embarazo ectpica asintomtico.
Aborto precoz completo con expulsin total de restos ovulares intrauterinos.
Embarazo molar de inicio, sin signos ecogrficos intrauterinos.

FACTORES DE RIESGO
Anomalas tubarias

Enfermedad plvica inflamatoria 30-50%


Adherencias (endometriosis)
Antecedente de salpingoclasia
Alteraciones anatmicas

EMBARAZO ECTPICO
< 6500mUI

USG normal

Cuantificacin seriada
de HGC

Descenso

Estabilizacin o
incremento

Dilatacin y legrado

Laparoscopia Dx

64

Repetir USG
02/07/2013

Diagnstico:

Aumento

HGC > 1500 U/L


Sin evidencia de saco
Lquido libre en fondo de
saco
Sensibilidad 63%
Especificidad 100%

Ankum WM., Mol Bw.,Van der Veen F.,Bossuyt PM.


Risk factors for ectopic pregnancy:a meta-analysis. Fertil Steril 1996:65:1093-9.

65.- Femenino de 26 aos G-3, P-1, A-1 con 39 SDG por FUR. Reporta contracciones
uterinas que han sido regulares las ltimas tres horas. Al examen encuentras que las
contracciones son cada tres minutos y duran 50 segundos y son firmes a la palpacin. Tuvo
ruptura de membranas hace una hora y lo demuestras con papel de nitrazina. El examen digital
cervical demuestra una dilatacin de 5 cm, con borramiento del 100% y presentacin en vrtex
en estacin 0. De los siguientes criterios el ms preciso para decir que se encuentra en la
fase activa del trabajo de parto es :
a)
b)
c)
d)

Borramiento cervical ms de 90%


Duracin de las contracciones de ms de 30 seg
Dilatacin cervical mayor de tres centmetros
Ruptura de membranas

FASES DEL TRABAJO DE PARTO


El trabajo de parto se divide en tres fases:
Fase 1 latente
Es llamado as al periodo que sirve para la preparacin uterina del parto, ocurre al final del
embarazo y va hasta el inicio de las contracciones del trabajo de parto. Los aspectos a
destacar en este lapso es el reblandecimiento cervical, el aumento importante en el nmero de
receptores para oxitocina a nivel de las clulas endometriales, un aumento sustancial en los
puentes de unin y el nmero de conexinas a nivel miometrial y por consiguiente una mayor
sensibilidad a los agentes uterotnicos.
Fase 2 activa
Es el lapso que representa el trabajo de parto activo, y se acepta que se inicie cuando existen 3
cm de dilatacin y las contracciones uterinas son aptas para producir un avance en el trabajo
de parto; se divide en tres periodos:
Primer periodo. Se inicia cuando las contracciones uterinas alcanzan la frecuencia, intensidad y
duracin suficientes para causar borramiento y dilatacin del cuello uterino, y finaliza cuando
ste se encuentra en completa dilatacin.
El lapso de tiempo que dura es variable, pero se acepta como normal hasta diez horas en
primigrvidas y ocho horas en multigrvidas; pero independientemente de esto, se debe
considerar como adecuado si el borramiento y la dilatacin cervical son progresivos e
ininterrumpidos.
Segundo periodo. Se inicia con una dilatacin cervical completa y termina con la expulsin del
feto; tiene una duracin variable, pero se acepta como normal una hora en pacientes
primparas y 30 minutos en multparas; y tiene como caracterstica que debe de ser progresivo
e ininterrumpido.
Tercer periodo. Este comienza inmediatamente finalizada la expulsin fetal y termina con la
expulsin total de la placenta y las membranas corioamniticas; a este periodo se le conoce
tambin como de alumbramiento y es el ms corto de los periodos del parto; como norma
general se acepta que no debe de extenderse ms all de 10 minutos.
Existen algunos autores que incluyen un cuarto periodo dentro del trabajo de parto, el cual

abarca aproximadamente la hora posterior al alumbramiento, y comprende el lapso de tiempo


cuando ocurre la contraccin y retraccin de las fibras miometriales, as como la trombosis de
los vasos adyacentes, lo cual es un efectivo control de la hemorragia del sitio de implantacin
de la placenta.
Fase 3
Este periodo es el que representa el regreso de la mujer a su estado previo al embarazo, y se
caracteriza por la involucin uterina, la eyeccin lctea y por ltimo la restauracin de la
fertilidad; existen estudios que involucran en esta fase a la endotelina-1 y a la oxitocina como
substancias responsables de estos cambios postparto.

PROGRAMA DE ACTUALIZACION CONTINUA PARA GINECOLOGA Y OBSTETRICIA


PAC GO-1 Libro 3 Obstetricia 2005

66.- Femenino de 19 aos, gesta 1, tuvo un parto con un producto nico masculino de 3,600 g.
la calificacin de Apgar en el RN fue de 9 al primer minuto y 9 a los 5 minutos. La revisin de
sus registros de trabajo de parto mostr que tuvo ruptura de membranas 7 horas antes del
parto. 40 horas despus del parto la paciente presenta: temperatura 38.2 C, FC 105 x, TA
110/70, FR 16x; E.F. dolor leve a la palpacin del tero, las mamas se encuentran sin eritema,
ni dolor a la palpacin, no dolor a la compresin de las pantorrillas. De los siguientes pasos el
ms indicado antes de iniciar terapia con antimicrobiana ES:

a) Hemocultivo
b) Cultivo de secrecin vaginal
c) Examen general de orina y cultivo
d) Espirometra por incentivo
.

El vaciamiento incompleto ocasiona orina residual, distensin vesical excesiva y estasis,


adems del cateterismo intermitente con sonda vesical durante el trabajo de parto. Por lo
tanto, la vejiga en el puerperio est predispuesta a infecciones. El dolor leve a la palpacin del
tero puede ser normal en el puerperio y no se debe suponer de inmediato endometritis
puerperal. Cuando se sospecha endometritis, los cultivos de secrecin vaginal tienen poca
utilidad porque se encuentran los mimos microorganismos que en mujeres purperas sanas.
Los hemocultivos son apropiados para la valoracin diagnstica de la fiebre puerperal, pero no
son el paso inicial. La espirometra por incentivo se utiliza en el posoperatorio de inmediato
para fomentar la expansin pulmonar y disminuir las atelectasias. El legrado uterino se utiliza
para tratar la hemorragia
Morgan M, Siddighi S. Ginecologa y obstetricia, National Medical Series. 5 edicin. Mc Graw
Hill. Pp. 29.

67.- Femenino de 22 aos de edad, acude al servicio de urgencias por referir malestar general,
fosfenos, nusea y vmito. Antecedentes: cursa con embarazo de 36.5 SDG, G2, A1, C0.
Exploracin fsica: TA 185/110 mmHg, FC 120 lpm, FR 35 x, T37.9, alerta, inquieta, aprecia
ictericia, cardiopulmonar sin compromiso, abdomen globoso a expensas de tero grvido, se
detecta PUVI, longitudinal, ceflico, dorso a la derecha, FCF 142 x, al tacto genital crvix,
central, formado, cerrado, extraccin de guantes sin evidencia de prdidas genitales,
extremidades inferiores edema +++.
El dato de laboratorio que apoya el diagnstico en esta paciente es:
a)
b)
c)
d)

Hematuria.
Coluria.
Piuria.
Proteinuria.

Referencia:
La preeclampsia se define por aumento de la presin arterial y la presencia de proteinuria
durante el embarazo.
3) Leve: las pacientes suelen tener unas cuantas manifestaciones, y su presin arterial
diastlica es menor de 110 mmHg. En ocasiones hay edema. La cifra de plaquetas es
mayor de 100 000/l.
4) Grave: los sntomas son ms notorios y persistentes. La presin arterial casi siempre
es con niveles mayores de 160/110 mmHg. Puede haber trombocitopenia que avance
hasta coagulacin intravascular diseminada.
Se requiere hospitalizacin para las mujeres con preeclampsia; debe obtenerse una biometra
hemtica completa con cifra de plaquetas y determinacin de electrolitos, que adems incluya
enzimas hepticas. Se obtiene una muestra de orina de 24 horas para determinar la
depuracin de creatinina y protenas totales al ingreso hospitalario.
Se debe controlar la hipertensin arterial, para evitar sufrimiento fetal, as como empeoramiento
de la paciente.
Bibliografa:
3. McPhee S, Papadakis M, et. al. Diagnstico Clnico y Tratamiento 2010. Lange, McGraw
Hill,
49 edicin, Mxico, 2010.
4. Sibai BM, Diagnosis, prevention, and management of eclampsia. Obstet Gynecol. 2005.
Feb; 105;: 402 410.

68.- Femenino de 37 aos gestante, acude a ultrasonido de control. Reportando: Placenta


marginal, embarazo de 37 SDG. Antecedentes: G- 5 P- 4.
La complicacin esperada en esta paciente es:
a)
b)
c)
d)

Hemorragia materna.
Desprendimiento de placenta normoinserta.
Parto pretrmino.
Restriccin en el crecimiento intrauterino.

Placenta previa: La placenta en lugar de estar implantada en el cuerpo del tero por fuera del
orificio cervical interno, se localiza por encima del orificio interno o cerca de l. En la placenta
marginal el borde la placenta est en el margen del orificio. La multiparidad y la edad avanzada
parecen favorecer la placenta previa. El hecho ms carcterstico de la placenta previa es la
hemorragia no dolorosa, la cual no suele aparecer hasta casi del final de segundo trimestre o
ms tarde. Complicaciones maternas: Hemorragia y muerte.

Pritchard, Williams Obstetricia, 3Edicin, Salvat editores, pag. 395-399

69.- Se trata de femenino de 31 aos nuligesta pero con actividad sexual regular, sin mtodo
de planificacin familiar, con ciclos regulares, sin leucorrea, refiere dispareunia profunda,
sangrado intermestrual y dismenorrea secundaria ocacionalmente presenta urgencia urinaria.
Establecido su diagnstico el tratamiento de eleccin en esta patologa es:
a)
b)
c)
d)

Doxicilclina
Ceftriaxona
Leuprolide
Tibolona

Endometriosis
Tratamiento
Anlogos de GnRH por 6 meses (Leuprolide, Nafarelina,
Triptorelina,Goserelina.

DisminuyelasecrecindeFSHyLH.
Danazol
ACOpor 9meses
Laparoscopia
RadicalconHTA+SOB
Cncer?
Clulas claras
Endometroide deovario

70.- Recibe usted los resultados histopatolgicos de una paciente de 24 aos de edad que
acudi a revisin rutinaria, los resultados reportan imagen histolgica de coilocitos lo cual
sugiere infeccin por:
a)
b)
c)
d)

Herpes virus tipo 2


Citomegalovirus
Virus del papiloma humano
Vaginosis bacteriana.

El coilocito es un tipo de clula hallada en lesiones precancerosas cervicales. Tambin es


comn apreciarla microscpicamente en lesiones reaccionales en la mucosa oral, debido a su
similitud con la mucosa vaginal, en enfermedades como Papilomas, o en Condiloma
acuminado.
El coilocito es la manifestacin clsica de la infeccin por VPH en la clula. Fue descrito por
primera vez por Koss y Durfee en 1956. Esta clula tambin ha sido llamada clula en baln.
El coilocito es una clula epitelial escamosa, ms comnmente superficial e intermedia, aunque
tambin puede verse en clulas parabasales y metaplsicas. Esta clula presenta cambios
tpicos tanto en su ncleo como en su citoplasma, pierde los bordes angulados usuales de la
clula escamosa superficial y su forma tiende a ser redondeada y ovoide. El citoplasma
muestra una condensacin perifrica que le da un aspecto en asa de alambre, es opaco,
denso y de aspecto creo, anfoflico, acidoflico o de color rojo/naranja brillante. Adems se
observa una gran cavidad o halo con un margen muy bien definido, de forma oval o
ligeramente festoneado. El ncleo de la clula se localiza de manera excntrica, lo que lo
convierte en un halo paranuclear, no perinuclear. Ocasionalmente puede encontrarse material
fagocitado dentro del espacio coiloctico.

Lesin Intraepitelial de Bajo Grado. Alteraciones Celulares compatibles con Infeccin


(Coilocitos).
Schlecht, N.F., Kulaga, S., Robitaille, J., Ferreira, S., Santos, M., Miyamura, R.A., DuarteFranco, E., Rohan, T.E., Ferenczy, A., Villa, L.L., & Franco, E.L. (2002) Persistent Human
Papillomavirus Infection as a Predictor of Cervical Intraepithelial Neoplasia. JAMA, 286,
3106-3114

71.- Femenino de 32 aos segunda gesta a trmino sin anormalidades en el transcurso de


ste. Inicia trabajo de parto de forma espontnea, con evolucin normal hasta que se rompe la
bolsa, con una dilatacin de 4 cm. A partir de entonces, comienza con hemorragia de sangre
roja, en moderada cantidad y aparecen signos de sufrimiento fetal agudo. El estado general de
la mujer es bueno y la dinmica uterina es normal. Este cuadro corresponde a:
a) Abruptio placentae.
b) Rotura uterina.
c) Placenta previa central
d) Rotura de vasa previa.

La vasa previa es una condicin de alto riesgo obsttrico en la cual vasos fetales o placentarios
cruzan el segmento uterino por debajo de presentacin.
Estos vasos estn desprotegidos de gelatina de Wharton o tejido placentario, lo que los hace
altamente vulnerables y susceptibles de ruptura o laceraciones en cualquier perodo del
embarazo, principalmente en el momento del parto . Tambin es frecuente la compresin de
estos vasos, especialmente durante el tercer trimestre de la gestacin, lo que puede
condicionar asfixia y muerte fetal.
Esta condicin ocurre como resultado de que vasos velamentosos cruzan por el segmento
uterino debido a una insercin velamentosa del cordn, situacin en la cual el cordn umbilical
se inserta en las membranas ovulares en vez del tejido placentario (vasa previa tipo I), o por el
cruce de vasos fetales entre uno o ms lbulos accesorios de la placenta (vasa previa tipo II)
(Figura 1).

Figura 1. A, Vasa previa tipo I, debida a insercin velamentosa.


B, Vasa previa tipo II, debida a cotiledones aberrrantes. Reproducido de Daly-Jones y cols.
Ultrasound 2008.
Vasa previa se puede presentar si existe alguna (o ninguna) de las siguientes condiciones:
placenta baja (que puede ser causa de abortos previos seguidos por legrado o por operaciones
uterinas, que provocan cicatrices en el tero), placenta bilobada o de lbulo succensuriado,
embarazos resultado de fertilizacin in vitro, o embarazos mltiples (5-6). El sangrado por vasa
previa no es doloroso. Otros sangrados por complicaciones o por nacimiento no
necesariamente son sin dolor.

Referencias:
1. Oyalese Y, Smulian JC. Placenta previa, placenta acreta, and vasa previa. Obstet Gynecol
2006; 107:
927-941.
2. Oyalesse KO, Turner M, Less C, Campbell S. Vasa previa: an avoidable obstetric tragedy.
Obstet Gynecol Surv 1999; 54: 138-145.
3. Seplveda W, Sebire NJ, Harris R, Nyberg DA. The placenta, umbilical cord, and
membranas. In Diagnostic Imaging of Fetal Anomalies, Nyberg DA, MaGahan JP, Pretorius DH,
Pilu G (eds). Philadelphia, PA: Lippicont Williams & Wilkins 2003, 85-132.

4. Daly-Jones E, John A, Leahy A, McKenna C, Sepulveda W. Vasa praevia; a preventable


tragedy. Ultrasound 2006; 16: 8-14.
5. Derbala Y, Grochal F, Jeanty P. Vasa previa. J Prenat Med 2007; 1: 2-13.
6. Fung TY, Lau TK. Poor perinatal outcome associated with vasa previa. It is preventable? A
report of three cases and review of the literature. Ultrasound Obstet Gynecol 1998; 12: 430-433.
7. Robert JA, Sepulveda W. Fetal exsanguination from ruptured vasa previa: still a catastrophic
event in modern obstetrics. J Obstet Gynaecol 2003; 23: 574.
8. Cordero DR, Helfgott AW, Landy HJ, et al. A non-hemorrhagic manifestation of vasa previa: a
clinicopathologic case report. Obstet Gynecol 1993; 82: 698-700.
9. Schachter M, Tovbin Y, Arieli S, et al. In vitro fertilization as a risk factor for vasa previa. Fertil
Steril 2002; 78: 642-643.

72.- Se trata de femenino de 24 aos de edad que acude a consulta, cursando su 14 semana
de gestacin, refiere tenesmo vesical, disuria y escalofro. De acuerdo a su estado actual el
tratamiento de eleccin que recomienda a la paciente es:
a)
b)
c)
d)

Ciprofloxacino
Tetraciclina
Ampicilina
Metronidazol

El Metronidazol no ha mostrado efectos txicos en humanos, pero es teratognico en modelo


animal. Las tetraciclinas ocasionan coloracin anormal de los dientes, hepatotoxicidad y
alteracin en el desarrollo de huesos. Las sulfas podran tener un efecto deletreo en el primer
trimestre dada su actividad como antimetabolitos, y en los ltimos meses pueden favorecer
kernicterus en el recin nacido si es que tiene alteraciones metablicas que favorezcan anemia
hemoltica. Las quinolonas se han asociado a malformaciones seas en modelos animales, y
se recomienda evitarlas si existen mejores opciones.
Kasper DL, Braunwald E, Fauci AS, Hauser SL, Longo DL, Jameson JL. Harrisons
Principles of Internal Medicine. McGraw Hill. 16 Ed. 789-806 pp.

73.- Se trata de femenino de 33 aos con antecedentes patolgicos de hipertensin arterial


crnica bien controlada tratada con IECAS , actualmente cursa con 7 semanas de gestacin ,
signos vitales dentro del parmetro normal y exmenes de laboratorio sin alteraciones, se
refiere asintomtica. La conducta ms adecuada a seguir es:

a) b) Mantener el tratamiento y asociar alfametildopa para disminuir los riesgos fetales de


los IECAs
b) Mantener el tratamiento y asociar hidralacina para disminuir los riesgos maternos de los
IECAs.
c) Suspender los IECAs dado el riesgo que presentan para el feto.
d) Mantener el tratamiento dado el buen control tensional.

El uso de IECA y ARAII durante el segundo y tercer trimestre de embarazo est


contraindicado, debido a que estos medicamentos inducen toxicidad fetal (descenso de la

funcin renal, oligohidramnios, retraso en la osificacin del crneo) y toxicidad neonatal (insufi
ciencia renal, hipotensin, hiperpotasemia).
En cuanto a su uso durante el primer trimestre de embarazo, un estudio publicado en el ao
20061 mostraba un incremento de la incidencia de malformaciones congnitas, en particular
malformaciones cardiacas, en nios nacidos de madres expuestas a IECA durante el primer
trimestre de embarazo en comparacin con las mujeres que no recibieron tratamiento
antihipertensivo o que recibieron tratamiento con otros medicamentos antihipertensivos.
Estudios posteriores realizados no han confirmado a da de hoy los resultados de este estudio.
En lo referente a los ARAII, no se dispone de estudios epidemiolgicos analticos apropiados,
por lo que no se puede descartar que exista el mismo riesgo que para los IECA.
A pesar de estas incertidumbres, el Comit de Medicamentos de Uso Humano (CHMP) de la
Agencia
Europea de Medicamentos (EMEA) ha recomendado prudencialmente evitar el uso de IECA y
ARAII durante el primer trimestre del embarazo.
Cooper WO et al. Major congenital malformations after fi rst-trimester exposure to ACE
inhibitors. N Engl J Med 2006; 354 (23): 243- 51.
(ref.: 2008/10, junio

74.- Se trata de femenino de 27 aos, G1 en trabajo de parto prematuro con embarazo de 30


semanas de gestacin. A pesar del uso de agentes tocolticos, estos no han dado resultado. Se
puede inducir la maduracin pulmonar del producto por medio de:

a) Betametasona
b) Sulfato de magnesio
c) Hidroxiprogesterona
d) Clorprocana

La utilizacin de betametasona como inductor de madurez pulmonar fetal (IMPF) disminuye la


morbilidad neonatal relacionada con prematurez pero su efecto diabetgeno materno ha sido
poco estudiado.
La revisin Cochrane de un ciclo nico de corticosteroides se actualiz en 2006. En esta
actualizacin se incluyeron 21 estudios con un total de 3885 mujeres y 4269 lactantes.
En la revisin se descubri que la administracin de determinados corticosteroides a mujeres
con riesgo de tener un parto prematuro reduce considerable los riesgos de complicaciones
relacionadas con la prematurez como muerte fetal y neonatal combinada, sndrome de
dificultad respiratoria, hemorragia cerebroventricular, enterocolitis necrotizante, infecciones
sistmicas y retraso en el desarrollo durante la niez. Los beneficios estaban presentes cuando
el tratamiento se iniciaba entre las 26 y las 35 semanas de gestacin y en los nios que nacan
entre 1 y 7 das despus de haber comenzado el tratamiento; tambin se observaron beneficios
en los subgrupos de mujeres con rotura prematura de membranas y trastornos hipertensivos.
La muerte fetal y neonatal combinada se redujo incluso en neonatos que nacieron a menos de
las 24 horas de haber administrado la primera dosis.
No se demostraron beneficios cuando el tratamiento comenz antes de las 26 semanas de
gestacin, tampoco se observaron beneficios en los recin nacidos antes de las 26 semanas

de gestacin ni en los que nacieron despus de 7 das o ms de la administracin del


tratamiento.
En el caso de los neonatos que nacieron despus de las 36 semanas hubo una tendencia a
aumentar la muerte fetal y neonatal combinada.
Se observ una reduccin en el peso al nacer en los neonatos que nacieron entre los das 1 y
7, al igual que en los que nacieron ms de 7 das despus del primer tratamiento.
Un estudio que reclut mujeres con preeclampsia severa sugiri que las mujeres tratadas
tenan un mayor riesgo de sufrir diabetes gestacional.
La evidencia epidemiolgica y en animales sugiere que pueden haber efectos adversos a largo
plazo por la exposicin prenatal a los corticosteroides, entre ellos la alteracin de la tolerancia a
la glucosa y la hipertensin. Los estudios en animales tambin han sugerido que afecta el
crecimiento del cerebro.
1. National Institute of Health (NIH). Consensus Conference; Effect of corticos-teroide for
fetal maturation on perinatal outcomes. JAMA 1994;(12):1-19.
2. White A, Marcucci G, Andrews E, Edwards K. Antenatal steroids and neonatal
outcomes in controlled clinical trials of surfactant replacement. Am J Obstet Gynecol
1995; (173):286-90.
3. Klauss MH, Fanaroff AA, Martin RJ. Problemas respiratorios. En: Asistencia del recin
nacido de alto riesgo. 2 ed. La Habana: Editorial Cientfico-Tcnica, 1981:194.
4. Avery M, Frank N, Gribetz I. The inflationary force produced by pulmonary vascular
distention in excised lungs. The possible relation of this force to that needed to inflatc
the lungs at birth. J Clin Invest 1959;38:456.
5. Chu J, Clements J, Cotton E. Neonatal pulmonary ischemia. Pediatrics 1965;40:733.
6. Liggins GC, Howle RN. A controlled trial of antepartum glucocorticoid treatment for
prevention of respiratory distress syndrome in premature infants. Pediatrics 1972;50:
515-25.
7. Wright LL, Verter J, Younes N. Antenatal corticosteroids administration and neonatal
outcome in infants 501 to 1500 g. Am J Obstet Gynecol 1995; (173):263.

75.- Ante una paciente de 35 aos que acude a consulta con antecedentes de G4 C2 A1,
refiere que ha presentado durante el primero y segundo trimestres de su embarazo manchado
con frecuencia intermitente ,a las 34 SDG inicia con hemorragia abundante, repentina e
indolora, su principal sospecha es:
a)
b)
c)
d)

Coriocarcinoma
Ruptura uterina
Placenta previa
Desprendimiento grave de placenta normoinserta

PLACENTA PREVIA
DEFINICIN:
Es cuando la placenta se implanta sobre o muy cerca del orificio cervical interno y una parte de
la placenta precede a la parte fetal que se presenta.
INCIDENCIA:
Esta es difcil determinar ya que muchos casos pasan desapercibidos, sobre todo cuando
ocurren los abortos en embarazos tempranos.
La prevalencia vara de 1 en 100 a 1 en 850 nacidos vivos, pero solo el 20% total.
Etiologa:
Edad avanzada, multparas, paciente con cesreas previas, paciente con aborto de repeticin,
esto debido a las gestaciones previas.

TIPOS:
Insercin baja.- Es cuando el borde placentario se encuentra en el segmento inferior a menos
de 6 cm del orifico cervical interno.
Marginal.- Es cuando el borde placentario alcanza los mrgenes del orificio cervical interno.
Parcial.- Es esta la placenta cubre parcialmente el orificio cervical interno.
Total.- La placenta cubre la totalidad del orificio cervical interno an con dilatacin cervical
avanzada.
DIAGNSTICO:
La caracterstica es el STV de aparicin brusca en forma indolora en el segundo o tercer
trimestre. Frecuentemente hay ausencia de dolor a actividad uterina que son parmetros para
hacer el diagnstico.
La mayor incidencia de sangrado aparece a las 33-34 sdg.
ESTUDIOS DE GABINETE:
El estudio ms utilizado es la ultrasonografa obsttrica.
TRATAMIENTO:
El manejo va a depender de factores como son:
Edad gestacional, magnitud del sangrado, si hay trabajo de parto, variedad de placenta previa y
complicaciones materna.
BIBLIOGRAFA:
1.- Waxler P, Gottesfeld KR. Early diagnosis of placenta previa. Obstet Gynecol 1979;54:23132.
2.- Cabrero-Roura L. Riesgo elevado obsttrico. Ed. Masson 1996; pp; 109-118.
3.- Patrick J, Placenta Previa, Clinical Obst and Gynecology 1990;33(3): 414-421.
4.- Chapman M, Furtenes ET, Significance of ultrasound in location of placenta in early
pregnancy Br J Obst Gynecol 197;86: 846.
57

76.- Mujer de 25 aos de edad, es atendida en consulta por secrecin lctea bilateral hace 4
meses y menstruaciones cada 21 das, en escasa cantidad desde hace 6 meses.
Antecedentes: hace 6 meses padece gastritis tratada con cimetidina durante 2 meses y
posteriormente ha tomado el medicamento de forma irregular. a.g.o.: menarca 12 aos, ritmo
30/7, nbil. e.f.: talla 1.63, peso 54 kg. Al efectuar compresin en glndulas mamarias se
produce salida de secrecin lctea bilateral, resto normal.
El tratamiento de primera eleccin para esta paciente es:
a)
b)
c)
d)

Metimazol
Bromocriptina.
Levotiroxina S.
omeprazol.

La hiperprolactinemia (hiperprl) es un trastorno frecuente, ms en la mujer que en el hombre, y


puede ser la manifestacin de un adenoma hipofisario (prolactinoma). La medicin de
prolactina (prl) en la sangre es ahora un procedimiento de rutina en la deteccin de trastornos
de la menstruacin y galactorrea que se resuelven efectivamente con el uso de frmacos con
actividad dopaminrgica (bromocriptina y cabergolina). La causa ms frecuente de hiperprl es
por el uso de frmacos que inhiben la accin de dopamina, pero se debe descartar inicialmente
hipotiroidismo y un prolactinoma mediante resonancia magntica.
Los prolactinomas son fcilmente controlados con dopaminrgicos y excepcionalmente se
requiere ciruga. En caso de infertilidad por exceso de prl existe una respuestafavorable con
bromocriptina o cabergolina, aunque en la actualidad se prefiere la ltima por su efectividad y
menos efectos indeseables.

Hiperprolactinemia. Gua teraputica y diagnstica


Arturo Zrate*
* Unidad de investigacin de endocrinologa, diabetes y metabolismo,
Centro Mdico Nacional IMSS y Hospital ngeles Mxico.
77.- A woman presents with painless vaginal bleeding at 37 weeks gestation. The fetal heart
rate is stable in the 150 pbm. Which of the following is indicated?
a)
b)
c)
d)

Ultrasound examination
Nonstress test
Induction of labor
Digital examination of the cervix

Diagnstico por USG.

Sangrados 2da Mitad Embarazo


Diagnstico diferencial (1)
DPPNI
DOLOR
HIPOGASTRICO

PLACENTA PREVIA

PRESENTE

AUSENTE

AUMENTADO

RELAJADO

COMPROMISO
FETAL

FRECUENTE

INFRECUENTE

U.S.G.

DIAGNOSTICA

TONO UTERINO

DIAGNOSTICA

78.- Se trata de paciente femenino de 55 aos de edad que presenta prolapso uterino e
incontinencia urinaria de esfuerzo el procedimiento de eleccin en esta paciente es:
a) Histerectomia total abdominal.
b) Histerectoma vaginal reconstructiva
c) Colpoperineoplasta.
d) Uterosuspensin.
El prolapso genital y su tratamiento ha sido siempre un importante captulo de la ginecologa.
No existe consenso clnico del concepto que define al prolapso genital como patolgico. Cierto
grado de descenso y relajacin de la pared vaginal es considerado normal en la gran mayora
de las mujeres, siendo ms frecuente en mujeres de mayor edad. En la poblacin general solo
un 3% presenta prolapso genital severo, entendiendo como tal al de III y IV grado. Es

probable que alrededor de 3 a 6% de la poblacin femenina desarrolle un prolapso de esas


caractersticas en algn momento de su vida
La intervencin quirrgica por excelencia para el tratamiento quirrgico del prolapso uterino es
la histerectoma vaginal. Esta intervencin se clasifica como "limpia-contaminada

INDICACIONES PARA LA HISTERECTOMA VAGINAL


Prolapso uterino
Hemorragia uterina disfuncional
Carcinoma crvico-uterino "in situ"
Miomatosis uterina poco voluminosa
Hiperplasia endometrial
Piometra

VENTAJAS DE LA HISTERECTOMA VAGINAL


No deja cicatriz abdominal
Mnimo trauma abdominal
Escasa hemorragia transoperatoria
Mnima manipulacin intestinal
Menos dolor postoperatorio
CUADRO
CONTRAINDICACIONES PARA LA HISTERECTOMA VAGINAL
Impericia
tero muy voluminoso
Tumores ovricos
Endometriosis
Enfermedad plvica inflamatoria
Ciruga previa en tero, trompas y ovarios
Cncer de endometrio
Histerectoma obsttrica

79.- Se trata de femenino de 26 aos de edad, que acude a revisin por 8 semanas de
amenorrea, asintomtica. En la ecografa vaginal se observa una tumoracin anexial izquierda,
sin evidencia de embrin. En el tero no se visualiza saco gestacional. La actitud ms correcta
en ste caso es:
a)
b)
c)
d)

Ingreso para legrado.


Culdocentsis.
Salpingectoma urgente.
Ingreso y determinacin de niveles deB-HCG.

FACTORES DE RIESGO
Anomalas tubarias

Enfermedad plvica inflamatoria 30-50%


Adherencias (endometriosis)
Antecedente de salpingoclasia
Alteraciones anatmicas

EMBARAZO ECTPICO
< 6500mUI

USG normal

Cuantificacin seriada
de HGC

Descenso

Dilatacin y legrado

Estabilizacin o
incremento

Laparoscopia Dx

64

Aumento

Repetir USG
27/01/2014

Diagnstico:

HGC > 1500 U/L


Sin evidencia de saco
Lquido libre en fondo de saco
Sensibilidad 63%
Especificidad 100%

Ankum WM., Mol Bw.,Van der Veen F.,Bossuyt PM.


Risk factors for ectopic pregnancy:a meta-analysis. Fertil Steril 1996:65:1093-9.

80.- Femenino de 20 aos, atendida en sala de urgencias ginecoobsttricas, Antecedente:


cursa embarazo de 38 SDG. Exploracin Fsica: en trabajo de parto. Repentinamente presenta
sangrado profuso transvaginal y dolor abdominal. La causa ms probable de la sintomatologa
de esta paciente es:

a) Laceracin vaginal por coito


b) Cervicitis
c) Placenta previa
d) Abruptio placentae

DESPRENDIMIENTO PREMATURO DE PLACENTA NORMOINSERTA (DPPNI):


Constituye la separacin de la placenta de su rea de insercin antes del 3 perodo del parto.
La mortalidad fetal es muy alta (superior al 15%) y la materna es tres veces superior a la
esperada.
La aparicin frecuente de SFA, prematuridad, anemia, etc., hace que el nmero de secuelas
tanto sensitivas como motoras sea alto. Desde el punto de vista materno, complicaciones
secundarias a la hemorragia, a las alteraciones de la coagulacin o bien a la embolia pulmonar
tambin tienen una tasa muy alta.
Desde la antigedad se identific al cuadro clnico caracterizado con la trada sintomtica de
hipertona, metrorragia y muerte fetal, con pronstico materno comprometido. En 1775, Ricci
diferenci la separacin prematura de una placenta de insercin normal, de aquella de
insercin baja; a la primera llam hemorragia accidental, y a la segunda hemorragia
imprevisible. Couvelaire introdujo el trmino de desprendimiento prematuro de placenta y
describi la aparicin de la apopleja uterina
(tero de Couvelaire)

Obstetricia. Scwarcz, Sala, Duverges. 7 edic. Edit. El Ateneo. (Biblioteca Fac. Med.
UNNE).

S-ar putea să vă placă și